You are on page 1of 107

Program Pengembangan Mutu dan Relevansi Program Studi

Halaman Sampul
Modul Pembelajaran Matakuliah

Teori Bilangan
Kode Matakuliah : 146H1103

Oleh:
Nur Erawaty,
Loeky Haryanto,
Syamsuddin Toaha

Dibiayai oleh DIPA Universitas Hasanuddin Tahun 2009


Sesuai dengan Surat Perjanjian Pelaksanaann Pekerjaan Modul Pembelajaran
Fak. MIPA Unhas No. 41/H4-LK.26/SP3-UH/2009 tertanggal 22 Juni 2009

Halaman Pengesahan

MODUL PEMBELAJARAN
Program Pengembangan Mutu dan Relevansi Program Studi
Fakultas Matematika dan Ilmu Pengetahuan Alam
Universitas Hasanuddin Tahun 2009

Nama/Kode Matakuliah

: Teori Bilangan/146H1103

Nama Lengkap Penulis Utama : Dra. Nur Erawaty, M.Si.


NIP

: 132 050 973

Pangkat/Golongan

: Penata Muda/IIIC

Jurusan/Program Studi

: Matematika/Matematika

Fakultas/Universitas

: Fakultas MIPA

Jangka Waktu Kegiatan

: 15 Mei 31 Juli 2009

Biaya

Rp 3, 500, 000, 00 oleh DIPA Universitas Hasanuddin


Tahun 2009

Makassar, 31 Juli.2009

Mengetahui:
Fakultas MIPA, Universitas Hasanuddin
Dekan

Prof. DR. H.Abd. Wahid Wahab, M.Sc.


NIP: 130535950

ii

Kata Pengantar
Teori Bilangan adalah salah satu dari beberapa cabang matematika klasik yang sudah
lama dipelajari dan dikembangkan oleh banyak matematikawan. Pada awalnya teori bilangan
dipelajari dan dikembangkan sebagai kesenangan dan pemenuhan rasa ingin tahu belaka,
tetapi saat ini beberapa cabang dari teori bilangan telah mendapatkan tempat sebagai alat dari
teknologi modern, misalnya dalam konstruksi kriptografi RSA.
Seperti halnya berbagai cabang matematika yang lain, konsep teori bilangan semakin
terlihat kaitannya dengan berbagai teori di cabang matematika yang lain. Fakta ini diperkuat
dengan terbuktinya Teorema Terakhir Fermat secara tak langsung sebagai hasil sampingan
dari studi yang sangat dalam terhadap beberapa konsep-konsep matematika yang baru ada
dan lahir di abad ke 20, padahal teorema tersebut sudah dipublikasikan sejak tahun 1673 dan
dianggap sebagai bagian dari teori bilangan.
Lepas dari penerapan modern di masa kini dan kaitannya dengan cabang matematika
yang lain, teori bilangan masih didominasi oleh pembuktian-pembuktian matematika. Dalam
hal ini, diperlukan kematangan yang cukup bagi pembaca untuk bisa mengerti alur logika
yang digunakan dalam pembuktian-pembuktian. Sebagai ilustrasi, pernyataan Teorema 1.15
adalah Jika p membagi ab maka p membagi a atau p membagi b. Tetapi yang dibuktikan
dalam pembuktian teorema ini adalah pernyataan yang ekuivalen (berdasarkan logika): Jika
p membagi ab dan p tidak membagi a, maka p membagi b.
Walaupun banyak pembuktian, materi yang disajikan dalam modul pembelajaran ini
masih merupakan materi pengenalan dasar-dasar Teori Bilanga. Meskipun para pembaca
dianjurkan untuk mempelajarinya secara berjenjang, tetapi Bab 4 yang bisa langsung
dipelajari setelah Bab 1 dan Bab 2, tanpa melalui Bab 3.
Modul pembelajaran Teori Bilangan ini dirancang untuk satu semester dan dibuat
sebagai bagian dari Program Pengembangan Mutu dan Relevansi Program Studi tahun 2009
pada program studi Matematika, Jurusan Matematika, Fakultas MIPA Universitas
Hasanuddin. Untuk ini, tim penulis mengucapkan banyak terimakasih atas segala bantuan
yang diberikan oleh semua pihak, khususnya oleh fakultas MIPA.
Makassar, 31 Juli 2009
NE, LH & ST
iii

Daftar Isi
Halaman Sampul.......................................................................................................... i
Halaman Pengesahan .............................................................................................. ii
Kata Pengantar ......................................................................................................... iii
Daftar Isi....................................................................................................................... iv
Bagan Ketergantungan Antar Bab .................................................................. v
1. Keterbagian............................................................................................................. 1
1.1. Pendahuluan............................................................................................................. 1
1.2. Keterbagian .............................................................................................................. 2
1.3. Bilangan Prima....................................................................................................... 13
2. Kongruensi ............................................................................................................ 23
2.1 Kongruensi.............................................................................................................. 23
2.2 Solusi Kongruensi................................................................................................... 30
2.3 Kongruensi Derajat 1 .............................................................................................. 32
2.4 Fungsi Euler (n) .................................................................................................... 38
2.5 Kongruensi Derajat Lebih Tinggi ........................................................................... 41
2.6 Modulo Perpangkatan Bilangan Prima ................................................................... 44
2.7. Kongruensi Modulo Prima..................................................................................... 52
2.8 Kongruensi Derajat Dua, Modulo Prima ................................................................ 56
2.9 Residu Pangkat........................................................................................................ 57
2.10 Teori Bilangan Ditinjau Secara Aljabar................................................................ 62
2.11 Grup Perkalian, Gelanggang dan Lapangan ......................................................... 67
3. Resiprositi Kuadratik ....................................................................................... 73
3.1 Residu Kuadratik .................................................................................................... 73
3.2 Resiprositi Kuadratik .............................................................................................. 78
3.3 Simbol Jacobi.......................................................................................................... 83
4. Beberapa Persamaan Diophantine ............................................................. 89
4.1 Persamaan Diophantine .......................................................................................... 89
4.2 Persamaan ax + by = c ............................................................................................ 90
4.3 Solusi-Solusi Bulat Positif ...................................................................................... 91
4.4 Persamaan Linear Yang Lain.................................................................................. 92
4.5 Persamaan x2 + y2 = z2............................................................................................. 93
4.6 Persamaan x4 + y4 = z2............................................................................................. 95
4.7 Jumlah Dari Empat Kuadrat.................................................................................... 98
Daftar Pustaka....................................................................................................... 101
Indeks........................................................................................................................... 102

iv

Bagan Ketergantungan
Antar Bab

Bab 1
Keterbagian

Bab 2
Kongruensi

Bab 3
Resiprositi Kuadratik

Bab 4
Beberapa Persamaan
Diophantine

1. Keterbagian
1.1. Pendahuluan
Bahasan utama dalam teori bilangan adalah tentang bilangan-bilangan bulat
positif. Tetapi teori yang terlibat tidak terbatas pada bilangan-bilangan bulat positif, atau
bahkan terbatas pada bilangan-bilangan bulat. Mungkin saja suatu hasil tentang bilanganbilangan bulat diperoleh dari teori bilangan-bilangan kompleks atau dari teori turunan
suatu fungsi.
Teori bilangan berpijak pada hasil-hasil pembuktian dari berbagai ide dan metoda.
Dua di antara hasil-hasil ini memerlukan perhatian khusus. Hasil pertama adalah, setiap
himpunan tak kosong dari bilangan-bilangan bulat memuat unsur terkecil. Hasil yang
kedua adalah induksi matematis. Hasil yang kedua ini merupakan akibat dari hasil yang
pertama.
Para pembaca dianggap sudah cukup menguasai perumusan berbagai pernyataan
matematis. Sebagai contoh, para pembaca sudah mengetahui bahwa untuk setiap pasang
pernyataan matematis A dan B, pernyataan-pernyataan matematis berikut ekuivalen:
Jika A maka B,
Dari A diturunkan B,
Jika A benar, maka B benar
A adalah syarat cukup untuk B
B adalah syarat perlu untuk A.
Semua pernyataan ini lazim ditulis dengan lambang
A B.
Jika dari A bisa diturunkan B dan dari B bisa diturunkan A, kita mengatakan A adalah
syarat cukup dan perlu untuk B dan hal ini lazim ditulis dengan lambang
AB
dan diucapkan A jika dan hanya jika B.
Demikian pula, kata jika dalam suatu definisi - seperti dalam Definisi 1.1 di bagian
mendatang - seringkali bermakna jika dan hanya jika (disingkat jhj).

Lebih jauh, banyak pernyataan matematis yang bisa dinyatakan dengan lambanglambang baku. Sebagai contoh, karena Z adalah lambang baku untuk himpunan semua
bilangan-bilangan bulat, pernyataan matematis
z adalah bilangan bulat
bisa dinyatakan secara singkat dengan lambang
z Z.
Demikian pula, beberapa pernyatan baku terkait dengan Q (himpunan semua bilangan
rasional) dan R (himpunan semua bilangan real).
1.2. Keterbagian
Definisi 1.1
Bilangan b Z habis dibagi bilangan bulat a 0, ditulis a|b; jika terdapat bilangan
x Z sedemikian rupa sehingga b = ax.
Ungkapan lain untuk menyatakan a|b adalah a habis membagi b, a adalah pembagi b
dan b adalah kelipatan a. Jika a tidak membagi b, yaitu jika pernyataan a|b adalah salah,
kita menulis
ab.
Perhatikan, untuk setiap bilangan bulat a berlaku a|0.
Teorema 1.1
Jika a, b, c Z, maka pernyataan-pernyataan berikut berlaku.
(1) a|b a|bc.
(2) a|b & b|c a|c.
(3) a|b & a|c a|(bx + cy), untuk setiap x, y Z.
(4) a|b & b|a a = b.
(5) a|b & a > 0 & b > 0 a b.
Bukti.
Langsung diperoleh dari definisi keterbagian.

Sifat (3) dalam teorema di atas bisa diperluas ke pernyataan

Apabila a, b1, b2, , bn Z dan a|b1 & a|b2 & & a|bn, maka untuk sembarang x1, x2,
, xn Z berlaku
n

a|

b x
i =1

i i

Demikian pula, sifat (2) masih bisa diperluas dengan cara yang serupa.
Teorema 1.2 (Teorema Algoritma Pembagian)

Untuk setiap pasangan a, b Z dengan a > 0, terdapat pasangan q, r Z dengan


b = qa + r dan 0 r < a. Jika ab, maka r memenuhi ketaksamaan murni 0 < r < a.
Bukti.
Perhatikan barisan aritmatika tak hingga dengan bilangan pembeda a > 0 berikut
, b 3a, b 2a, b a, b, b + a, b + 2a, b + 3a, .

(1.1)

Misalkan r adalah bilangan tak negatif terkecil di antara bilangan-bilangan bulat dalam
barisan di atas. Dalam kasus a|b, salah satu di antara bilangan-bilangan tersebut adalah 0
sehingga dalam kasus ini jelas r = 0. Dalam kasus ab, setiap bilangan dalam barisan tak
ada yang sama dengan 0. Jadi r > 0. Lebih jauh, terdapat suatu bilangan bulat k sehingga

r = b + ka. Karena r adalah bilangan bulat positif terkecil dalam barisan tersebut, setelah
dikurangi a, hasilnya adalah bilangan negatif r a = b + (k 1)a < 0. Ini berarti r < a.
Jadi terbukti: dalam kasus a|b, r = 0 sedangkan dalam kasus ab berlaku 0 < r < a.

ra

b + (k 1)a

r+a

b + (k + 1)a

r = b + ka

Ilustrasi Pembuktian Dalam Kasus ab

Teorema 1.2 di atas mensyaratkan a > 0, walaupun sebenarnya syarat tersebut bisa
diperlemah dan teorema bisa diperluas sebagai berikut:
Untuk setiap a, b Z dengan a 0, terdapat sepasang bilangan bulat q dan r yang
memenuhi b = qa + r dengan 0 r < |a|.
Teorema 1.2 sering disebut Teorema Algoritma Pembagian walaupun teorema
tersebut memuat kata terdapat q, r Z yang memberi kesan bahwa Teorema 1.2
adalah teorema klaim keberadaan, bukan algoritma. Walaupun demikian Teorema 1.2,

membuka rintisan sebuah algoritma untuk mendapatkan bilangan q, r Z dengan hanya


menggunakan sebagian yang diperlukan (bagian tak negatif) dari barisan tak hingga (1.1).
Dalam definisi berikut, digunakan notasi Z+ untuk melambangkan himpunan semua
bilangan-bilangan bulat positif (yang sering disebut bilangan-bilangan asli dan diberi
lambang N).
Definisi 1.2

Suatu bilangan a Z+ disebut pembagi persekutuan dari b dan c jika a|b dan a|c.
Lebih jauh jika b 0 atau c 0 dan untuk setiap pembagi persekutuan a yang
membagi b dan c berlaku a a, maka a disebut pembagi persekutuan terbesar1) dari
b dan c.
Pembagi persekutuan terbesar dari b dan c dilambangkan
(b, c).
Definisi pembagi persekutuan terbesar dari dua bilangan bulat secara alamiah bisa
diperluas ke lebih dari dua bilangan bulat. Pembagi persekutuan terbesar dari n bilangan
bulat b1, b2, , bn dilambangkan
(b1, b2, , bn).
Berdasarkan definisi, setiap bilangan bulat membagi bilangan 0. Jadi ada tak hingga
faktor dari 0. Tetapi hanya ada sebanyak hingga faktor dari bilangan bulat tak nol. Jadi
jika b 0 atau c 0 maka hanya ada sebanyak hingga pembagi persekutuan yang berbeda
dari b dan c.
Perhatikan, jika a|b dan a|c, maka a|b dan a|c. Karena fakta ini, kita memilih
bilangan positif (b, c) sebagai pembagi persekutuan terbesar dari b dan c.
Contoh 1.1
Karena 15|300 dan 15|225, 15 adalah pembagi persekutuan dari 300 dan 225.
Demikian pula, 15|300 dan 15|225. Tetapi 15 (300, 225). Sesungguhnya,
75 = (300, 225).

1)

Di Indonesia, sekolah-sekolah tingkat menengah ke bawah menggunakan istilah faktor pembagi terbesar
b dan c yang disingkat FPB(b, c) walaupun istilah faktor memiliki makna agak berbeda dengan istilah
pembagi.

Teorema 1.3

Jika g = (b, c), maka terdapat bilangan bulat x0 dan y0 sedemikian rupa sehingga
g = bx0 + cy0.
Bukti.
Pandang himpunan
H = {bx + cy Z | x, y Z}
yang berisi bilangan-bilangan bulat positif dan negatif. Pilih x0, y0 Z sedemikian rupa
sehingga l = bx0 + cy0 adalah bilangan positif terkecil dalam H. Pertama kali dibuktikan
bahwa l adalah pembagi persekutuan b dan c. Dalam hal ini, hanya dibuktikan l|b (sebab
pembuktian l|c dikerjakan dengan cara yang analog).
Andaikan lb. Menurut bagian kedua dari Teorema 1.2, terdapat bilangan q, r Z
dengan b = lq + r dan 0 < r < l. Jadi

r = b lq = b (bx0 + cy0)q = b(1 qx0) + c(qy0)


dan ini membuktikan r H dengan 0 < r < l. Hal ini kontradiksi dengan ketentuan bahwa

l adalah bilangan positif terkecil dalam H. Karena timbul kontradiksi, pengandaian lb


harus diingkari menjadi l|b.
Karena g = (b, c), terdapat B, C Z sedemikian rupa sehingga b = gB dan c = gC.
Sebagai akibatnya, l = bx0 + cy0 = g(Bx0 + Cy0). Jadi g|l sehingga menurut bagian 5 dari
Teorema 1.1, g l. Dari lain pihak, karena g = (b, c), l|b dan l|c, disimpulkan l g. Ini
berarti g = l = bx0 + cy0.
Teorema 1.4

Misalkan g, b, c Z dengan g 0. Ketiga pernyataan berikut ekuivalen:


(1) g = (b, c);
(2) g adalah bilangan positif terkecil dalam H = {bx + cy Z | x, y Z};
(3) Untuk setiap a Z, jika a|b dan a|c, maka a|g.

Bukti.
(1) (2) adalah bagian dari bukti Teorema 1.3 sedangkan pembuktian (2) (3)
diperoleh dari bagian (3) Teorema 1.1 yang menyatakan jika a|b dan a|c maka a|(bx + cy),
di mana x, y Z.
5

Teorema 1.5

Misalkan b1, b2, , bn Z tidak semuanya nol. Jika g = (b1, b2, , bn), maka
terdapat x1, x2, , xn Z sedemikian rupa sehingga
g = b1x1 + b2x2 + + bnxn
dan ketiga pernyatan berikut ekuivalen:
(1) g = (b1, b2, , bn);
n

(2) g adalah bilangan positif terkecil dalam H = { bi xi Z | xi Z};


i =1

(3) Jika a Z dengan a|b1, a|b2, , a|bn, maka a|g.

Ungkapan lain untuk menyatakan bagian 2 dari Teorema 1.4 (yang juga bisa diperluas ke
Teorema 1.5) adalah

g = min{bx + cy Z | x, y Z}.
Teorema 1.6

Untuk setiap m Z dengan m > 0, berlaku


(ma, mb) = m(a, b).

Bukti.
Dari bagian (2) Teorema 1.4, diperoleh
(ma, mb) = bilangan positif terkecil dari semua bentuk max + mby;
= m( bilangan positif terkecil dari semua bentuk ax + by);
= m(a, b).
Teorema 1.7

Jika d|a dan d|b dan d > 0, maka

( da , db ) = d1 (a, b).

( )

a b
,
= 1.
Lebih jauh, jika g = (a, b) maka
g g
Bukti.

Pada Teorema 1.6, ganti m, a dan b masing-masing dengan d, a/d dan b/d dan kemudian
bagi kedua ruas dengan d. Akhirnya jika d diganti g = (a, b), maka diperoleh kesamaan
yang kedua.

Teorema 1.8

Jika (a, m) = 1 dan (b, m) = 1, maka (ab, m) = 1.


Bukti.
Menurut Teorema 1.3, terdapat x0, y0, x1, y1 Z sedemikian rupa sehingga
ax0 + my0 = 1 = bx1 + my1
sehingga ax0 = 1 my0 dan bx1 = 1 my1. Apabila x2 = x0x1, maka
abx2 = (ax0)(bx1) = (1 my0)( 1 my1) = 1 my0 my1 + m2y0y1 = 1 my2,
di mana y2 = y0 + y1 my0y1. Dari kesamaan di atas, diturunkan
abx2 + my2 = 1.
Selanjutnya Teorema 1.1 bagian (3) menyatakan bahwa setiap pembagi dari ab dan m
adalah pembagi dari abx2 + my2, yaitu pembagi dari 1. Karena pembagi dari 1 adalah 1
dan 1, maka jelas (ab, m) = 1.
Definisi 1.3

Dua bilangan a, b Z disebut relatif saling prima jika (a, b) = 1. Lebih umum,
bilangan-bilangan a1, a2, , an dikatakan relatif saling prima jika (a1, a2, , an) = 1.
Kita mengatakan a1, a2, , an sepasang-sepasang relatif saling prima jika untuk
setiap pasang indeks i {1, 2, , n} dan j {1, 2, , n} dengan i j berlaku (ai,
aj) = 1.
Teorema 1.9

Misalkan a,b Z. Untuk setiap x Z berlaku


(a, b) = (b, a) = (a, b) = (a, b +ax).
Bukti.
Namakan d = (a, b) dan g = (a, b +ax). Jelas (a, b) = (b, a) = (a, b) = d. Selanjutnya dari
Teorema 1.1 bagian (3) dan (4), d|g dan g|d sehingga d = g.
Teorema 1.10

Misalkan c|ab dan (b, c) = 1, maka c|a.


Bukti.

Dari Teorema 1.6, (ab, ac) = a(b, c) = a. Karena c|ab dan c|ac, menurut Teorema 1.4,
c|a.
Definisi 1.2 tidak memberikan cara yang konstruktif untuk mencari pembagi
persekutuan terbesar (a, b). Teorema mendatang memberikan cara konstruktf tersebut.
Tetapi sebelumnya kita buktikan lema berikut.
Lemma 1.1

Jika b = qc +r, maka (b, c) = (c, r).


Bukti.
Jika kita bisa membuktikan bahwa pembagi-pembagi persekutuan dari b dan c sama
dengan pembagi-pembagi persekutuan c dan r, maka otomatis (b, c) = (c, r). Sekarang
jika d|b dan d|c, maka (bagian (3) Teorema 1.1) d membagi b qc = r. Terbukti setiap
pembagi persekutuan b dan c juga merupakan pembagi persekutuan c dan r. Sebaliknya
dengan alasan yang sama, jika d|c dan d|r, maka d membagi qc + r = b.
Karena (b, c) = (b, c) = (b, c) = (b, c), kita bisa menganggap b > 0 dan c > 0.
Juga jika b < c, bentuk b = qc + r adalah b = 0c + b sehingga dalam teorema berikut
dianggap b c.
Teorema 1.11 (Algoritma Euklid)

Apabila diberikan bilangan bulat b = r0 dan c = r1 dengan 0 < c < b, maka dengan
mengulang algoritma pembagian (Teorema 1.2) beberapa kali secara berurutan,
diperoleh persamaan-persamaan
r0 = q1r1 + r2,

0 < r2 < r1,

r1 = q2r2 + r3,

0 < r3 < r2,

r2 = q3r3 + r4,

0 < r4 < r3,

rn2 = q n1r n1 + rn,

0 < rn < rn1,

r n1 = qnrn.
di mana (b, c) = (r0, r1) = rn adalah merupakan sisa (residu) tak nol terakhir dalam
proses pembagian di atas.

Bukti.
Dalam kasus b|c, berlaku b = q1c. Ini adalah bentuk r n1 = qnrn dengan n = 1. Dalam
kasus ini diperoleh (b, c) = (r0, r1) = r1.
Dalam kasus bc, tulis r n1 = qnrn + rn+1 dengan rn+1 = 0. Karena r0 > r1 > r2 adalah
barisan bilangan-bilangan bulat positif yang menurun murni, proses pembagian pada
suatu saat berhenti. Ini membuktikan keberadaan indeks n dengan rn+1 = 0 dan rn > 0
adalah sisa tak nol (residu) terakhir dalam proses pembagian di atas. Akhirnya dengan
menerapkan Lemma 1.1 di atas beberapa kali diperoleh
(b, c) = (r0, r1) = (r1, r2) = = (rn1, rn) = (rn, r0).
Solusi yang mudah diterapkan secara manual belum tentu mudah dan efisien jika
diterapkan dalam bentuk program komputer. Agar bisa diimplementasikan sebagai
program komputer, sebuah perumusan kuantitatif sangat membantu. Hal ini disebabkan
setiap perumusan secara kuantitatif secara implisit menunjukkan urutan langkah-langkah
(prosedur) untuk mendapatkan nilai yang dirumuskan secara kuantitatif tersebut.
Teorema 1.3 hanya mengklaim keberadaan dua bilangan bulat x0 dan y0 yang
memenuhi (a, b) = ax0 + by0, tidak memberikan cara menentukan nilai x0 dan y0. Berikut
diberikan sebuah cara konstruktif untuk mencari (a, b) dan sekaligus mendapatkan x0 dan
y0. Cara ini memberikan solusi untuk menentukan kedua bilangan bulat x0 dan y0 melalui
perumusan kuantitatif yang bersifat rekursif.
Akibat Teorema 1.11 (Perluasan Algoritma Euklid)

Dengan algoritma Euklid seperti yang dinyatakan dalam Teorema 1.11, tetapkan
s0 = 1,

s1 = 0,

t0 = 0,

t1 = 1

dan untuk setiap k yang memenuhi 1 < k n, secara rekursif tetapkan


sk+1 = qksk + sk1,

tk+1 = qktk + tk1

maka
rk = (1)ksk a + (1) k1tkb.
Pada khususnya, jika

x0 = (1) sn

dan

y0 = (1)

n+1

tn ,

maka
rn = (b, c) = ax0 + by0.

Kita tak akan membuktikan Akibat Teorema 1.11, tetapi memberi ilustrasi
bagaimana perumusan di atas berlaku.
Contoh 1.2

Untuk mencari (1668, 486), dilakukan proses pembagian


1668 = 3486 + 210;
486 = 2210 + 66;
210 = 366 + 12;
66 = 512 + 6
12 = 26.
Disimpulkan (1668, 486) = 6 sebab 6 merupakan sisa tak nol (residu) terakhir hasil
algoritma pembagian dalam Teorema 1.11 di atas.
Untuk melihat berlakunya Akibat Teorema 1.11, tulis
r0 = 1668 = 11668 + 0486,
r1 = 486 = 01668 + 1486,
kemudian tetapkan s0 = 1, s1 = 0, t0 = 0 dan t1 = 1, yaitu koefisien-koefisien jumlahan
di atas. Selanjutnya karena
r2 = 210 = 1668 3486 = (1)211668 + (1)3486,
tetapkan s2 = 1 dan t2 = 3. Selanjutnya karena
r3 = 66
= 486 2210
= 486 2(1668 3486)
= 21668 + 7486,
= (1)321668 + (1)27486,
tetapkan s3 = 2 dan t3 = 7. Demikian proses terus berlanjut sampai diperoleh sisa tak
nol terakhir r5 = 6 sedangkan r6 = 0. Lebih jauh, proses di atas menghasilkan
6 = (1668, 486) = (37)1668 + (127)486.
Proses di atas bisa digambarkan melalui diagram berikut.
k
rk
qk
sk
tk

0
1668
1
0

1
486
3
0
1

2
210
2
1
3

3
66
3
2
7

4
12
5
7
24

5
6
2
37
127

6
0
81
278

10

Dalam algoritma di atas, qkrk = rk1 rk+1 > 0 sehingga untuk setiap k, qk > 0.
Diberikan sebuah bilangan bulat positif a dan himpunan semua pembagi a, maka
untuk setiap pasang bilangan b dan c yang membagi a, terdapat pengawanan 1-1 antara
pembagi persekutuan terbesar (a/b, a/c) dengan kelipatan persekutuan terkecil [b, c] yang
didefinisikan sebagai berikut.
Definisi 1.4

Bilangan a Z+ disebut kelipatan persekutuan dari n bilangan-bilangan bulat a1, a2,


, an jika untuk setiap i = 1, 2, , n berlaku ai|a. Lebih jauh, jika untuk setiap
kelipatan persekutuan a yang lain berlaku a a, bilangan b ini disebut kelipatan
persekutuan terkecil dari a1, a2, , an. Dalam hal ini ditulis a = [a1, a2, , an]
Teorema 1.12

Jika b adalah sebuah kelipatan persekutuan dari a1, a2, , an, maka [a1, a2, , an]|b.
Dengan kata lain, jika a = [a1, a2, , an], maka setiap kelipatan persekutuan dari a1,
a2, , an berbentuk ka, untuk suatu k {0, 1, 2, }.
Bukti.
Misalkan b adalah kelipatan persekutuan dari a1, a2, , an. Menurut Teorema 1.2,
terdapat bilangan k, r Z yang memenuhi b = ka + r dengan 0 < r < a. Andaikan r 0.
Karena untuk setiap i {1, 2, , n} berlaku ai|a dan ai|b, maka ai|r. Jadi r adalah
kelipatan persekutuan a1, a2, , an yang lebih kecil dari a, kontradiksi dengan fakta a =
[a1, a2, , an]. Jadi pengandaian harus diingkari dan r = 0.
Teorema 1.13

Untuk setiap a, b, m Z dengan m > 0 berlaku [ma, mb] = m[a, b]. Lebih jauh,
[a, b](a, b) = |ab|.
Bukti.
Selain merupakan kelipatan persekutuan dari ma dan mb, [ma, mb] juga merupakan
kelipatan dari m sehingga [ma, mb] = mh1 untuk suatu h1 Z. Jika kita tulis [a, b] = h2,
maka a|h2 dan b|h2. Jadi ma|mh2 dan mb|mh2 dan mh2 merupakan kelipatan persekutuan
dari ma dan mb sehingga menurut Teorema 1.12, [ma, mb]|mh2, yaitu mh1|mh2. Jadi h1|h2.

11

Dari lain pihak, ma|mh1 dan mb|mh1 sehingga a|h1 dan b|h1 yang berakibat [a, b]|h1, yaitu
h2|h1. Terbukti h1 = h2 = [a, b] sehingga [ma, mb] = mh1 = m[a, b].
Karena [b, c] = [b, c] = [b, c] = [b, c] > 0 jika a 0 atau b 0, maka bagian kedua
cukup dibuktikan untuk kasus a > 0 dan b > 0. Pertama kali anggap (a, b) = 1 dan amati
bahwa [a, b] adalah kelipatan dari a, katakana [a, b] = ma. Jadi b|ma dan karena (a, b) = 1,
maka menurut Teorema 1.10, b|m. Sebagai akibatnya, b m dan ba ma = [a, b]. Tetapi
ba adalah kelipatan persekutuan dari a dan b sehigga ba [b, a] sehingga dalam kasus
(a, b) = 1, teorema terbukti. Dalam kasus g = (a, b) > 1, Teorema 1.7 menyatakan bahwa

( ag , bg ) = 1.
Dengan menerapkan hasil sebelumnya, diperoleh
a , b a , b = a b .
g g g g
g g
2
Setelah kedua ruas dikali g , diperoleh
a b
a b
g , g ,
= ab.
g g g g
Dengan menggunakan bagian pertama teorema ini dan Teorema 1.6, diperoleh

( ) ( )( )
( )

[a, b](a, b) = ab.


Soal-Soal

1. Gunakan algoritma Euklid untuk mencari pembagi persekutuan terbesar dari


a. 7469
c. 2947

dan
dan

3464
3997

b. 2689
d. 1109

dan
dan

4001
4999

2. Untuk setiap pasangan bilangan a dan b dalam soal butir 1 di atas, gunakan perluasan
algoritma Euklid untuk mencari sepasang bilangan bulat x0 dan y0 sedemikian rupa
sehingga (a, b) = ax0 + by0.
3. Tentukan sepasang bilangan bulat x dan y sedemikian rupa sehingga
a. 243x + 198y = 9
c. 43x + 64y = 1

b. 71x 51y = 1
d. 93x 81y = 3

4. Tentukan KPK (kelipatan persekutuan terkecil) dari


a. 482

dan

1687

b.

60

dan

61

5. Buktikan bahwa hasil kali tiga bilangan bulat yang berurutan habis dibagi 6 dan hasil
kali empat bilangan bulat berurutan habis dibagi 24.

12

6. Tunjukkan tiga buah bilangan yang relatif saling prima tetapi tidak saling prima
sepasang-sepasang.
7. Dua bilangan bulat dikatakan sama paritasnya jika keduanya bersama-sama genap
atau bersama-sama ganjil sedangkan jika salah satu genap dan yang lain ganjil, kedua
dikatakan berlawanan paritasnya. Diberikan sepasang bilangan, tunjukkan bahwa
jumlah dan selisih keduanya selalu sama paritasnya.
8. Untuk setiap bilangan bulat n, buktikan n2 n habis dibagi 2, n3 n habis dibagi 6
dan n5 n habis dibagi 30.
9. Untuk setiap bilangan bulat ganjil n, buktikan n2 1 habis dibagi 8.
10. Buktikan, jika x dan y adalah bilangan bulat ganjil, x2 + y2 adalah bilangan genap
yang tak habis dibagi 4.
11. Buktikan, jika a dan b adalah dua bilangan bulat yang memenuhi [a, b] = (a, b) maka
a = b.
12. Buktikan tak ada bilangan bulat x dan y yang memenuhi x + y = 100 dan (x, y) = 3.
13. Buktikan ada tak hingga pasangan bilangan bulat x dan y yang memenuhi x + y = 100
dan (x, y) = 5.
14. Diberikan dua buah bilangan bulat s dan g > 0. Buktikan terdapat bilangan bulat x dan
y yang memenuhi x + y = s dan (x, y) = g jika dan hanya jika s|g.
15. Cari pasangan bilangan bulat a dan b yang memenuhi (a, b) = 10 dan [a, b] = 100
secara bersamaan. Tentukan semua solusi yang mungkin.
16. Cari semua triple bilangan-bilangan bulat a, b dan c yang memenuhi (a, b, c) = 10
dan [a, b, c] = 100 secara bersamaan. Tentukan semua solusi yang mungkin.
17. Diberikan dua buah bilangan bulat positif g dan l. Buktikan terdapat bilangan bulat x
dan y yang memenuhi (x, y) = g dan [x, y] = l jika dan hanya jika g|l.
1.3. Bilangan Prima
Definisi 1.5

Sebuah bilangan bulat p disebut bilangan prima jika p tak memiliki pembagi d yang
memenuhi 1 < d < p. Sebuah bilangan bulat a > 1 yang bukan bilangan prima disebut
bilangan majemuk.

13

Sebagai contoh, 2, 3, 5, 7 dan 17 adalah bilangan-bilangan prima sedangkan 4, 6, 8, 10


dan 16 adalah bilangan-bilangan majemuk.
Teorema 1.14

Setiap bilangan bulat n > 1 merupakan hasil kali faktor-faktor prima (mungkin hanya
terdiri atas satu faktor).
Bukti.
Jika n sendiri adalah bilangan prima, maka n berbentuk hasil kali dari satu faktor
perkalian. Jika bukan prima, n bisa difaktorkan, katakan n = n1n2, dengan 1 < n1 < n dan 1
< n2 < n. Jika n1 adalah bilangan prima, biarkan apa adanya. Jika bukan prima, n1 = n3n4
untuk suatu bilangan bulat n3 dan n4 dengan 1 < n3 < n1 dan 1 < n4 < n1, dst. Argumentasi
yang sama dikenakan pada n2. Proses menyatakan setiap faktor majemuk sebagai hasil
kali bilangan-bilangan bulat pada suatu saat pasti berhenti sebab banyaknya faktor tak
akan melebihi n, mengingat setiap faktor adalah bilangan-bilangan bulat > 1. Jadi kita
bisa menulis n sebagai hasil kali bilangan-bilangan prima dan karena faktor-faktor prima
ini tidak selalu berbeda, hasilnya bisa ditulis sebagai
n = p11 p2 2 pR R

(1.2)

dengan p1, p2, , pR adalah bilangan-bilangan prima yang berbeda sedangkan 1, 2, ,


R adalah bilangan-bilangan bulat positif.

Ekspresi (1.2) disebut representasi kanonik dari bilangan bulat n sebagai hasil
kali bilangan-bilangan prima. Representasi n yang lain (juga disebut faktorisasi) adalah
dengan menulis
n = p1p2 pr

(1.3)

di mana p1, p2, , pr adalah bilangan-bilangan prima (tak harus berbeda, apa sebabnya?)
Sekilas terkesan jelas bahwa setiap bilangan bulat bisa direpresentasikan sebagai
hasil kali beberapa (mungkin hanya satu) faktor-faktor prima secara tunggal, kecuali
urutan faktor-faktor pi prima yang bisa berbeda. Padahal kenyataannya ketunggalan
representasi bilangan-bilangan bulat ini memerlukan bukti (Teorema 1.16). Bahkan pada
beberapa jenis himpunan dari bilangan-bilangan yang juga bersifat tertutup terhadap
operasi tambah dan kali aritmatik, representasi atas faktor-faktor prima tidak selalu
tunggal.

14

Kita akan sedikit menyimpang dari topik untuk menunjukkan dua contoh di mana
representasi atas faktor-faktor prima tidak selalu tunggal. Contoh pertama adalah
himpunan semua bilangan bulat genap positif
E = {0, 2, 4, }.

Anggap untuk sementara bahwa sebuah bilangan harus diartikan sebagai unsur dari E.
Himpunan E tertutup terhadap operasi tambah dan kali aritmatik, sebab hasil
tambah dan hasil kali dua bilangan bulat genap menghasilkan bilangan bulat genap.
Bilangan-bilangan prima dalam E adalah bilangan-bilangan 2, 4, 6, 10, 14, (kecuali
2, setiap bilangan prima di C adalah kelipatan 2 dari bilangan-bilangan prima dalam Z).
Sebagai contoh, bilangan 8 adalah bilangan majemuk sebab 8 = 24 sedangkan 10 adalah
bilangan prima. Sekarang kita lihat bahwa
60 = 230 = 610,
yaitu bilangan 60 bisa ditulis dengan dua bentuk hasil kali faktor-faktor prima.
Contoh kedua adalah himpunan bilangan-bilangan kompleks
C = {a + b 6 | a, b Z}.

Perhatikan dengan menetapkan b = 0, diperoleh sub himpunan Z C. Himpunan C


tertutup terhadap operasi tambah aritmatik (antara dua bilangan kompleks):
(a1 + b1 6 ) + (a2 + b2 6 ) = (a1 + a2) + (b1 + b2) 6 C,
sebab a1 + a2 Z dan b1 + b2 Z. Demikian pula, C tertutup terhadap operasi tambah
aritmatik (antara dua bilangan kompleks):
(a1 + b1 6 )(a2 + b2 6 ) = (a1a2 6b1b2) + (a1b2 + a2b1) 6 C,
sebab a1a2 6b1b2 Z dan a1b2 + a2b1 Z.
Pertama kali kita definisikan norma N dari setiap unsur a + b 6 C sebagai
N(a + b 6 ) = (a + b 6 )(a b 6 ) = a2 + 6b2

(Jadi N ekuivalen dengan kuadrat modulus bilangan kompleks atau panjang vektor).
Tampak jika a + b 6 0, 1, 1 maka N(a + b 6 ) > 1. Bahkan jika b 0, maka
N(a + b 6 ) 6.

(1.4)

Sebuah bilangan a + b 6 C dikatakan majemuk (bisa difaktorkan) jika bisa


dinyatakan sebagai hasil kali seperti berikut:

15

a + b 6 = (x1 + y1 6 )( x2 + y2 6 )

(1.5)

dengan N(x1 + y1 6 ) > 1 dan N( x2 + y2 6 ) > 1. Jadi 1 dan 1 tak bisa berperan
sebagai faktor (sebab seandainya dibolehkan, N(1) = N(1) = 1). Pembatasan ini
diperlukan untuk menghindari faktorisasi trivial semacam
a + b 6 = (1)(a + b 6 ) = (1)(a b 6 ).

Jadi banyaknya faktor dalam representasi unsur-unsur C selalu berhingga, kurang dari
jumlah norma faktor-faktornya. Lebih jauh, dari bentuk (1.5) bisa diturunkan
N(a + b 6 ) = N(x1 + y1 6 )N( x2 + y2 6 ).

(1.6)

Sebuah bilangan a + b 6 C dengan norma N(a + b 6 ) > 1 disebut prima


jika tidak bisa difaktorkan sesuai ekspresi (1.5). Sebagai contoh, 5 C adalah prima.
Alasan pertama, 5 tidak bisa dinyatakan sebagai hasil kali dua bilangan bulat dengan
norma > 1. Kedua, seandainya 5 bisa difaktorkan atas dua bilangan dalam C, yaitu
5 = (x1 + y1 6 )( x2 + y2 6 ),
maka menurut (1.6)
25 = N(x1 + y1 6 )N( x2 + y2 6 ).
Ruas kanan berbentuk hasil kali dua norma, yaitu hasil kali dua bilangan bulat positif > 1.
Satu-satunya faktorisasi 25 atas dua bilangan bulat positif N(x1 + y1 6 ) > 1 dan N(x2 +
y2 6 ) > 1 yang mungkin adalah apabila N(x1 + y1 6 ) = 5 dan N(x2 + y2 6 ) = 5, hal

yang kontradiksi dengan (1.4). Jadi terbukti 5 adalah prima.


Akhirnya kita buktikan bahwa 10 C memiliki lebih dari satu representasi:
10 = 25 = (2 +

6 )( 2 6 ).

Salah satu faktorisasi - dalam hal ini 10 = 25 - harus berbentuk faktorisasi atas bilanganbilangan prima. Faktorisasi yang lain - dalam hal ini 10 = (2 +

6 )( 2 6 ) - tidak

perlu berbentuk faktorisasi atas dua bilangan prima (walaupun sesungguhnya kedua
faktor 2 +

6 dan 2 6 adalah prima) sebab sudah cukup bagi kita untuk

menunjukkan adanya faktorisasi lain yang berbeda dari faktorisasi 10 = 25.

16

Kita kembali pada faktorisasi tunggal bilangan-bilangan bulat dalam Z 2).


Teorema 1.15

Untuk setiap bilangan prima p dan setiap a, b Z berlaku implikasi:


jika p|ab maka p|a atau p|b. Lebih umum lagi, untuk setiap bilangan prima p dan
bilangan-bilangan a1, a2, , an Z berlaku implikasi: jika p|a1a2an maka terdapat
paling sedikit satu indeks i {1, 2, , n} dengan p|ai.
Bukti.

Misalkan p|ab. Jika pa maka menurut Teorema 1.10, p|b 3). Untuk kasus kedua yang
lebih umum, pembuktian bagian pertama ini merupakan langkah pertama (basis) dalam
induksi matematis. Langkah berikutnya: anggap teorema benar apabila p adalah pembagi
bilangan bulat yang merupakan hasil kali faktor-faktor prima yang banyaknya kurang
dari n. Sekarang misalkan p|a1a2an. Namakan c = a2an sehingga p|a1c. Dalam kasus
p|a1, teorema otomatis benar. Dalam kasus pa1, kembali menurut Teorema 1.10, p|c.

Karena c merupakan hasil kali faktor-faktor prima yang banyaknya kurang dari n,
hipotesis induksi berlaku: p membagi salah satu dari faktor-faktor prima (salah satu dari
a2, a3, , an) dari c.
Teorema 1.16 (Teorema Dasar Aritmatik)

Setiap bilangan n Z bisa direpresentasikan secara tunggal sebagai hasil kali


faktor-faktor prima (kecuali mungkin ada dua representasi yang berbeda urutan
faktor-faktor primanya).
Bukti.

(Cara pertama). Andaikan terdapat sebuah bilangan n Z dengan dua cara faktorisasi
menurut (1.3). Apabila dibuat kesamaan kedua faktorisasi ini, kemudian kedua ruas
kesamaan dibagi oleh faktor-faktor prima yang sama, maka tersisa kesamaan
p1p2 pr = q1q2 qs

2)

3)

Dalam aljabar, Z (bersama operator tambah dan kali aritmatik) memenuhi kriteria sebagai suatu unique
factorization domain sedangkan E dan C yang diberikan dalam contoh sebelumnya tidak memenuhi.
Sebenarnya proposisi yang akan dibuktikan berbentuk implikasi: A B C. Tetapi yang dibuktikan
adalah pernyataan berbentuk A B C. Berdasarkan sistem logika, kedua pernyataan ini ekuivalen.

17

di mana semua faktor-faktor prima di ruas kiri berbeda dengan faktor-faktor prima di ruas
kanan. Tetapi hal ini mustahil sebab p1|p1p2 pr, yaitu p1| q1q2 qs sehingga menurut
Teorema 1.15, terdapat suatu indeks i sehingga p1|qi. Tetapi ini hanya terjadi jika dan
hanya jika p1 = qi, suatu kontradiksi.
(Cara kedua). Andaikan teorema tidak benar dan misalkan n adalah bilangan bulat positif
terkecil yang memiliki lebih dari satu representasi sebagai hasil kali faktor-faktor prima,
katakan n =
p1p2 pr = q1q2 qs.

(1.7)

Karena n majemuk, jelas r > 1 dan s > 1. Bilangan-bilangan prima p1, p2, , pr tak ada
satu pun yang sama dengan bilangan-bilangan prima q1, q2, , qs sebab seandainya ada
yang sama, misalnya p1, maka setelah kedua ruas (1.7) dibagi p1, diperoleh bilangan bulat
n/p1 yang lebih kecil dari n dengan dua representasi, ini kontradiksi dengan asumsi dari

hipotesis induksi bahwa setiap bilangan bulat yang lebih kecil dari n memiliki
representasi tunggal.
Tanpa mengurangi berlaku umumnya bukti, anggap p1 < q1, kemudian definisikan
bilangan bulat
N = (q1 p1)q2q3qs = p1(p2p3ps q2q3qs).

(1.8)

Jelas N < n dan hipotesis induksi, seharusnya N memiliki representasi tunggal. Tetapi
tampak dari (1.8) bahwa pasti ada dua representasi N yang berbeda: representasi dari
p1(p2p3ps q2q3qs) yang memuat faktor p1 dan representasi dari (q1 p1)q2q3qs

yang tak memuat faktor p1.


Pembuktian dengan cara kedua di atas tidak tergantung pada Teorema 1.15 atau
teorema-teorema terdahulu yang lain. Jadi sebenarnya bisa ditaruh di bagian awal dan
Teorema 1.8, 1.10 dan 1.15 bisa diturunkan sebagai akibatnya. Masalahnya, tidak semua
teorema yang terdahulu bisa diturunkan dari Teorema Dasar Aritmatik.
Telah disinggung di muka bahwa bentuk kanonik representasi dari n Z adalah
n = p11 p2 2 pr r

dengan setiap bilangan prima pi berbeda satu sama lain. Dalam praktek, kita sering
menambahkan faktor prima berpangkat 0 dalam representasi ini. Sebagai contoh, untuk

18

merumuskan pembagi persekutuan terbesar dari a, b Z, kita menulis bentuk kanonik


representasi keduanya dengan faktor-faktor perpangkatan bilangan prima pi yang sama
dan banyak bilangan prima dibuat sama (hanya berbeda pangkatnya, kalau perlu dengan
menambah pangkat 0) seperti berikut
a = p11 p2 2 pr r ,

b = p11 p22 prr .

i 0, i 0

Dengan penyajian demikian, pembagi persekutuan terbesar a dan b dirumuskan sebagai


(a, b) = p1min{1 , 1 } p2min{ 2 , 2 } prmin{ r , r } .
Sebagai contoh, karena 3600 = 243252 = 24325270 dan 756 = 22337 = 22335071,
(3600, 756) = 2min{4,2}3min{2,3}5min{0,2}7min{0,1} = 22325070= 49 = 36.
Sebuah bilangan bulat n yang memiliki bentuk kanonik (1.2) dengan i 1 untuk setiap i,
disebut bilangan bebas kuadrat (square free). Contoh bilangan bebas kuadrat: 30, 34, 209.
Teorema 1.17 (Teorema Euklid)

Banyak bilangan-bilangan prima adalah tak hingga.


Bukti.
Andaikan hanya ada berhingga bilangan-bilangan prima p1, p2, , pr. Pandang bilangan

n = 1 + p1p2pr.
Perhatikan, untuk setiap i {1, 2, , n} berlaku pin. Jadi, sembarang bilangan prima p
yang membagi n adalah bilangan prima yang berbeda dari p1, p2, , pr. Karena n adalah
bilangan prima p itu sendiri atau memiliki faktor prima p, terbukti masih ada bilangan
prima yang berbeda dari p1, p2, , pr, kontradiksi dengan pernyataan awal.
Apabila p dan q adalah dua bilangan prima yang berurutan dengan p < q, maka
selisih q p (yang merupakan bilangan bulat) disebut sebuah jarak (gap) antara kedua
bilangan prima.
Teorema 1.18

Jarak antara dua bilangan prima yang berurutan bisa dipilih sembarang besarnya.
Dengan kata lain, untuk setiap bilangan bulat positif k, bisa dicari k buah bilanganbilangan bulat majemuk yang berurutan.
Bukti.

19

Pandang k buah bilangan-bilangan bulat yang berurutan


(k + 1)! + 2,

(k + 1)! + 3,

(k + 1)! + (k + 1).

Bilangan-bilangan bulat ini semuanya bilangan majemuk, sebab masing-masing habis


dibagi oleh 2, 3, , k + 1.
Besar jarak antara dua bilangan prima yang berurutan ternyata tak beraturan, sesuai
dengan yang tersirat dari isi teorema di atas. Jika (x) menyatakan banyaknya bilangan
prima yang tak melebihi x, kita bisa menyelidiki prilaku fungsi bernilai bulat ini. Karena
ketak-teraturan kemunculan bilangan prima, tak ada harapan (x) bisa dinyatakan secara
sederhana. Tetapi sebuah hasil yang mengejutkan dalam teori bilangan tingkat lanjut,
teorema bilangan prima, adalah adanya pendekatan asimptotik terhadap (x). Teorema ini
menyatakan bahwa
lim ( x)
x

ln x
= 1.
x

Soal-Soal

1. Buktikan, sebuah bilangan bulat dengan 3 angka atau lebih habis dibagi 2 jhj angka
satuannya habis dibagi 2, habis dibagi 4 jhj jumlah angka satuan dan angka puluhan
habis dibagi 4, habis dibagi 8 jhj jumlah angka ratusan ditambah angka puluhan dan
ditambah angka satuan habis dibagi 8.
2. Buktikan, sebuah bilangan bulat habis dibagi 3 jhj jumlah angka-angkanya habis
dibagi 3, dan habis dibagi 9 jhj jumlah angka-angkanya habis dibagi 9.
3. Buktikan, sebuah bilangan bulat habis dibagi 11 jhj selisih jumlah angka-angka pada
posisi ganjil dengan jumlah angka-angka pada posisi genap habis dibagi 11.
4. Buktikan, setiap bilangan bulat positif bisa dinyatakan dalam bentuk
2 j0 + 2 j1 + 2 j2 + + 2 jm
dengan m > 0 dan 0 j0 < j1 < j2 < jm.
5. Buktikan, setiap bilangan bulat positif a bisa dinyatakan dalam bentuk

a = 3m+bm13m1 + bm23m2 + + b0 3 jm
dengan m 0 dan bi {0, 1, 1}.

20

6. Misalkan a/b dan c/d adalah dua bilangan pecah yang tak bisa disederhanakan lagi,
yaitu memenuhi (a, b) = 1 dan (c, d) = 1. Buktikan, jika jumlah keduanya bulat, maka
|b| = |d|.
7. Buktikan, apabila x = r/s adalah bilangan rasional yang tak bisa disederhanakan lagi
dan memenuhi xm = a, untuk suatu bilangan bulat a, maka s = 1.
8. Gunakan hasil butir soal terakhir di atas untuk membuktikan bahwa x2 = 2, x3 = 3 dan

x4 = 7 tak memiliki solusi rasional. Dengan kata lain, bilangan-bilangan


4

2,

3 dan

7 adalah bilangan-bilangan tak rasional. Secara umum, buktikan bahwa xm = a tak

memiliki solusi rasional kecuali a berbentuk pangkat ke m dari suatu bilangan bulat.
9. Buktikan, setiap bilangan prima berbentuk 3k + 1 juga berbentuk 6k + 1.
10. Buktikan, setiap bilangan bulat positif berbentuk 3k + 2 memiliki faktor prima dalam
bentuk yang sama. Demikian pula unntuk bentuk-bentuk 4k + 3 dan 6k + 5.
11. Buktikan, jika x dan y adalah dua bilangan bulat ganjil, maka x2 + y2 bukan kuadrat
sempurna.
12. Buktikan, jika x dan y adalah dua bilangan bulat yang relatif prima terhadap 3, maka

x2 + y2 bukan kuadrat sempurna.


13. Misalkan a dan b adalah dua bilangan bulat positif dengan (a, b) = 1 sedangkan ab
adalah bentuk kuadrat sempurna. Buktikan, a dan b berbentuk kuadrat sempurna.
Buktikan, hasil ini bisa diperluas untuk sembarang pangkat ke-k.
14. Buktikan (a, b) = (a, b, a + b) atau dalam bentuk yang lebih umum,
(a, b) = (a, b, ax + by)
untuk setiap bilangan x, y Z.
15. Buktikan, untuk setiap a, b Z yang keduanya tak bersama-sama nol, (a, a + k)|k.
16. Tentukan apakah pernyataan-pernyataan berikut benar atau salah. Apabila benar,
buktikan dan apabila salah, berikan sebuah contoh penyangkal (counter example).

a. Jika (a, b) = (a, c), maka [a, b] = [a, c].


b. Jika (a, b) = (a, c), maka (a2, b2) = (a2, c2).
c. Jika (a, b) = (a, c), maka (a, b) = (a, b, c).
d. Jika p prima, p|a dan p|(a2 + b2), maka p|b.
e. Jika p prima dan p|a7, maka p|a.
f. Jika a3|c3 maka a|c.
21

g. Jika a3|c2 maka a|c.


h. Jika a2|c3 maka a|c.
i. Jika p prima, p|a dan p|(a2 + b2), maka p|(b2 + c2) dan p|(a2 c2).
j. Jika p prima, p|a dan p|(a2 + b2), maka p|(b2 + c2) dan p|(a2 + c2).
k. Jika (a, b) = 1 maka (a2, ab, b2) = 1.
l. [a2, ab, b2)] = [a2, b2].
17. Tentukan semua n Z+ sedemikian rupa sehingga
n

j =1

j =1

j = j.
18. Diberikan dua bilangan positif a dan b yang memenuhi a|b2, b2|a3, a3|b4, b4|a5, ... ,
buktikan a = b.

22

2. Kongruensi
2.1 Kongruensi

Suatu kongruensi tidak lebih dari sebuah pernyataan tentang keterbagian.


Kongruensi sering memudahkan pencarian bukti-bukti dan kita akan melihat kongruensi
bisa memunculkan jenis masalah-masalah baru yang akan membawa kita ke topik-topik
baru yang menarik.
Definisi 2.1

Apabila sebuah bilangan bulat positif m membagi selisih a b, kita menyatakan


bahwa a kongruen b modulo m, dan ditulis a b (mod m). Jika a b tak habis dibagi
m, kita menyatakan a tak kongruen dengan b modulus m, dalam hal ini kita menulis
a b (mod m).
Dalam definisi di atas disyaratkan m > 0, walaupun definisi tetap berlaku untuk
sembarang bilangan m 0. Tetapi karena a b habis dibagi m jika dan hanya jika habis
dibagi m, kita cukup memilih m > 0 untuk mendefinisikan relasi kongruensi antara dua
bilangan.
Teorema 2.1

Misalkan a, b, c, d, x dan y melambangkan bilangan-bilangan bulat, maka


(a) Ketiga pernyataan a b (mod m), b a (mod m) dan a b 0 (mod m) adalah

pernyataan-pernyataan yang ekuivalen.


(b) Jika a b (mod m) dan b c (mod m) maka a c (mod m).
(c) Jika a b (mod m) dan c d (mod m) maka ax + cy bx + dy (mod m).
(d) Jika a b (mod m) dan c d (mod m) maka ac bd (mod m).
(e) Jika a b (mod m) dan d|m, d > 0 maka a b (mod d).
Teorema 2.2

Misalkan f adalah polinom dengan koefisien-koefisien bulat.


Jika a b (mod m) maka f(a) f(b) (mod m).

Hukum Pembatalan antara dua bilangan real: ax = ay x = y yang berlaku untuk setiap a,

x, y Z dengan a 0 dalam relasi kongruensi harus digunakan secara hati-hati.


Teorema 2.3

(a) ax ay (mod m) jika dan hanya jika x y (mod (am


).
,m )
(b) ax ay (mod m) dan (a,m) = 1, maka x y (mod m).
(c) x y (mod mi), untuk setiap i = 1, 2, , r, jika dan hanya jika

x y (mod [m1, m2, , mr]).


Bukti.
(a) Jika ax ay (mod m) maka ax ay = mz untuk suatu bilangan bulat z. Jadi,
a x a y= m z
( a ,m )
( a ,m )
( a ,m )

sehingga

m a (x y).
( a ,m ) ( a ,m )
Tetapi menurut Teorema 1.7, (am
dan (aa,m) saling prima. Akibatnya,
,m )
m (x y).
( a ,m )

Ini berarti

x y (mod (am
).
,m )
Sebaliknya jika x y (mod (am
), maka (am
(x y) sehingga m|(a, m)(x y).
,m )
,m )

Jelas m|a(x y), yaitu ax ay (mod m).


(b) Ini adalah kasus khusus dari (a).
(c) Jika untuk setiap i = 1, 2, , r berlaku x y (mod mi), maka untuk setiap i = 1, 2, ,

r berlaku mi|(x y). Ini berarti, x y mengandung kelipatan persekutuan dari m1, m2,
, mr. Jadi [m1, m2, , mr]|(x y). Ini berakibat x y (mod [m1, m2, , mr]).
Sebaliknya jika x y (mod [m1, m2, , mr]) dan mengingat mi|[m1, m2, , mr], maka
menurut Teorema 2.1e, x y (mod mi).
Diberikan sembarang bilangan bulat a, misalkan q dan r masing-masing adalah
hasil bagi dan sisa pembagian a oleh m; a = mq + r. Jadi a r (mod m), dengan 0 r < m.
Ini berarti bahwa sembarang bilangan bulat kongruen modulo m dengan salah satu dari m
bilangan bulat 0, 1, , m 1.

24

Definisi 2.2

Jika x y (mod m), maka y disebut residu dari x modulo m. Suatu himpunan {x1, x2,
, xm} disebut sistem residu lengkap modulo m jika untuk setiap bilangan bulat y,

terdapat satu dan hanya satu bilangan xj sehingga y xj (mod m).


Jelas, ada sebanyak tak hingga sistem residu lengkap modulo m dan {1, 2, , m 1}
adalah salah satu di antaranya.
Teorema 2.4

Jika x y (mod m) maka (x, m) = (y, m).


Bukti.
Diketahui y x = mz untuk suatu bilangan bulat z. Karena (x, m)|x dan (x, m)|m,
maka (x, m) membagi y = mz + x. Dari (x, m)|m dan (x, m)|y, disimpulkan (x, m)|(y, m).
Dengan mempertukarkan peran x dan y, diperoleh (y, m)|(x, m). Ini berarti (x, m) = (y, m),
karena per definisi, (x, m) dan (y, m) positif.
Sebagai contoh, karena 63 117 (mod 27), maka (63, 27) = 9 = (117, 27).
Apabila m prima, maka teorema di atas hanya menyisakan dua kesimpulan:

i. (x, m) = m = (y, m), dalam kasus x y 0 (mod m)


ii. (x, m) = 1 = (y, m), dalam kasus x y (mod m)
Definisi 2.3

Suatu himpunan {r1, r2, , rl} disebut sistem residu tereduksi modulo m jika
a. untuk setiap bilangan i = 1, 2, , l, berlaku (ri , m) = 1; dan
b. untuk setiap pasang indeks i dan j dengan i j berlaku ri rj (mod m); dan
c. setiap bilangan bulat x dengan (x, m) = 1 kongruen modulo m dengan salah satu
bilangan ri, i = 1, 2, , l.
Dengan memperhatikan Teorema 2.4, jelas bahwa suatu sistem residu tereduksi modulo

m bisa diperoleh dengan membuang semua unsur suatu sistem residu lengkap modulo m
yang tak saling prima dengan m. Lebih jauh, semua sistem residu tereduksi modulo m
terdiri atas sebanyak (m) bilangan-bilangan bulat. (m) disebut fungsi- Euler.

25

Teorema 2.5

Bilangan (m) menyatakan banyaknya bilangan bulat positif x yang memenuhi:


a. 1 x < m, dan
b. (x, m) = 1.
Teorema 2.6

Misalkan (a, m) = 1. Jika {r1, r2, , rn} adalah suatu sistem residu lengkap
[tereduksi] modulo m maka {ar1, ar2, , arn} adalah sistem residu lengkap
[tereduksi] modulo m.

Bukti.
Jika (ri, m) = 1 maka menurut Teorema 1.8, (ari, m) = 1. Karena jumlah unsur dalam
{r1, r2, , rn} dan {ar1, ar2, , arn} sama banyak, maka cukup dibuktikan bahwa ari

arj jika i j. Tetapi Teorema 2.3b menunjukkan bahwa ari arj (mod m) berakibat ri rj
(mod m).
Teorema 2.7 (Teorema Fermat)

Misalkan p adalah bilangan prima. Jika p a, maka ap1 1 (mod m).


Teorema ini merupakan akibat dari Teorema berikut.
Teorema 2.8 (Generalisasi Teorema Fermat oleh Euler)

Jika (a, m) = 1, maka a(m) 1 (mod m).


Bukti.
Misalkan r1, r2, , r(m) adalah sistem residu tereduksi modulo m. Berdasarkan Teorema
2.6, ar1, ar2, , ar(m) juga merupakan sistem residu tereduksi modulo m. Jadi untuk
setiap ri terdapat satu dan hanya satu arj sedemikian rupa sehingga ri arj mod m. Lebih
jauh, nilai ri yang berbeda akan kongruen modulo m dengan nilai arj yang berbeda. Ini
berarti bahwa ar1, ar2, , ar(m) adalah sistem residu tereduksi modulo m yang sama
dengan r1, r2, , r(m), tetapi mungkin dengan urutan yang berbeda.
Dengan mengalikan unsur-unsur kedua kelompok ini dan dengan menggunakan
(m)

(m)

j =1

i =1

Teorema 2.1d, diperoleh (arj ) (ri ) (mod m) sehingga

26

(m)

(m)

j =1

j =1

a ( m ) rj (rj ) (mod m).


Karena (rj, m) = 1, kita bisa menggunakan Teorema 2.3b untuk membatalkan suku-suku
rj dan mendapatkan a(m) 1 (mod p).
Akibat: Bukti Teorem 2.7

Jika pa maka (a, p) = 1 dan a(p) 1 (mod p). Untuk menentukan (p) digunakan
Teorema 2.5. Semua bilangan 1, 2, , p 1, p kecuali bilangan p, relatif prima terhadap
p. Jadi (p) = p 1 dan Teorema Fermat terbukti.
Akibat Teorema 2.9

Jika (a, m) = 1 maka ax b (mod m) memiliki sebuah solusi x1. Lebih jauh, semua
solusi berbentuk x = x1 + jm dengan j Z.
Bukti.
Karena (1, m) = 1 dan 1 m, kita peroleh (m) 1. Jelas x1 = a(m)1b adalah sebuah
solusi. Jika x adalah sembarang solusi, maka ax ax1 b b 0 (mod m) yang berakibat
a(x x1) 0 (mod m). Berdasarkan Teorema 2.3b berlaku x x1 0 (mod m) sehingga x
= x1 + jm. Fakta bahwa setiap bentuk x yang demikian merupakan solusi dari kongruensi
ax b (mod m), adalah akibat dari Teorema 2.2.
Fungsi Euler (m) memerlukan perhatian khusus dan akan dibahas di bagian 2.4
dan 4.2.
Teorema 2.10 (Teorema Wilson)

Jika p adalah bilangan prima maka (p 1)! 1 (mod p)


Bukti.
Jika p = 2 atau p = 3, pembuktian langsung dan gampang. Jadi dianggap p 5.
Ide pembuktian sangat sederhana dan memerlukan kehati-hatian. Kita perhatikan
bilangan-bilangan yang hasil kalinya adalah (p 1)! Kita pasang-pasangkan bilanganbilangan ini sedemikian rupa sehingga hasil kali kedua bilangan dari setiap pasang akan
kongruen dengan 1 modulo p.

27

Diberikan sembarang bilangan j memenuhi 1 j p 1, maka (j, p) = 1 dari Akibat


Teorema 2.9 disimpulkan adanya satu bilangan bulat i memenuhi ji 1 (mod p) dan 0 i
p 1. Jadi setiap bilangan j bersekawan dengan suatu bilangan i sedemikian rupa
sehingga ji ij 1 (mod p).
Pada khususnya, bilangan 1 bersekawan dengan dirinya sendiri, demikian pula
bilangan p 1. Abaikan untuk sementara kedua bilangan ini, hanya bilangan j yang
memenuhi 2 j p 2 yang diperhatikan. Untuk nilai-nilai j yang demikian, berlaku
(j 1, p) = (j + 1, p) = 1 sehingga berdasarkan Teorema 1.8 berlaku j2 1 0 (mod p).
Jadi setiap nilai j dalam kelompok ini bersekawan dengan suatu i j, 2 i p 2, dan
sebaliknya kawan dari i adalah j.
Jadi semua bilangan 2, 3, , p 2 dapat bersekawan, i bersekawan dengan j yang
memenuhi ji 1 (mod p). Apabila pasangan-pasangan ini dikalikan, diperoleh
23(p 2) 1 (mod p).
Akhirnya dari 1(p 1) 1 (mod p), Teorema Wilson bisa langsung diturunkan.
Teorema Wilson dan Teorema Fermat bisa digunakan untuk menentukan bilangan prima
p agar kongruensi x2 1 (mod p) mempunyai solusi. Ini adalah kasus khusus dari
kasus yang lebih umum yang akan kita bahas belakangan. Hal yang menarik dari kasus
khusus ini karena kasus ini bisa diselesaikan dengan cara sederhana.
Teorema 2.11

Jika p prima, maka x2 1 (mod p) memiliki solusi jika dan hanya jika p = 2 atau
p 1 (mod 4).
Bukti.
Dalam kasus p = 2 solusinya adalah x = 1. Dalam kasus p ganjil, Teorema Wilson
dirumuskan dalam bentuk
p1

p+1

(12j 2 )( 2 (p j)(p 2)(p 1)) 1 (mod p).

28

Bentuk perkalian di ruas kiri terbagi dua atas dua kelompok faktor-faktor yang sama
banyaknya. Dengan memasangkan satu faktor di kelompok kiri dengan satu faktor di
kelompok kanan sesuai urutannya secara terbalik, kongruensi di atas bisa ditulis ulang
sebagai

p 1
2

j ( p j ) 1 (mod p).
j =1

Tetapi j(p j) j (mod p) sehingga apabila p 1 (mod 4) maka (p 1)/2 adalah


2

bilangan genap dan ruas kiri kesamaan di atas menjadi


p 1
2

p 1
2

j =1

j =1

j ( p j ) ( j

) (1)

p1
2

( p 1) / 2

j =1

( p 1) / 2
j

j =1

j (mod p).

( p 1) / 2
Ini berarti bahwa x0 = j merupakan sebuah solusi dari x2 = 1 (mod p).
j =1

Jika p 2 dan p 1 (mod 4), maka p 3 (mod 4). Seandainya ada x dengan x2 1
(mod p) maka karena (p 1)/2 1 (mod 2), diperoleh
xp1 (x2)(p1)/2 (1)(p1)/2 1 (mod p).
Tetapi dari kongruensi x2 = 1 (mod p), jelas px sehingga menurut Teorema 2.7
(Teorema Fermat), xp1 1 (mod p). Ini kontradiksi dengan hasil sebelumnya sehingga
disimpulkan jika p 3 (mod 4), maka tak a\da solusi dari x2 1 (mod p).
Soal-soal.

1.

Tulis semua bilangan bulat x dengan 1 x 100 yang memenuhi x 7 (mod 17)

2.

Tentukan sebuah sistem residu lengkap modulo 17 yang terdiri atas bilanganbilangan kelipatan 3.

3.

Buktikan bahwa kuadrat suatu bilangan angka satuannya selalu angka 0, 1, 4, 5, 6


atau angka 9.

4.

Buktikan bahwa pangkat 4 dari suatu bilangan angka satuannya selalu angka 0, 1, 5
atau 6.

5.

Tentukan nilai (m) untuk setiap m = 1, 2, 3, , 12.

6.

Tentukan bilangan bulat positif terkecil sehingga 13| x2 + 1.

7.

Buktikan bahwa 19 tidak membagi 4n2 + 4, untuk setiap bilangan bulat positif n.

29

8.

Tentukan sebuah sistem residu tereduksi modulo 7 yang terdiri atas bilanganbilangan kelipatan 3.

9.

Selesaikan 3x 5 (mod 11) dengan menggunakan Akibat Teorema 2.9.

10. Beri ilustrasi bukti Teorema 2.10 (Wilson) untuk p = 10 dan p = 11 dengan sungguhsungguh menentukan pasangan bilangan-bilangan bulat yang saling berasosiasi.
11. Bilangan-bilangan bulat 12, 23, 34, 45, 56 adalah kongruen ke 1 modulo 11 . Untuk
menyelesaikan 5x 1 (mod 11) kita hanya memperhatikan bahwa 45 = 59 sehingga
x = 9 adalah sebuah solusi. Selesaikan ax 1 (mod 11) untuk a = 2, 3, , 10.
12. Buktikan n6 1 habis dibagi 7 jika (n, 7) = 1.
13. Buktikan n7 n habis dibagi 42, untuk sembarang bilangan bulat n.
14. Buktikan n12 1 habis dibagi 7 jika (n, 7) = 1.
15. Buktikan n6k 1 habis dibagi 7 jika (n, 7) = 1, di mana k adalah sebarang bilangan
bulat positif.
16. Buktikan n13 n habis dibagi 2, 3, 5, 7 , dan 13 untuk sebarang bilangan bulat n.
7 n Z.
17. Buktikan, untuk setiap n Z berlaku 15 n5 + 13 n3 + 15

18. Buktikan jika m ganjil maka (m 1)/2, (m 3)/2, ..., (m 3)/2, (m 1)/2
membentuk sistem residu lengkap modulo m dan apabila m genap maka (m 2)/2,

(m 4)/2, ..., (m 4)/2, (m 2)/2 membentuk sistem residu lengkap modulo m.


19. Buktikan, untuk setiap a, m, n Z+ dengan m n berlaku
m
n
1, jika a genap
(a 2 + 1, a 2 + 1) =
2, jika a ganjil
m

Petunjuk: Jika p adalah salah satu pembagi bersama, a 2 = 1 (mod p). Bawa bentuk
ini ke bentuk dengan pangkat 2nm dengan asumsi m < n.
2.2 Solusi Kongruensi

Sampai akhir bab 2, lambang f(x) menyatakan suatu polinom dengan koefisienkoefisien bilangan bulat dan kita menulis lengkap f(x) = a0xn + a1xn + + an. Apabila u
adalah sebuah bilangan bulat sedemikian rupa sehingga f(u) 0 (mod m), maka kita
mengatakan bahwa u adalah solusi dari kongruensi f(x) 0 (mod m). Apakah suatu

30

bilangan bulat merupakan solusi atau bukan solusi dari kongruensi ini, jawabnya juga
tergantung pada bilangan m, selain tergantung pada polinom f(x).
Jika u adalah solusi dari kongruensi f(x) 0 (mod m), dan jika u v (mod m),
maka menurut Teorema 2.2, v juga merupakan solusi. Karena fakta ini, pernyataan bahwa
x u (mod m) adalah solusi dari f(x) 0 (mod m) harus diartikan bahwa setiap bilangan
bulat kongruen ke u modulo m adalah solusi dari f(x) 0 (mod m). Sebagai contoh,
karena 3 dan 8 adalah solusi dari kongruensi x2 x + 4 0 (mod 10), maka kita menulis
bahwa x 3 (mod 10) dan x 8 (mod 10) adalah solusi dari x2 x + 4 0 (mod 10).
Khusus untuk contoh ini, karena 8 3 (mod 5), kita bisa menulis lebih jauh bahwa x 3
(mod 5) adalah solusi dari x2 x + 4 0 (mod 10).
Definisi 2.4

Misalkan r1, r2, , rm adalah suatu sistem residu lengkap modulo m. Banyak solusi
f(x) 0 (mod m) adalah banyaknya ri sedemikian rupa sehingga f(ri) 0 (mod m).
Dari Teorema 2.2 bisa disimpulkan bahwa banyaknya solusi tidak tergantung pada sistem
residu lengkap yang dipilih. Lebih jauh, banyaknya solusi tak akan melebihi modulus m.
Jika m kecil, solusinya bisa dicari dengan cara sederhana: mensubstitusi setiap r1, r2, ,
rm untuk x ke dalam kongruensi f(x) 0 (mod m). Sebagai contoh, dengan cara sederhana
ini diperoleh hasil bahwa x2 + 1 0 (mod 7) tak memiliki solusi, x2 + 1 0 (mod 5)
memiliki dua solusi, x2 1 0 (mod 8) memiliki empat solusi.
Definisi 2.5

Jika f(x) = a0xn + a1xn1 + + an dengan a0 0 (mod m) maka


f(x) 0 (mod m)
disebut kongruensi berderajat n.

31

Perlu dicatat, derajat kongruensi f(x) 0 (mod m) tidak sama dengan derajat polinom f(x)
karena besar derajat kongruensi tergantung pada modulus m. Misalmya, jika f(x) = 6x4 +
3x3 + 2x2 + 1, maka f(x) 0 (mod 2) adalah kongruensi berderajat 3 sedangkan f(x) 0
(mod 3) adalah kongruensi berderajat 2, padahal polinom f(x) sendiri berderajat 4.
Teorema 2.12

Jika d | m, d > 0, dan jika u adalah solusi dari f(x) 0 (mod m), maka u adalah
solusi dari f(x) 0 (mod d).
Bukti: Langsung dari Teorema 2.1e.

Soal-soal.

1. Jika f(x) 0 (mod p) memiliki sebanyak j solusi sedangkan g(x) 0 (mod p) tak ada
solusinya, buktikan bahwa f(x)g(x) 0 (mod p) memiliki sebanyak j solusi.
2. Apabila banyaknya solusi dari f(x) k (mod m) dilambangkan N(k), buktikan bahwa
m

N (k ) = m.
k =1

3. Apabila kongruensi f(x) 0 (mod m) memiliki sebanyak m solusi, buktikan bahwa


sebarang bilangan bulat merupakan solusi (Dalam kasus seperti ini, kongruensinya
biasa disebut kongruensi identitas).
4. Fakta bahwa jumlah tiga bilangan bulat berurutan merupakan kelipatan 3 mengimbas
kongruensi identitas x(x + 1)(x + 2) 0 (mod 3). Generalisir hasil ini dengan menulis
suatu kongruensi identitas modulo m.
2.3 Kongruensi Derajat 1

Setiap kongruensi derajat 1 bisa ditulis dalam bentuk


ax b (mod m),

a 0 (mod m).

(2.1)

Dari Akibat Teorema 2.9 disimpulkan bahwa jika (a, m) = 1, maka (2.1) hanya memiliki
tepat satu solusi, yaitu x x1 (mod m).

32

Misalkan g = (a, m). Jika u adalah solusi dari ax b (mod m) maka b au (mod
m) sehingga b au 0 (mod g). Sebagai akibatnya, g | b. Jadi jika gb, maka ax b
(mod m) tak memiliki solusi.
Tetapi dari arah sebaliknya, jika g | b maka menurut Teorema 2.3a, untuk setiap
bilangan bulat u berlaku au b (mod m) jika dan hanya jika (a/g)u b/g (mod m/g).
Mengingat (a/g, m/g) = 1, kongruensi (a/g)x b/g (mod m/g) hanya memiliki tepat satu
solusi x x1 (mod m/g). Dengan kata lain, solusi dari kongruensi ax b (mod m) adalah
bilangan-bilangan bulat u yang memenuhi u x1 (mod m/g), yaitu bilangan-bilangan
bulat u = x1 + t(m/g) dengan t Z.
Jika t dipilih di antara g buah bilangan-bilangan 0, 1, , g 1, maka ada
sebanyak g buah nilai-nilai u yang bersesuaian, satu sama lain tidak kongruen modulo m.
Jika t yang terpilih adalah bilangan yang berbeda dari g bilangan 0, 1, , g 1, maka
nilai u yang bersesuaian akan kongruen modulo m dengan salah satu dari g bilangan ini.
Jadi solusi dari ax b (mod m) adalah bilangan-bilangan x x1 + t(m/g) (mod m), di
mana 0 t g 1.
Teorema 2.13

Misalkan g = (a, m). Jika g b, maka kongruensi ax b (mod m) tak memiliki solusi
dan jika g | b, maka solusinya adalah x (b/g)x0 + t(m/g) (mod m), t = 0, 1, , g 1,
di mana x0 adalah sebarang solusi dari (a/g)x 1 (mod m/g).
Bukti.
Dari diskusi sebelumnya, sebab jika x0 adalah solusi dari (a/g)x 1 (mod m/g) maka x1 =
(b/g)x0 adalah solusi dari (a/g)x (b/g)(mod m/g).
Untuk m kecil, solusi dari kongruensi ax b (mod m) bisa didapat dengan cara
mencoba setiap bilangan dalam sistem residu lengkap modulo m. Tetapi jika m cukup
besar, solusinya bisa panjang. Bagian yang tersulit adalah pada kasus (a, m) = 1. Solusi

33

yang diberikan Akibat Teorema 2.9 biasanya tidak praktis. Sejumlah cara-cara khusus
telah dikembangkan untuk menyelesaikan kongruensi ini, tetapi barangkali cara umum
yang terbaik adalah dengan menggunakan generalisasi algoritma Euklid. Dengan cara ini,
diperoleh (a, m) dan sekaligus mendapatkan dua bilangan bulat u dan v sedemikian rupa
sehingga (a, m) = au + mv. Dengan demikian dalam Teorema 2.13, kita bisa memilih u
untuk x0 dan langkah selanjutnya menjadi mudah.
Contoh 2.1

Menurut Teorema 2.13, terdapat 6 solusi dari kongruensi


1668x 60 mod 486.
Untuk menentukan ke-6 solusi ini, pertama kali ditentukan solusi dari 278x 1 (mod
81), atau 35x = 1 mod 81. Demgan menerapkan perluasan algoritma Euklid
(dikerjakan dalam Contoh 1.2), diperoleh 6 = (1668, 486) = (37)1668 + (127)486
atau
1 = (278, 81) = (37)278 + (127)81
yang berarti (37)(278) 1 (mod 81). Karena 37 44 (mod 81) dan 278 35 (mod
81), ini berarti
(35)(44) 1 (mod 81).
Jadi x = 44 adalah solusi dari 35x 1 (mod 81).
Ke-6 solusi dari 1668x 60 mod 486 adalah
x = (60/6)44 + 81t (mod 486),
di mana t = 0, 1, 2, 3, 4, 5.
Cara lain untuk menyelesaikan kongruensi derajat 1 adalah dengan menguraikan
modulus m atas faktor-faktornya:
m=

ei
i

i =1

Dengan menulis mi = piei dan mengingat setiap pasang di antara m1, m2, , mr saling
prima satu sama lain dengan [m1, m2, , mr] = m, maka dari Teorema 2.3c, masalah
penyelesaian kongruensi ax b (mod m) ekuivalen dengan masalah menyelesaikan
sistem kongruensi ax b (mod m1), ax b (mod m2), , ax b (mod mr) sekaligus.

34

Secara individu, solusi masing-masing kongruensi dari sistem ini lebih mudah didapat
karena masing-masing mi lebih kecil daripada m = m1m2mr. Tinggal dicari solusi
bersama yang merupakan sebuah solusi sekaligus untuk semua kongruensi dalam sistem
kongruensi di atas. Teorema berikut memberikan cara mencari solusi bersama ini.
Teorema 2.14 (Teorema Sisa Cina; Chinese Remainder Theorem)

Misalkan m1, m2, , mr adalah bilangan-bilangan bulat positif yang sepasangsepasang saling prima satu sama lain sedangkan a1, a2, , ar adalah sebarang r
bilangan-bilangan bulat positif. Sistem kongruensi

x a1 (mod m1),
x a2 (mod m2),

(2.2)

x ar (mod mr)
memiliki solusi-solusi bersama dan setiap dua solusi akam saling kongruen modulo m
= m1m2mr satu sama lain.
Bukti:
Apabila m = m1m2mr, maka m/mj adalah bilangan bulat (m/mj, mj) = 1. Berdasarkan
Akibat Teorema 2.9, terdapat bilangan-bilangan bulat bj sedemikian rupa sehingga
(m/mj)bj 1 (mod mj). Jelas (m/mj)bj 0 (mod mi) apabila i j. Apabila didefinisikan
x0 =

j =1

maka diperoleh
r
m
x0 = b j a j
j =1 m j
m

biai (mod mi),


mi
ai (mod mi),

m b a
j

(2.3)

(sebab apabila j i maka (m/mj)bj 0 (mod mi))


(sebab (m/mi)bi 1 (mod mi)).

Jadi, x0 merupakan solusi bersama dari (2.2).


Misalkan x0 dan x1 adalah dua solusi bersama dari (2.2). Ini berakibat x0 x1 (mod mi),
untuk setiap i = 1, 2, , r sehingga menurut Teorema 2.3c, x0 x1 (mod m).

35

Contoh 2.2

Akan dicari semua bilangan-bilangan bulat yang setelah dibagi oleh bilangan 3, 4 dan
5 sisanya 1 atau 2. Dengan kata lain, kita mencari bilangan bulat x yang memenuhi
sistem kongruensi
x 1 atau 2 (mod 3),

x 1 atau 2 (mod 4),

x 1 atau 2 (mod 5).

Di sini m1 = 3, m2 = 4 dan m3 = 5 sehingga m = 60. Selanjutnya dicari b1, b2 dan b3


yang memenuhi kongruensi
(60/3)b1 1 (mod 3),

(60/4)b2 1 (mod 4),

(60/5)b3 1 (mod 5).

Kongruensi 20b1 1 (mod 3) ekuivalen dengan kongruensi b1 1 (mod 3). Dengan


m ) b = 20b = 20. Secara analog, diperoleh b = 1
memilih b1 = 1, diperolah ( m
1
1
2
1

sehingga ( mm ) b2 = 15b2 = 15. Kongruensi 12b3 1 (mod 5) ekuivalen dengan 2b3


2
1 (mod 5). Setelah kedua ruas dikali 2, kongruensi ini ekuivalen dengan kongruensi

4b3 2 (mod 5) atau b3 2 (mod 5). Dengan memilih sebuah solusi b3 = 2,


diperoleh

( mm3 )

b3 = (2)(12) = 24. Nilai-nilai yang diperoleh selanjutnya

disubstitusi ke dalam ekspresi (2.3) untuk mendapatkan


x0 = 20a1 15a2 24a3 (mod 60).
Solusi lengkap diperoleh dari substitusi semua kemungkinan nilai a1, a2, a3 {1, 2}
dan solusi ini bisa digambarkan melalui tabel berikut.
a1

a2

a3

Solusi x0 (mod 60)

20 15 24 59

20 15 48 83 23

20 30 24 74 14

20 20 48 98

40 15 24 79 19

40 15 48 103

17

40 30 24 94

26

40 30 48 118

22

36

Jadi, semua bilangan bulat yang sisanya 1 atau 2 apabila dibagi oleh bilangan 3, 4 dan
5 diwakili oleh bilangan-bilangan 1, 2, 17, 22, 26 14, 19, 23 (mod 60). Sebagai
contoh, solusi 17 mewakili barisan solusi-solusi , 103, 43, 17, 77, 137, .
Soal-soal.

1. Cari semua solusi dari kongruensi


a. 20x 4 (mod 30)

b. 20x 30 (mod 4)

b. 353x 254 (mod 400)

2. Ada berapa banyak solusi dari kongruensi berikut?


a. 15x 25 (mod 35)

b. 15x 24 (mod 35)

b. 15x 0 (mod 35)

3. Cari solusi bersama (keculi solusi x0 = 1) dari sistem kongruensi berikut


x 1 (mod 3), x 1 (mod 5), x 1 (mod 7).
4. Cari semua bilangan-bilangan bulat yang sekaligus memenuhi kongruensi-kongruensi
x 2 (mod 3), x 3 (mod 5), x 5 (mod 2).
5. Cari solusi bersama dari kongruensi-kongruensi berikut
x 1 (mod 4), x 0 (mod 3), x 5 (mod 7).
6. Cari bilangan-bilangan bulat yang menghasilkan sisa 1 apabila dibagi oleh bilangan 3,
sisa 2 apabila dibagi oleh bilangan 4 dan menghasilkan sisa 1 apabila dibagi oleh
bilangan 5.
7. Jika a dipilih secara acak dari 1, 2, 3, , 14 dan b dipilih secara acak dari 1, 2, 3, ,
15, berapa besar peluangnya ax b (mod 15) menghasilkan paling kurang satu
solusi? Tepat satu solusi?
8. Misalkan (a, b) = 1 dan c > 0. Buktikan bahwa ada bilangan bulat x sedemikian rupa
sehingga (a + bx, c) = 1.
9. Diberikan sembarang bilangan bulat positif k, buktikan bahwa ada k bilanganbilangan bulat berurutan yang masing-masing habis dibagi oleh kuadrat suatu
bilangan > 1.

37

10. Jika x2 adalah solusi dari ax b (mod m), buktikan bahwa x x2 + t(m/g) (mod m)
menghasilkan semua solusi apabila t melalui semua bilangan 0, 1, , g 1, di mana
g = (a, m).
2.4 Fungsi Euler (n)

Di sini akan disajikan penerapan Teorema Sisa Cina untuk mendapatkan sifatsifat penting dari fungsi Euler (n).
Teorema 2.15

Jika m dan n adalah dua bilangan bulat positif yang relatif prima, maka
(mn) = (m)(n).
Bukti. Misalkan j = (m), k = (n) dan r1, r2, , rj adalah sistem residu tereduksi modulo
m sedangkan s1, s2, , sk adalah sistem residu tereduksi modulo n. Jika x adalah salah
satu bilangan dalam sistem residu tereduksi modulo mn, maka (x, m) = (x, n) = 1 dan ini
berarti x = rh (mod m) untuk suatu h {1, 2, , j} dan x = si (mod n) untuk suatu i {1,
2, , k}. Sebaliknya jika x = rh (mod m) untuk suatu h {1, 2, , j} dan x = si (mod n)
untuk suatu i {1, 2, , k}, maka (x, mn) = 1. Jadi sistem residu tereduksi modulo mn
bisa ditentukan dengan mencari semua x yang memenuhi x = rh (mod m) untuk suatu h
{1, 2, , j} dan x = si (mod n) untuk suatu i {1, 2, , k}. Menurut Teorema Sisa Cina,
setiap pasang kongruensi x = rh (mod m) dan x = si (mod n) memberi satu solusi bersama
x modulo mn. Jelas dua pasangan (rh, sj) yang berbeda menghasilkan dua solusi bersama
yang berbeda. Karena semuanya ada jk pasang (rh, sj) yang berbeda maka ini berarti ada
sebanyak jk = (m)(n) solusi bersama yang berbeda, dengan kata lain, ada sebanyak jk =
(m)(n) bilangan-bilangan bulat yang berbeda dalam sistem residu tereduksi modulo mn.
Dengan kata lain, (mn) = (m)(n).
Persyaratan m dan n relatif prima tidak bisa dilepaskan. Sebagai contoh, (2) = 1
dan (22) = (4) = 2 1 = 11 = (2)(2).
Teorema 2.16

Jika n > 1 maka (n) = n (1 1p ). Juga (1) = 1.


p| n

38

Catatan. Dalam notasi di atas, lambang

p| n

menyatakan hasil kali atas indeks bilangan-bilangan prima p pembagi n. Notasi


ini kelak akan sering digunakan. Kita juga akan sering menggunakan notasi

d |n

untuk menyatakan hasil kali atas indeks semua bilangan-bilangan bulat positif d
(prima atau bukan prima) pembagi n.
Lebih jauh, diadopsi konvensi bahwa jumlah atas indeks kosong adalah 0 dan
hasil kali atas indeks kosong adalah 1. Pada khususnya,

a = 0
i

b = 1.

dan

Bukti Teorema 2.16.


Jelas bahwa (1) = 1. Jika n > 1, maka n =
e

ei
i

dan untuk setiap j = 1, 2, , r 1

i =1

berlaku ( p j j , p j +j+11 p j +j+22 ... prer ) = 1. Dengan menerapkan Teorema 2.15 beberapa kali,
diperoleh (n) =

( p

ej
j

).

j =1

Untuk menghitung (pe) dengan p prima, diingatkan bahwa (pe) adalah banyaknya
bilangan bulat x yang memenuhi kedua syarat: 1 x pe dan (x, pe) = 1. Karena ada
sebanyak pe bilangan bulat x yang memenuhi 1 x pe sedangkan ada sebanyak pe1 di
antaranya: yaitu p, 2p, , pe1p; tak memenuhi syarat (x, pe) = 1, maka ada sebanyak pe
pe1 = pe(1 1/p) bilangan-bilangan bulat tersisa yang sekaligus memenuhi kedua syarat
di atas. Ini berarti (pe) = pe(1 1/p). Jadi,
(n) =

( p j ) =
j =1

ej

e
p j j (1 p1 ) =
j =1

j =1

j =1

e
p j j (1 p1 ) = n (1 p1 ) .

j =1

Teorema 2.17

Untuk setiap n 1 berlaku n =

Bukti.

(d ) .
d |n

Pembuktian menggunakan induksi matematis pada banyaknya faktor-faktor prima dari n.


Jika n = pe dengan p prima, maka berlaku

39

(d )

= (1) + (p) + (p2) + + (pe)

d |n

= 1 + (p 1) + p(p 1) + + pe1(p 1)
= pe = n.
Jadi teorema terbukti untuk kasus n hanya memiliki satu faktor prima.
Anggap teorema benar untuk semua bilangan-bilangan bulat n yang merupakan
hasil kali sebanyak k atau kurang dari k faktor-faktor prima. Akan dibuktikan teorema
benar untuk setiap bilangan bulat N yang merupakan hasil kali sebanyak k + 1 faktorfaktor prima.
Misalkan p adalah salah satu faktor prima dari N dengan pe adalah perpangkatan
tertinggi di antara semua perpangkatan faktor-faktor prima dari N. Jadi ada bilangan bulat
n yang merupakan hasil kali sebanyak k faktor-faktor prima sedemikian rupa sehingga N
= pen dan (p, n) = 1.
Jika d adalah sebuah bilangan bulat positif pembagi n, maka d, pd, p2d, , ped
adalah bilangan-bilangan bulat pembagi N. Mengingat hasil ini dan (p, d) = 1,

(d ) = (d ) + ( pd ) + ( p d ) + + ( p d )
2

d |N

d |n

d |n

(d ){1 + ( p) + ( p

d |n

d |n

) + ... + ( p e )}

d |n

(d ) ( ) .
d |n

| pe

Soal-soal.

1. Tentukan semua nilai-nilai n yang membuat (n) ganjil.


2. Hitung banyaknya semua bilangan-bilangan bulat positif 3600 yang relatif prima
terhadap 3600.
3. Hitung banyaknya semua bilangan-bilangan bulat positif 3600 yang memiliki faktor
persekutuan bersama dengan 3600.
4. Hitung banyaknya semua bilangan-bilangan bulat positif 7200 yang relatif prima
terhadap 3600.
5. Hitung banyaknya semua bilangan-bilangan bulat positif 25200 yang relatif prima
terhadap 3600 (Amati bahwa 7 3600 = 25200).

40

6. Jika m dan k adalah dua bilangan bulat positif, buktikan bahwa banyaknya semua
bilangan positif mk yang relatif prima terhadap m adalah k(m).
7. Tunjukkan (mn) = n(m) jika setiap bilangan prima yang membagi n juga membagi
m.
8. Jika P menyatakan hasil kali bilangan-bilangan prima yang bersekutu dalam m dan n,
buktikan bahwa (mn) = P(m)(n)/(P). Jadi apabila (m, n) > 1, maka (mn) >
(m)(n).
2.5 Kongruensi Derajat Lebih Tinggi

Tidak ada metoda umum pencarian solusi yang bisa diberlakukan untuk semua
kongruensi. Walaupun demikian, beberapa masalah kongruensi bisa direduksi menjadi
masalah kongruensi dengan modulus-modulus prima sehingga Teorema Sisa Cina bisa
diterapkan.
Setiap bilangan bulat positif merupakan hasil kali perpangkatan faktor-faktor
prima. Jadi apabila m > 0 bulat, maka m = p1e1 p2e2 prer dan kongruensi f(x) 0 (mod m)
ekuivalen dengan sistem r kongruensi-kongruensi f(x) 0 (mod piei ), i = 1, 2, , r,
dimana solusi yang diperoleh dari salah satu kongruensi juga harus merupakan solusi dari
kongruensi-kongruensi yang lainnya. Ini berarti jika ada suatu j, 1 j r, kongruensi f(x)
e

0 (mod p j j ) tak memiliki solusi, maka f(x) 0 (mod m) tak akan memiliki solusi.
Dari lain pihak, jika setiap kongruensi f(x) 0 (mod piei ), i = 1, 2, , r, memiliki
solusi, maka kita bisa memisalkan kongruensi ke-i memiliki ki solusi ai(1) , ai(2) , ,

ai(ki ) (Definisi 2.4) sedemikian rupa sehingga tak ada sepasang di antaranya yang
kongruen modulo piei dan setiap solusi dari f(x) 0 (mod piei ) kongruen dengan suatu

ai( j ) modulo pi.


Sekarang, suatu bilangan bulat u adalah akar dari f(x) 0 (mod m) jika dan hanya
jika untuk setiap i terdapat suatu ji sedemikian rupa sehingga u ai( ji ) (mod piei ). Dengan
demikian, u adalah solusi dari kongruensi f(x) 0 (mod m) jika dan hanya jika u adalah
solusi bersama dari sistem kongruensi

x ai( ji ) (mod piei ),

i = 1, 2, , r.
41

Karena berbentuk kongruensi derajat 1 dan setiap pasangan modulus-modulus piei , i = 1,


2, , r, relatif prima satu sama lain, Teorema Sisa Cina bisa diterapkan. Dalam hal ini,
dicari terlebih dulu bilangan-bilangan bi sedemikian rupa sehingga
m
b 1 (mod piei )
piei i
dan kemudian menentukan solusi u dengan menggunakan rumus (2.3) Teorema Sisa Cina,
r m
u ei bi ai( ji ) (mod m) .
(2.4)
i =1 pi
Ekspresi (2.4) menyatakan semua solusi dari kongruensi f(x) 0 (mod m). Karena

ai( ji ) meliputi semua kemungkinan ji, i = 1, 2, , r, maka ada sebanyak k1 k2 kr


m
solusi. Dalam praktek, lebih baik mendapatkan terlebih dulu koefisien-koefisien ei bi
pi
dari jumlahan (2.4) sebab nilainya tak tergantung dari nilai-nilai ai( ji ) .
Teorema 2.18

Misalnya m = p1e1 p2e2 prer adalah faktorisasi bilangan bulat positif m atas faktorfaktor perpangkatan prima dan N(m) menyatakan banyak solusi yang berbeda dari
kongruensi f(x) 0 (mod m). Dalam hal ini, berlaku
N(m) =

N( p

ei
i

).

i =1

Dalam rumus di atas, kasus N ( piei ) = 0 untuk suatu indeks i (yang berakibat N(m) = 0),

tidak dikecualikan.
Contoh 2.3

Untuk menyelesaikan x2 + x + 7 0 (mod 15), kita mencoba ke-5 buah bilanganbilangan 0, 1, 2, kongruensi x2 + x + 7 0 (mod 5) tidak memiliki solusi. Karena di
sini m = 15 = 35, maka bisa disimpulkan bahwa kongruensi x2 + x + 7 0 (mod 15)
tidak memiliki solusi bersama.
Contoh 2.4

Kita selesaikan x2 + x + 7 0 (mod 189). Di sini m = 189 = 277 dan solusi dari

x2 + x + 7 0 (mod 27)

42

ada sebanyak k1 = 3, yaitu a1(1) = 4, a1(2) = 13 dan a1(3) = 5 (Bagaimana cara


mendapatkan solusi ini secara sistematis, akan dibahas dalam Contoh 2.5 pada bagian
mendatang). Dari pihak lain, solusi dari

x2 + x + 7 0 (mod 7)
ada sebanyak k2 = 2, yaitu a2(1) = 0 dan a2(2) = 1.
Nilai b1 diperoleh dari
189
b 1(mod 27)
27 1

atau

7b1 1 (mod 27)

atau

28 b1 4 (mod 27), yaitu

b1 4 (mod 27).
Nilai b2 diperoleh dari

189
b 1(mod 7)
7 2

atau

27b2 1 (mod 7)

atau

b2 1 (mod 7), yaitu

b2 1 (mod 7).
Jadi, (2.4) di sini berbentuk
u 7(4) a1( j1 ) + 27(1) a2( j2 ) 28 a1( j1 ) 27 a2( j2 ) (mod 189)
Perhatikan koefisien-koefisien

m
m
b = 27 dari jumlah di atas. Dari
e1 b1 = 28 dan
p1
p2e2 2

kesamaan terakhi, diperoleh semua k1k2 = 6 solusi, yaitu


a1(1) = 4, a2(1) = 0 memberikan solusi

u1 28(4) 27(0) 77 (mod 189);

a1(2) = 13, a2(1) = 0 memberikan solusi

u2 28(13) 27(0) 14 (mod 189);

a1(3) = 5, a2(1) = 0 memberikan solusi

u3 28(5) 27(0) 49 (mod 189);

a1(1) = 4, a2(2) = 1 memberikan solusi

u4 28(4) 27(1) 50 (mod 189);

a1(2) = 13, a2(2) = 1 memberikan solusi

u5 28(13) 27(1) 13 (mod 189);

a1(3) = 5, a2(2) = 1 memberikan solusi

u6 28(5) 27(1) 76 (mod 189).

Soal-soal.

1. Selesaikan kongruensi-kongruensi berikut


x3 + 2x 3 0 (mod 9)
x3 + 2x 3 0 (mod 5)
x3 + 2x 3 0 (mod 45)

43

2. Selesaikan kongruensi
x3 + 4x + 8 0 (mod 15)
3. Selesaikan kongruensi x3 9x2 + 23x 15 0 (mod 503) dengan mengamati bahwa
503 adalah bilangan prima dan polinom bisa difaktorkan menjadi (x 1)(x 3)(x 5).
4. Selesaikan kongruensi x3 9x2 + 23x 15 0 (mod 143).
2.6 Modulo Perpangkatan Bilangan Prima

Jika r adalah solusi dari f(x) 0 (mod ps), maka f(r) 0 (mod pt) untuk setiap t =
1, 2, , s. Sebagai akibatnya, jika s 2 dan r xs(i ) merupakan salah satu solusi dari
(j )
kongruensi f(x) 0 (mod ps) maka xs( ji1) dengan xs(i ) xs i1 (mod ps1) merupakan ssalah

satu solusi dari kongruensi f(x) 0 (mod ps1). Ini berarti

xs(i) xs( ji1) + vs1ps1(mod ps)

(2.5)

untuk suatu bilangan bulat vs1. Karena (2.5) merupakan solusi dari f(x) 0 (mod ps),
f( xs( ji1) + vs1ps1) f( xs(i ) ) 0 (mod ps).
Substitusi x = xs( ji1) dan h = vs1ps1 ke dalam deret Taylor fungsi f berderajat n
f(x + h) = f(x) + f(x)h + f(x)h2/2 + +

1 (n)
f ( x ')hn ,
n!

x < x' < x + h

dan mengingat untuk setiap t 2 berlaku ht = (vs1ps1)t 0 (mod ps), dalam modulus ps
deret Taylor di atas menjadi
f( xs( ji1) + vs1ps1) f( xs( ji1) ) + f( xs( ji1) )vs1ps1 (mod ps).
Mengingat ruas kiri kongruensi ini ekuivalen dengan f( xs(i ) ) 0 (mod ps), diperoleh
f( xs( ji1) ) f( xs( ji1) )vs1ps1 (mod ps).
Karena f( xs( ji1) ) 0 (mod ps1), disimpulkan f( xs( ji1) ) adalah kelipatan dari ps1 sehingga
f( xs( ji1) )vs1 =

1
f ( xs( ji1) ) (mod p).
p s 1

(2.6)

Sebaliknya, jika
f( xs( j1) )v =

1
f ( xs( j1) ) (mod p),
s 1
p

(2.7)

44

maka f( xs( j1) + vps1) 0 (mod ps). Hasil ini memberikan cara mencari solusi dari f(x) 0
(mod ps) apabila diketahui solusi f(x) 0 (mod ps1), s 2.
Setiap akar xs( j1) dari f(x) 0 (mod ps1) memberikan nilai-nilai v dari (2.7)
sedemikian rupa sehingga menurut (2.5), xs( j1) + vps1 adalah solusi dari f(x) 0 (mod ps).
Tentu saja bisa terjadi sbuah kasus di mana tak ada satu pun v yang bersesuaian dengan
xs( j1) . Dalam kasus ini, tak ada solusi dari f(x) 0 (mod ps) yang diperoleh dari xs( j1) .
Penyelesaian f(x) 0 (mod ps) dengan s 2, diawali oleh pencarian solusi-solusi
x1( j ) dari f(x) 0 (mod p). Dengan menetapkan satu solusi x1( ji ) , pertama kali kita
selesaikan (2.6) untuk s = 2 agar bisa mendapatkan nilai-nilai v1 {0, 1, 2, ,
(p 1)/2}. Untuk setiap v1 yang diperoleh, dengan menggunakan (2.5) didapatkan akar
x2( k ) x1( ji ) + v1p (mod p2)
dari kongruensi f(x) 0 (mod p2). Dengan menggunakan setiap solusi x2( k ) , ekspresi (2.6)
diselesaikan untuk s = 3 dan ji = k, diperoleh solusi dari f(x) 0 (mod p3). Tetapi
kongruensi (2.6) untuk mencari nilai-nilai v2 modulusnya p dan x2( k ) x1( ji ) (mod p)
sehingga kongruensi (2.6) menjadi berbentuk
f( x1( ji ) )v2 =

1
f ( x2( k ) ) (mod p).
p2

Langkah di atas berulang pada setiap nilai-nilai s berikutnya. Secara umum, vs1
ditentukan dari
f( x1( ji ) )vs1 =

1
f ( xs( k1) ) (mod p).
s 1
p

(2.8)

untuk setiap xs( k1) yang diperoleh secara bertahap, diawali oleh solusi awal x1( ji ) dari
kongruensi f(x) 0 (mod p).
Ekpresi (2.8) memberikan solusi tunggal untuk vs1 {0, 1, 2, , (p 1)/2}
jika dan hanya jika f( x1( ji ) ) 0 (mod p), yaitu terdapat tepat satu vs1 untuk setiap solusi
xs( k1) yang diawali oleh solusi x1( ji ) dari kongruensi f(x) 0 (mod p).
Sebaliknya apabila f( x1( ji ) ) 0 (mod p), maka (2.8) berbentuk

45

(0)vs1

1
f ( xs( k1) ) (mod p)
s 1
p

atau (setelah kedua ruas dikali ps1)


(0)vs1 f ( xs( k1) ) (mod ps).

(2.9).

Dalam kasus f ( xs( k1) ) 0 (mod ps), kesamaan (2.9) berbentuk


(0)vs1 = 0 (mod p)
dan semua p buah nilai-nilai vs1(mod p), yaitu 0, 1, 2, , (p 1)/2 (mod p) akan
memenuhi kesamaan ini.
Dalam kasus f ( xs( k1) ) 0 (mod ps), ruas kanan (2.9) tidak sama dengan 0, padahal
ruas kirinya bernilai 0. Dalam kasus ini, tak ada satu pun nilai vs1 yang akan memenuhi
(2.9) sehingga tak ada satu pun solusi xs( k ) yang bisa diturunkan dari xs( k1) .
Contoh 2.5

Pada kongruensi

x2 + x + 7 0 (mod 27).

hanya ada sebanyak k = 1 solusi, katakan x1(1) 1 (mod 3), dari x2 + x + 7 0 (mod 3).
Untuk s = 2, nilai v1 diperoleh dari nilai f(x) = 2x + 1 di titik x1(1) 1 (mod 3).
Ternyata
f( x1(1) ) f(1) 0 (mod 3).
Jadi ada dua kemungkinan, nilai v1 yang memenuhi (2.9) dipenuhi oleh ketiga nilai
1, 0, 1 (mod 3) atau tak ada satu pun nilai v1 yang memenuhi (2.9). Kedua
kemungkinan ini masing-masing ditentukan oleh dua kemungkinan nilai f ( xs( k1) ) =
f( x1(1) ) di ruas kanan (2.9) berikut: mungkin f( x1(1) ) 0 (mod 9) atau mungkin f( x1(1) )
0 (mod 9).

Karena f( x1(1) ) = f(1) 0 (mod 9) ketiga nilai v1 0, 1 dan 1 (mod 3)


memenuhi (2.9). Dari ekspresi (2.5), x2( k ) x1(1) + v1(3) (mod 9), diperoleh x2(1) 1 +
(0)(3) 1 (mod 9), x2(2) 1 + (1)(3) 4 (mod 9) dan x2(3) 1 + (1)(3) 2 (mod 9).

46

Secara analog, dengan substitusi x2(1) 1 (mod 9), x2(2) 4 (mod 9) dan x2(3)
2 (mod 9) pada f(x) = 2x + 1 diperoleh f(1) f(4) f(2) 0 (mod 3). Dari

persamaan
1
f ( x2(i ) ) (mod 3),
9
yang diperoleh dari substitusi s = 3, p = 3 pada (2.9), keberadaan nilai v2 tak ada atau
0v2

terdiri atas tiga nilai (1, 0 dan 1) tergantung pada dua kemungkinan: f( x2( i ) ) 0 atau
f( x2( i ) ) 0 (mod 27).

Karena f( x2(1) ) f( x2(3) ) 9 0 (mod 27), maka tak ada v2 yang memenuhi
(2.9) untuk x2(1) dan untuk x2(3) . Sebaliknya karena f( x2(2) ) 27 0 (mod 27), maka
ketiga nilai v2 = 0, 1, 1 memenuhi (2.9). Ketiga nilai v1 ini memberikan solusi x3( k )
x2(2) + v2(9) sehingga diperoleh x3(1) 4 + (0)(9) 4 (mod 27), x3(2) 4 + (1)(9) 13

(mod 27) dan x3(3) 4 + (1)(9) 5 (mod 27).


Contoh 2.6

Kita akan menyelidiki kongruensi x2 + x + 7 0 (mod 34). Dari contoh sebelumnya,


x1(1) = 1 merupakan solusi tunggal dari x2 + x + 7 0 (mod 3) dan karena f( x1(1) )
f(1) 0 (mod 3), ruas kiri kongruensi (2.8) bernilai 0. Dengan kata lain, kongruensi

(2.9) berlaku.
Juga dari Contoh 2.5, diperoleh solusi x3(1) 4 (mod 27), x3(2) 13 (mod 27) dan x3(3)
5 (mod 27). Untuk setiap solusi, masing-masing ruas kanan (2.9) adalah f( x3(1) )
f(4) 27 0 (mod 34), f( x3(2) ) f(13) 189 0 (mod 34) dan f( x3(3) ) f(2) 27

0 (mod 34), sehingga tak satu pun nilai v3 yang memenuhi (2.9). Jadi x4( k ) x3( j ) +
v3(33) (mod 34) tak bisa diselesaikan dan kongruensi x2 + x + 7 0 (mod 34) tak

memiliki solusi.
Contoh 2.7

Kita akan mencari solusi-solusi untuk kongruensi


x2 + x + 7 0 (mod 73).

47

Solusi dari x2 + x + 7 0 (mod 7) adalah x1(1) 0 (mod 7) dan x1(2) 1 (mod 7).
Di sini, f( x1(1) ) f(0) 1 0 (mod 7) sehingga (2.8) berlaku dan hanya ada
tepat satu v1 yang bersesuaian dengan x1(1) . Bentuk (2.8) di sini adalah
vs1

1
f ( xs( k1) ) (mod 7).
s 1
7

Untuk x1(1) 0, s = 2, f( x1(1) ) = f(0) = 7 dan v1 = 7/7 1 (mod 7) sehingga x2(1) =


x1(1) + v1.7 7 (mod 49). Akibatnya untuk s = 3, f( x2(1) ) = 49 sehingga v2 =
f( x2(1) )/49 = 1 dan
x3(1) = x2(1) + v2(49) = 7 49 56 (mod 73).

Ini adalah solusi x2 + x + 7 0 (mod 73) yang bersesuaian dengan x1(1) 0 (mod 7).
Selanjutnya untuk mencari solusi yang bersesuaian dengan x1(2) 1 (mod 7),
kita perhatikan f( x1(2) ) f(1) 1 0 (mod 7) sehingga (2.8) berlaku dan hanya
ada tepat satu bilangan v1 yang bersesuaian dengan x1(2) . Bentuk (2.8) di sini adalah
vs1
vs1

1
f ( xs( k1) ) (mod 7), atau
7 s 1
1
f ( xs( k1) ) (mod 7).
s 1
7

Untuk s = 2 dan k = 2, diperoleh f( x1(2) )= f(1) = (2)(1) + 1 7 (mod 7) dan v1 = 7/7


= 1 sehingga x2(2) = x1(2) + v1.7 6 (mod 49). Dari f( x2(2) ) = 49, diperoleh v2 = 49/49
= 1 sehingga diperoleh solusi kedua yang bersesuaian dengan solusi x1(2) ,
x3(2) = x2(2) + v2(49) = 6 + 49 55 (mod 73)

untuk kongruensi x2 + x + 7 0 (mod 73).


Dalam menyelesaikan suatu masalah berhitung, sering diperlukan jawaban apakah
suatu bilangan bulat k membagi bilangan bulat n k. Jika (k, 10) = 1 dan k tidak terlalu
besar, ada cara sederhana (tanpa melakukan operasi pembagian) untuk menjawabnya.

48

Contoh 2.8

Ambil k = 31, n = 23754. Dengan mengeksploitasi fakta bahwa angka satuan dari n
adalah 4 dan angka satuan dari k adalah 1, diperoleh
n 4k = 23754 431 = 23750 + 4 430 41 = 23750 430 = 10(2375 12).

Karena (31, 10) = 1, disimpulkan bahwa k = 31 membagi n jika dan hanya jika 31
membagi 2375 12 = 2363. Selanjutnya secara analog, prosedur di atas diulang
dengan n = 2363 (yang angka satuannya 3) untuk memperoleh
n 3k = 2363 331 = 2360 + 3 330 31 = 2360 330 = 10(236 9) = 10227

sehingga disimpulkan 31|2363 jika dan hanya jika 31|227. Jadi, 31|23754 jika dan
hanya jika 31|227.
Akhirnya, uji keterbagian oleh bilangan k = 31 terhadap n = 227 (yang angka
satuannya 7) dikerjakan dengan cara yang sama untuk memperoleh
n 7k = 227 731 = 220 + 7 730 71 = 220 730 = 10(22 21) = 101.

Dari sini disimpulkan bahwa 31|23754 jika dan hanya jika 31|1. Karena 31 1, maka
disimpulkan 31 23754.
Contoh 2.9

Cara serupa tetapi dalam kasus angka satuan dari bilangan k adalah 10 1 = 9, bisa
dikerjakan, sesuai ilustrasi berikut. Kita akan menentukan apakah bilangan k = 19
membagi n = 20520. Perhatikan,
20520 + 319 = 10(2052) + 0 + 019 01 = 102052.
Karena (10, 31) = 1, maka disimpulkan 31| 20520 jika dan hanya jika 31| 2052. Dari
2052 = 10(205) + 2 + 220 21 = 10(205 + 4) = 10209
sehingga disimpulkan 19|20520 jika dan hanya jika 19|209. Selanjutnya dari
209+ 919 = 10(20) + 9 + 920 91 = 10(20 + 18) = 1038,
Diperoleh kesimpulan 19|20520 jika dan hanya jika 19 | 38. Jadi, 19 | 20520.
Kedua contoh di atas bisa dinyatakan melalui dua skema berikut.
Skema 1: 31|23754?

Skema 2: 19|20520?

Jawab:

Jawab:

31 = 103 + 1,

19 = 102 1,
49

23754 = 102375 + 4

20520 = 102052 + 0

Jadi,

Jadi,
23754
12
2363
9
227
21
1

Kesimpulan:

20520
0+
2052
4+
209
18 +
38

(Catatan: 12 = 34)
(Catatan: 9 = 33)
(Catatan: 21 = 37)
31 23754.

Kesimpulan:

(Catatan: 0 = 20)
(Catatan: 4 = 22)
(Catatan: 18 = 29)
19 | 20520.

Jika angka terakhir (angka satuan) dari k adalah 3, maka terdapat bilangan bulat positif j
sehingga 3k = 10j 1 dan jika b adalah angka terakhir (angka satuan) dari n = 10a + b,
n + 3bk = 10(a + bj)

diperoleh. Ditarik kesimpulan bahwa k | n jika dan hanya jika k | (a + bj). Demikian pula,
jika angka terakhir k adalah 7 = 10 3 sedangkan angka terakhir dari n = 10a + b adalah
b, terdapat suatu bilangan bulat positif j yang memenuhi persamaan 3k = 10j + 1 sehingga
n 3bk = 10a + b (10bj + b) = 10(a bj).

Jadi k | n jika dan hanya jika k | a bj


Contoh 2.10

Contoh 2.11

Skema 3: 13|23765?

Skema 4: 17|20519?

Jawab:

Jawab:

313 = 104 1,

317 = 105 + 1,

23765 = 102376 + 5

20519 = 102051 + 9

Jadi,

Jadi,
23764
20 +
2396
24 +
263
12 +
38

Kesimpulan:

(Catatan: 20 = 45)
(Catatan: 24 = 46)
(Catatan: 28 = 43)
13 23765.

20519
45
2006
30
170
0+
17
Kesimpulan:

(Catatan: 45 = 59)
(Catatan: 30 = 56)
(Catatan: 0 = 50)
17 | 20519.

50

Walaupun menguji keterbagian dalam ke empat contoh di atas lebih mudah


dilakukan secara manual, tetapi secara algoritmis prosedur tanpa operasi pembagian bisa
jauh lebih efisien. Sebab di dalam sistem komputer, eksekusi satu operasi pembagian
memiliki kompleksitas yang lebih tinggi (lebih rumit) daripada eksekusi beberapa buah
operasi tambah/kurang dan operasi pemotongan (truncating) angka sekaligus. Pada
khususnya, operasi bagi dengan bilangan pembagi terlalu besar atau terlalu kecil sering
menjadi sumber dari ketidak-akuratan hasil komputasi.
Uraian cara di atas digunakan untuk menentukan keterbagian k terhadap n. Tetapi
hasil akhir dari uraian di atas lebih sering berbentuk bilangan yang tidak kongruen
dengan n modulo k (artinya, k tidak membagi n). Dengan cara di atas, tidak diketahui sisa
tak negatif hasil bagi n oleh k. Tiga butir pertama dalam soal-soal berikut memberikan
skema cara mencari sisa hasil bagi n oleh bilangan-bilangan k = 7, 13, 9, 11, 17 dan 19,
jadi langsung menghasilkan bilangan yang kongruen n modulo k.
Soal-soal.

1. Perhatikan skema di bawah yang dibuat dengan menggunakan bilangan pengali 3


untuk mencari bilangan bulat positif p sehingga n p (mod k), k = 7 atau 13.
Skema:

Catatan:

1234

n = 1234

369

369 = 3123

108

108 = 336

30

30 = 310

9 = 33

Skema di atas memberikan kesimpulan:


1234 4 + 9 + 8 + 0 + 9 30 2 (mod 7)
1234 4 9 + 8 0 + 9 12 2 (mod 13).
Jelaskan mengapa cara dalam skema ini berlaku untuk setiap bilangan bulat positif n?
Berapakah pengali (tidak lagi 3) yang harus digunakan untuk mencari bilangan p
yang memenuhi n p (mod k) dengan k = 9 atau 11? k = 17? k = 19?
Untuk k = 17 dan k = 19, cara di atas terlalu panjang dan tidak praktis. Cari variasi
cara di atas. Sebagai contoh, gunakan skema berikut ini

51

1734562
86725
4335
215
10
dengan kesimpulan: 1734562 62 + 25 + 35 + 15 + 10 147 52 (mod 19),
2. Tunjukkan bahwa untuk k = 9, cara yang diberikan dalam teks dan dalam butir soal 1
pada dasarnya sama dan mengarah pada proses yang dikenal sebagai casting out
nines.

3. Berdasarkan fakta bahwa 1001 = 71113 dan dengan anggapan saudara mengenali
kelipatan 7, 11 dan 13 dari bilangan yang terdiri atas paling banyak tiga angka, buat
skema keterbagian oleh 7, 11 atau 13!
4. Gunakan metoda di bagian ini untuk menyelesaikan ax 1 0 (mod ps) apabila (a, p)
= 1.
5. a. Untuk setiap s 2, buktikan (y + vps1) j y j + jy j1vps1 (mod ps)
n

b. Jika f(x) = ai x n i , buktikan f(y + vps1) j f(y)


i =0

n 1

(n i)a y
i =0

vp s 1 (mod ps).

n i 1

Hasil ini bisa digunakan untuk mengganti peran deret Taylor dalam pembahasan di
dalam teks.
6. Selesaikan x5 + x4 + 1 0 (mod 34)
7. Selesaikan x3 + x + 57 0 (mod 53)
8. Selesaikan x2 + 5x + 24 0 (mod 36)
9. Selesaikan x3 + 10x2 + x + 3 0 (mod 33)
10. Selesaikan x3 + x2 4 0 (mod 73)
11. Selesaikan x3 + x2 5 0 (mod 73)
2.7. Kongruensi Modulo Prima

Misalkan p adalah bilangan prima dan f(x) = a0xn + a1xn1 + + an adalah suatu
polinom dengan koefisien-koefisien bulat yang koefisien utamanya a0 0 (mod p).

52

Teorema 2.19

Jika derajat n dari f(x) lebih besar atau sama dengan p, maka ada dua kemungkinan:
a. setiap bilangan bulat adalah solusi dari f(x) 0 (mod p); atau
b. terdapat polinom g(x) dengan koefisien-koefisien bulat dan dengan koefisien
utama 1 sedemikian rupa sehingga derajat kongruensi g(x) 0 (mod p) kurang
dari p dan solusinya persis sama dengan solusi dari kongruensi f(x) 0 (mod p).
Bukti.

Setelah f(x) dibagi oleh xp x, diperoleh f(x) = q(x)(xp x) + r(x) di mana q(x)
adalah polinom dengan koefisien-koefisien bulat dan r(x) = 0 atau merupakan polinom
dengan koefisien-koefisien bulat berderajat kurang dari p. Teorema Fermat menyatakan
up1 1 (mod p), atau up u (mod p) atau up u 0 (mod p), untuk setiap bilangan u.

Jadi untuk setiap bilangan u berlaku f(u) = r(u) (mod p). Ini berarti, jika r(x) = 0 atau jika
setiap koefisien dari r(x) habis dibagi p, maka setiap bilangan bulat u adalah solusi dari
f(x) 0 (mod p). Ini membuktikan kemungkinan a.

Jika kemungkinan a tidak terjadi, maka kemungkinan lain adalah


r(x) = b0xm + b1xm1 + + bm,

dengan m < p dan (b0, p) = 1. Dalam hal ini, terdapat bilangan bulat c sedemikian rupa
sehingga cb0 1 (mod p). Karena jelas kedua kongruensi r(x) 0 (mod p) dan cr(x) 0
(mod p), yaitu xm + cb1xm1 + + cbm 0 (mod p), mempunyai solusi yang sama, maka
g(x) = cr(x) memenuhi syarat polinom yang disyaratkan dalam kemungkinan b.

Pernyataan g(u) cr(u) (mod p) harus diartikan bahwa koefisien-koefisien g(x)


dan cr(x) yang bersesuaian saling kongruen modulo p.
Teorema 2.20

Banyak solusi dari kongruensi f(x) 0 (mod p) berderajat n paling banyak adalah n.
Bukti.

Bukti berbentuk induksi pada derajat n dari kongruensi f(x) 0 (mod p). Jika n =
0, maka kongruensi berbentuk a0 0 (mod p) yang jelas tak memberikan solusi karena
(a0, p) = 1. Jika n = 1, maka kongruensi ekuivalen dengan bentuk kongruensi a0x b
(mod p) yang memberikan solusi sebanyak (a0, p) = 1 (cf. Teorema 2.13).

53

Selanjutnya sebagai hipotesis induksi, isi Teorema dianggap benar untuk semua
kongruensi berderajat kurang dari n. Andaikan terdapat n + 1 solusi-solusi u1, u2, , un+1
dari kongruensi f(x) 0 (mod p) yang berderajat n, maka ui uj (mod p), jika i j, atau
dengan kata lain, ui uj 0 (mod p) jika i j. Pada khususnya, un+1 u1, un+1 u2, ,
un+1 un semuanya bukan kelipatan p.

Mengingat a0 0 (mod p) merupakan koefisien utama dari f(x), maka polinom


g(x) = f(x) a0(x u1)(x u2) (x un)

berderajat kurang dari n, sebab kedua suku pertama a0xn , masing-masing dari polinom
a0(x u1)(x u2) (x un)

dan dari f(x), saling menghapuskan. Perhatikan, u1, u2, , un adalah n solusi dari g(x) 0
(mod p). Ada dua kemungkinan, g(x) = 0 atau g(x) adalah polinom berderajat k, 0 k < n.
Pertama kali dibuktikan bahwa g(x) = 0 atau setiap koefisiennya habis dibagi p.
Jika seandainya tidak demikian, maka g(x) = b0xk + b1xk1 + + bk dengan beberapa
koefisien bi saling prima dengan p. Misalkan bj adalah koefisien pertama yang saling
prima dengan p dan definisikan h(x) = bjxkj + bj+1xkj1 + + bk.
Kongruensi h(x) 0 (mod p) berderajat k j < n mempunyai n solusi u1, u2, , un
yang sama dengan n solusi g(x) 0 (mod p). Tetapi hal ini bertentangan dengan hipotesis
induksi yang mengasumsikan kebenaran Teorema untuk kongruensi berderajat kurang
dari n. Jadi disimpulkan g(x) = 0 atau semua koefisien g(x) habis dibagi p. Tetapi hasil ini
berakibat bahwa untuk setiap solusi u di antara ke n + 1 solusi u1, u2, , un+1 berlaku
f(u) a0(u u1)(u u2) (u un) (mod p).

Pada khususnya, kesamaan berlaku untuk solusi u = un+1 dari f(x) 0, sehingga
0 f(un+1) a0(un+1 u1)(un+1 u2) (un+1 un) (mod p),
yang kontradiksi dengan fakta bahwa a0, un+1 u1, un+1 u2, , un+1 un masing-masing
bukan kelipatan p. Kesimpulannya, pengandaian bahwa ada lebih dari n solusi u1, u2, ,
un, un+1 tidak benar.

Untuk selanjutnya, kita sudah bisa berasumsi bahwa kongruensi f(x) 0 (mod p)
berderajat n < p.

54

Akibat Teorema 2.21

Jika b0xn + b1xn1 + + bn 0 (mod p) mempunyai banyak solusi lebih besar


daripada n, maka setiap koefisien bi habis dibagi p.
Teorema 2.22

Kongruensi f(x) 0 (mod p) berderajat n dengan koefisien utama 1 mempunyai solusi


sebanyak n jika dan hanya jika f(x) adalah faktor dari xp x modulo p, yaitu jika dan
hanya jika xp x = f(x)q(x) + ps(x), di mana koefisien-koefisien dari q(x) dan s(x)
bulat dan s(x) = 0 atau s(x) berderajat kurang dari n.
Bukti.
Jika f(x) 0 (mod p) mempunyai n solusi, maka n < p. Setelah xp x dibagi f(x), maka

diperoleh xp x = f(x)q(x) + r(x) dengan r(x) = 0 atau r(x) berderajat kurang dari n. Untuk
setiap solusi u di antara n solusi dari f(x) 0 (mod p) berlaku up u 0 (mod p) sehingga
r(u) 0 (mod p). Jadi setiap u juga merupakan solusi dari r(x) 0 (mod p) sehingga r(x)

adalah polinom berderajat kurang dari n tetapi r(x) 0 (mod p) mempunyai n buah solusi.
Menurut Akibat Teorema 2.21, semua koefisien dari r(x) habis dibagi p, atau dengan kata
lain,
r(x) = ps(x).
Misalkan xp x = f(x)q(x) + ps(x), maka untuk setiap bilangan u berlaku
f(u)q(u) up u ps(u) 0 (mod p).

Ini berarti f(x)q(x) 0 (mod p) mempunyai p solusi. Karena q(x) berderajat p n, q(x) 0
(mod p) memiliki paling banyak p n solusi, katakan memiliki k solusi v1, v2, , vk
dengan k p n. Jika u adalah salah satu dari sisa p k bilangan yang bukan solusi dari
q(x) 0 (mod p), maka (q(u), p) = 1. Karena f(u)q(u) 0 (mod p), maka menurut

Teorema 1.9, f(u) 0 (mod p). Ini berarti kongruensi f(x) 0 (mod p) memiliki paling
sedikit p k p (p n) = n solusi. Bersama dengan Teorema 2.20, terbukti bahwa f(x)
0 (mod p) memiliki tepat n solusi.

55

Soal-soal.

1. Turunkan derajat kongruensi berikut menjadi berderajat 6.


a. x11 + x8 + 5 0 (mod 7)

b. x20 + x13 + x7 + x 2 (mod 7)

c. x15 x10 + 4x 3 0 (mod 7)

d. x15 x10 + 4x 3 0 (mod 11)

2. Buktikan dengan menggunakan Teorema 2.22 bahwa kongruensi 2x3 + 5x2 + 6x + 1


0 (mod 7) memiliki 3 solusi.
3. Buktikan bahwa x14 + 12x2 0 (mod 13) mempunyai tepat 13 solusi, jadi merupakan
kongruensi identitas.
4. Buktikan bahwa jika f(x) 0 (mod p) mempunyai j solusi-solusi x a1, x a2, , x
aj (mod p), maka terdapat polinom q(x) yang memenuhi
f(x) (x a1)(x a2) (x aj)q(x) (mod p).
Saran: Awali dengan menunjukkan bahwa terdapat q1(x) yang memenuhi
f(x) (x a1)q1(x) dan q1(x) 0 (mod p) memiliki solusi-solusi x a2, x a3,

, x aj (mod p). Selanjutnya gunakan induksi matematis.


2.8 Kongruensi Derajat Dua, Modulo Prima

Jika f(x) 0 (mod p) berderajat dua, maka f(x) = ax2 + bx + c dan a relatif prima
terhadap p. Karena kasus p = 2 mudah diselesaikan, diasumsikan p > 2. Jadi p ganjil dan
4af(x) = (2ax + b)2 + 4ac b2. Kita lihat, u adalah solusi dari f(x) 0 (mod p) jika dan
hanya jika 2au + b v (mod p) dengan v adalah solusi dari kongruensi
x2 b2 4ac (mod p).

Perhatikan (2a, p) = 1 sehingga untuk setiap solusi v, hanya terdapat tepat satu u modulo
p yang memenuhi kongruensi 2au + b v (mod p). Jadi ada korrespondensi 1-1 antara u

dengan v modulo p. Artinya, kongruensi derajat dua bisa dibawa ke bentuk kongruensi
x2 a (mod p).
Soal-soal.

Bawa kongruensi berikut ke bentuk x2 a (mod p).


a. 4x2 + 2x + 1 0 (mod 5)

b. 3x2 x + 5 0 (mod 7)

c. 2x2 7x 10 0 (mod 11)

d. x2 + x 1 0 (mod 13)

56

2.9 Residu Pangkat


Definisi 2.6

Jika xn a (mod p) mempunyai solusi, maka a disebut residu pangkat ke-n modulo p.

Diingatkan bahwa Z+ = {1, 2, 3, } adalah himpunan semua bilangan bulat positif yang
sering disebut bilangan asli. Lambang lain yang sering digunakan untuk Z+ adalah N.
Definisi 2.7

Misalkan a, m Z dengan m > 0 dan (a, m) = 1. Apabila


h = min{i Z+ | ai 1 (mod m) },
maka kita mengatakan a masuk ke pangkat h modulo m.

Teorema Euler menjamin (m) {i Z+ | ai 1 (mod m)} sehingga setiap a yang relatif
prima terhadap m masuk ke suatu pangkat h (m) modulo m. Apabila (m) dibagi h,
diperoleh (m) = qh + r, dengan 0 r < h. Tetapi karena ah 1 (mod m) berakibat aqh 1
(mod m), maka ar = ar1 = ar aqh = aqh + r = a(m) 1 (mod m), yaitu ar 1 (mod m).
Mengingat h = min{i Z+ | ai 1 (mod m) }, sedangkan ar 1 (mod m) dengan r < h,
maka r tak mungkin bulat positif. Jadi disimpulkan r = 0 dan (m) = qh, yaitu h|(m).
Diskusi ini memberikan bukti bagian pertama dari Teorema berikut.
Teorema 2.23

Jika a masuk ke pangkat h modulo m, maka h|(m). Lebih jauh, aj ak (mod m) jika
dan hanya jika h| j k.
Bukti (untuk bagian kedua Teorema).

Tanpa mengurangi berlaku umumnya bukti, anggap j > k. Karena (a, m) = 1,


kongruensi aj ak (mod m) ekuivalen dengan kongruensi ajk 1 (mod m). Pembuktian
diselesaikan dengan cara yang sama seperti pembuktian pernyataan pertama, setelah
mengganti (m) dengan j k.

57

Teorema 2.24

Jika a masuk ke pangkat h modulo m, maka ak masuk ke pangkat h/(h,k) modulo m,


yaitu
h/(h, k) = min{j Z+ | (ak)j 1 (mod m) }.
Bukti.

Menurut Teorema 2.23, (ak)j 1 (mod m) jika dan hanya jika h|kj. Tetapi h|kj jika
dan hanya jika {h/(h, k)}| {k/(h, k)}j. Karena {h/(h, k)}dan {k/(h, k)} saling relatif prima,
ini berarti h|kj jika dan hanya jika {h/(h, k)}| j. Jadi, bilangan positif terkecil j yang
memenuhi (ak)j 1 (mod m) adalah h/(h, k).
Teorema di atas menyatakan bahwa untuk setiap pangkat k dengan (h, k) = 1, jika
a masuk ke pangkat h modulo m, maka ak pun masuk ke pangkat h modulo m. Pada

khususnya, untuk setiap k dengan 1 k < h, banyaknya pangkat k dengan (h, k) = 1


adalah (h) dan untuk setiap pangkat k di antara (h) bilangan-bilangan ini, ak masuk ke
pangkat h modulus m. Untuk k dengan (h, k) > 1, ak masuk ke pangkat h = h/(h, k) < h
modulus m. Walaupun tidak eksplisit, tersirat dari hipotesis teorema bahwa (a, m) = 1.
Definisi 2.8

Jika a masuk ke pangkat (m) modulus m, maka a disebut sebuah akar primitif
modulo m.
Teorema 2.25

Jika p prima, maka terdapat sebanyak (p 1) akar-akar primitif modulo p.


Bukti.

Setiap bilangan bulat a, 1 a p 1, masuk ke suatu pangkat h modulo p dengan


h|p 1. Jika a masuk ke pangkat h modulo p, maka untuk setiap k berlaku (ak)h 1 (mod
p). Perhatikan, 1, a, a2, , ah1 adalah h bilangan-bilangan residu modulo p yang berbeda

sebab h p 1 dan 1, a, a2, , ap2 semuanya bilangan residu modulo p yang berbeda.
Menurut Teorema 2.20, ke h bilangan residu modulo p di atas adalah merupakan
kumpulan semua solusi dari kongruensi xh 1 (mo9d p) yang berderajat h. Penjelasan
yang diberikan setelah Teorema 2.24 menyatakan hanya (h) di antara ke h bilanganbilangan di atas yang masuk ke pangkat h modulo p sedangkan bilangan-bilangan lain
58

yang tersisa masuk ke pangkat lebih kecil h < h modulo p dengan h|p 1.
Karena setiap bilangan a 0 residu modulo p masuk ke suatu pangkat h modulo p,
maka jika (h) menyatakan banyak bilangan a yang masuk ke pangkat h modulo p
dengan h|p 1, jelas

(h) = p 1.

h| p 1

Perhatikan menurut paragraf terdahulu untuk setiap h|p 1, (h) = (h) atau(h)
= 0, jadi (h) (h). Sedangkan Teorema 2.17 menyatakan

( h) = p 1

sehingga

h| p 1

( (h) (h)) = 0

h| p 1

adalah jumlahan dengan hasil 0 tetapi dengan suku-suku (h) (h) 0. Disimpulkan
bahwa untuk setiap h|p 1, (h) = (h). Pada khususnya, (p 1) = (p 1).
Contoh 2.12

Misalkan p = 13. Karena (p) = p 1 = 12 adalah bilangan terkecil dengan 212 1


(mod 13), maka 2 adalah akar primitif modulo 13. Sebaliknya jika a = 3, maka h = 3
adalah bilangan bulat positif terkecil yang memenuhi ah = 33 = 27 1 (mod 13). Di
sini karena h (13), 3 bukanlah akar primitif modulo 13. Demikian pula, h = 6

(13) adalah bilangan bulat positif terkecil yang memenuhi 4h 1 (mod 13) sehingga
disimpulkan 4 bukan akar primitif modulo 13. Menurut Teorema 2.25, banyaknya
akar primitif modulo 13 adalah (12) = 4.
Teorema 2.26

Jika g adalah akar primitif modulo bilangan prima p, maka gj gk (mod p) jika dan
hanya jika j k (mod p 1). Pada khususnya, gj 1 (mod p) jika dan hanya jika
p 1| j. Juga himpunan g, g2, , gp1 membentuk sistem residu tereduksi modulo p
sedemikian rupa sehingga jika a saling prima dengan p, maka terdapat satu dan
hanya satu gj dengan gj a (mod p).
Bukti.

59

Bagian pertama teorema adalah keadaan khusus dari Teorema 2.23 untuk kasus a
masuk ke pangkat h = p 1 yang harus membagi j k, apabila gj gk (mod p). Seperti isi
pembuktikan Teorema 2.25, semua h = p 1 bilangan-bilangan g, g2, , gp1 membentuk
sistem residu tereduksi modulo p. Ini berarti, setiap bilangan a yang saling prima dengan
p bersekawan dengan tepat salah satu di antaranya, katakan dengan gj, melalui

kongruensi gj a (mod p).


Pangkat j dengan gj a (mod p) disebut indeks dari a. Selain tergantung pada a,
indeks ini juga tergantung pada g dan p.
Teorema 2.27

Diketahui p adalah prima dan (a, p) = 1. Kongruensi


xn a (mod p)
memiliki (n, p 1) solusi jika a(p1)/(n, p1) 1 (mod p) dan tak memiliki solusi jika
a(p1)/(n, p1) 1 (mod p).
Bukti.

Misalkan b = (n, p 1). Jika xn a (mod p) memiliki solusi u, maka


a(p1)/b un(p1)/b (mod p) u(p1)n/b 1 (mod p)

sebab berdasarkan Teorema Fermat, u(p1) 1 (mod p) dan n/b adalah bilangan bulat. Jadi
jika a(p1)/b 1 (mod p), maka kongruensi xn a (mod p) tak memiliki solusi.
Sebaliknya apabila a(p1)/b 1 (mod p), Menurut Teorema 2.25 dan 2.26, terdapat
suatu akar primitif g modulo p dan suatu pangkat j sedemikian rupa sehingga gj a (mod
p). Jadi gj(p1)/b a(p1)/b 1 (mod p) sehingga menurut Teorema 2.26, p 1|j(p 1)/b,

atau j(p 1)/b adalah kelipatan p 1 sehingga disimpulkan b| j.


Setiap solusi dari xn a (mod p), jika ada, bisa ditulis sebagai perpangkatan dari g,
katakan gy, modulo p. Akibatnya, setiap solusi dalam x, jika ada, dari xn a (mod p)
bersekawan 1-1 dengan sebuah solusi dalam y dari gyn gj (mod p). Menurut Teorema
2.26, kongruensi ini mempunyai solusi jika dan hanya jika yn j (mod p 1) memiliki
solusi. Menurut Teorema 2.13, karena b| j, solusi dari yn j (mod p 1) ada dan
banyaknya b. Karena keberadaan pengawanan 1-1 antar solusi kedua kongruensi di atas,
juga ada sebanyak b = (n, p 1) solusi untuk kongruensi xn a (mod p).

60

Akibat Teorema 2.28

Jika p > 2 adalah prima dan (a, p) = 1, maka kongruensi x2 a (mod p) memiliki dua
solusi jika a(p1)/2 1 (mod p) dan tak memiliki solusi jika a(p1)/2 1 (mod p).
Bukti.

Langsung dari Teorema 2.27 dengan n = 2 dan b = (2, p 1) = 2, karena p 1


genap. Kongruensi x2 a (mod p) memiliki dua solusi jika a(p1)/2 1 (mod p) dan tak
memiliki solusi jika a(p1)/2 1 (mod p). Tinggal dibuktikan bahwa a(p1)/2 1 (mod p)
jika dan hanya jika a(p1)/2 1 (mod p).
Dari Teorema Fermat,
(a(p1)/2 + 1)(a(p1)/2 1) ap1 1 0 (mod p).
Jadi dalam sistem residu tereduksi modulo p, a(p1)/2 hanya memiliki dua nilai, a(p1)/2 1
(mod p) atau a(p1)/2 1 (mod p).
Soal-soal.

1. Cari sebuah akar primitif dari bilangan prima 3, bilangan prima 5, bilangan prima 11
dan bilangan prima 23.
2. Ada berapa banyak akar primitif dari 17?
3. Masuk ke pangkat berapakah masing-masing bilangan 1, 2, 3, 4, 5, 6 modulo 7?
Masuk ke pangkat berapakah bilangan-bilangan tersebut dalam modulo 11?
4. Gunakan Teorema 2.27 untuk menguji apakah a = 2 merupakan residu pangkat ke-9
modulo 13 (cf. Definisi 2.6), yaitu apakah x9 2 (mod 13) memiliki solusi? Demikian
pula, uji apakah a = 3 merupakan residu pangkat ke-4 modulo dengan menguji
keberadaan solusi dari kongruensi yang bersesuaian. Jika kongruensi ini memiliki
solusi, berapa banyak solusi-solusinya?
5. Dengan menggunakan Teorema 2.26, buktikan bahwa apabila n 1 (mod 12), maka a
= 2 merupakan residu pangkat ke-n modulo 13.
6. Cari sebanyak (3, 12) = 3 solusi dari kongruensi x3 1 (mod 13)!
7. Misalkan p > 2 prima. Buktikan a masuk ke pangkat h modulo p jhj a 1 (mod p).
8. Jika a masuk ke pangkat h modulo m, buktikan bahwa a, a2, a3, ..., ah satu sama lain
tidak kongruen modulo m.

61

2.10 Teori Bilangan Ditinjau Secara Aljabar

Di sini para pembaca dianggap telah mengenal konsep grup, gelanggang dan
lapangan walaupun kita akan mendefinisikan ulang konsep-konsep ini. Paling tidak, para
pembaca dianggap mengenal konsep operator biner pada suatu himpunan H sebagai
fungsi (dua peubah) dengan daerah asal
H H = {(x, y) | x, y H}

dan daerah hasil H.


Umumnya dalam pelajaran matematika, peta atau nilai dari fungsi di titik (x, y)
H H biasanya ditulis dalam notasi prefix: (x, y). Tetapi di sini, kita menuliskannya

dalam notasi infix:


x y.

Fakta bahwa nilai fungsi (yang tunggal) ini adalah unsur dari H, yaitu untuk setiap
pasang (x, y) H H bersekawan dengan tepat satu x y H, lebih dikenal sebagai
konsep tertutup dari operator biner dalam H dan dinyatakan dengan bahasa sedikit
berbeda: untuk setiap dua unsur x, y H berlaku hasil operasi (yang tunggal) antara
unsur x dan y berada dalam H ( x y H).
Definisi 2.9

Suatu grup G adalah himpunan unsur-unsur a, b, c, bersama sebuah operator


biner sedemikian rupa sehingga ke-4 aksioma berikut

(1) G bersifat tertutup terhadap operator biner ; artinya untuk setiap a, b G


berlaku a b G.

(2) Di dalam G, operator biner bersifat asosiatif; artinya untuk setiap a, b, c G


berlaku a (b c) = (a b) c.

(3) G memiliki unsur identitas eG terhadap operator ; artinya untuk setiap a G


berlaku a eG = a = eG a.

(4) Setiap unsur a G memiliki balikan tunggal a G. Artinya, untuk setiap a G,


terdapat satu dan hanya satu unsur a G yang memenuhi a a = eG = a a .
G disebut grup abel atau grup komutatif jika untuk setiap pasang a, b G berlaku

a b = b a.

62

G disebut grup hingga jika banyak unsur-unsur G berhingga. Jika tidak demikian, G

disebut grup tak hingga. Banyak unsur dari grup hingga G disebut ordo dari G.
Dalam kaitannya dengan teori bilangan, kita hanya membahas grup-grup abel dan
umumnya merupakan grup hingga kecuali grup abel Z (atau yang isomorf) dari semua
bilangan-bilangan bulat terhadap operator aritmatik +.
Contoh 2.13

Terhadap operasi tambah kongruen modulo 10, himpunan


Z10 = {0, 1, 2, 3, 4, 5, 6, 7, 8, 9}

membentuk grup. Berdasarkan tabel operasi tambah antara unsur-unsur Z10 di bawah,
langsung terlihat bahwa aksioma-aksioma grup abel dipenuhi oleh Z10.

+ 0

9 0

9 0

9 0

9 0

9 0

9 0

9 0

9 0

9 0

Grup Z10 Terhadap Operator +


Definisi 2.10

Dua buah grup, grup G dengan operator biner dan grup G dengan operator biner
, dikatakan isomorf jika terdapat fungsi: G G yang 1-1 dan pada sedemikian
rupa sehingga untuk setiap a, b G berlaku (a b) = (a) (b).

Diingatkan, sistem residu lengkap modulo m adalah himpunan x1, x2, , xm


sedemikian rupa sehingga untuk setiap bilangan bulat y terdapat satu dan hanya satu xj
dengan y xj (mod m). Untuk satu nilai m, sistem residu lengkap modulo m tidak tunggal.
63

Contoh 2.14

Misalnya untuk m = 6, sebuah sistem residu lengkap modulo 6 bisa ditulis sebagai
himpunan
G = {0, 1, 2, 3, 4, 5}.

Sistem residu lengkap modulo 6 yang lain bisa ditulis sebagai


G = {30, 1, 14, 3, 2, 5}.

Menurut definisi sistem residu lengkap, sembarang bilangan bulat, misalnya 40,
kongruen modulo 6 dengan satu dan hanya satu bilangan dalam sistem residu lengkap.
Terhadap sistem residu lengkap G = {0, 1, 2, 3, 4, 5}, 40 4 (mod 6), yaitu 4 adalah
satu-satunya bilangan dalam G yang kongruen modulo 6 dengan 40. Sebaliknya
terhadap sistem residu lengkap G = {30, 1, 14, 3, 2, 5}, 40 2 (mod 6) , yaitu 2
adalah satu-satunya bilangan dalam G yang kongruen modulo 6 dengan 40. Bisa
ditunjukkan bahwa kedua sistem residu lengkap G dan G membentuk dua grup yang
isomorf.
Sistem residu lengkap G = {0, 1, 2, 3, 4, 5} modulo 6 secara formal membentuk grup
apabila untuk setiap pasang unsur a, b G, misalnya a = 4 dan b = 5, penulisan
kongruensi
a + b 3 (mod 6)

diganti dengan penulisan


a b = 3.

Dengan mudah dibuktikan bahwa G dengan operator tambah memenuhi ke-4


syarat definisi grup seperti yang dinyatakan oleh Definisi 2.9.
Demikian pula, sistem residu lengkap G = {30, 1, 14, 3, 2, 5} secara formal
membentuk grup apabila untuk setiap pasang unsur a, b G, penulisan kongruensi
a' + b 5 (mod 6)

diganti dengan penulisan


a b = 5.

Kedua grup isomorf sebab pengawanan : G G dengan (0) = 30, (1) = 1, (2) =
14, (3) = 3, (4) = 2 dan (5) = 5 memenuhi persyaratan fungsi 1-1 dan pada yang
memenuhi sifat: untuk setiap a, b G berlaku (a b) = (a) (b). Misalnya kita

64

lihat,

(2 3) = (5) = 5 = 14 (3) = (2) (3).


di mana kita bisa menulis 5 = 14 (3) sebab 14 + (3) 5 (mod 6). Dengan cara
yang sama, bisa dibuktikan bahwa untuk sembarang a, b G berlaku

(a b) = (a) (b).
Grup G = {0, 1, 2, 3, 4, 5} biasa diberi lambang Z6 (Lihat Z10 dalam Contoh 2.13).
0 1 2 3 4 5

30 1 14 3 2 5

C0 C1 C2 C3 C4 C5

30 30 1 14 3 2 5

C0 C0 C1 C2 C3 C4 C5

1 14 3 2 5 30

C1 C1 C2 C3 C4 C5 C0

14 14 3 2 5 30 1

C2 C2 C3 C4 C5 C0 C1

3 3 2 5 30 1 14

C3 C3 C4 C5 C0 C1 C2

2 2 5 30 1 14 3

C4 C4 C5 C0 C1 C2 C3

5 30 1 14 3 2

C5 C5 C0 C1 C2 C3 C4

Tabel Grup G dan G dalam Contoh 2.14

Grup Dalam Contoh 2.15

Tiga Penyajian Grup Tambah Yang Isomorf Dengan Z6

Cara lain untuk melihat sistem residu lengkap modulo m sebagai grup adalah
dengan menggunakan kelas-kelas residu yang membagi himpunan semua bilangan bulat
menjadi m kelas-kelas residu. Dua buah bilangan bulat a dan b berada di kelas residu
yang sama jika dan hanya jika a b (mod m).
Contoh 2.15

Misalkan m = 6. Ke-6 kelas-kelas residu adalah


C0 = {, 18, 12, 6, 0, 6, 12, 18, };
C1 = {, 17, 11, 5, 1, 7, 13, 19, };
C2 = {, 16, 10, 4, 2, 8, 14, 20, };
C3 = {, 15, 9, 3, 3, 9, 15, 21, };
C4 = {, 14, 8, 2, 4, 10, 16, 22, };
C5 = {, 13, 7, 1, 5, 11, 17, 23, }.

Kemudian kita mendefinisikan operasi tambah antara kelas-kelas residu dengan cara
sebagai berikut. Hasil tambah Ci Cj diperoleh dari hasil tambah antara sembarang wakil
ai Ci dengan wakil aj Cj. Jika ai + aj Ck, maka kita definisikan Ci Cj = Ck. Yang

65

perlu diperhatikan, hasil tambah Ck ini well-defined, tidak tergantung pada pilihan kedua
wakil-wakil ai Ci dan aj Cj.
Sebagai ilustrasi, untuk menentukan hasil tambah C4 C5, pilih sembarang wakil
dari C4 dan C5, misalnya 14 C4 dan dan 5 C5. Karena 14 + 5 = 9 C3, maka
disimpulkan C4 C5 = C3. Bisa dicoba wakil-wakil lain, misalnya 10 C4 dan 7 C5,
juga akan diperoleh 10 + (7) = 3 C3. Mudah dibuktikan, himpunan
{C0, C1, C2, C3, C4, C5}
dengan operator biner seperti yang didefinisikan di atas membentuk sebuah grup.
Untuk selanjutnya, kita hanya menggunakan lambang operator biner tambah +
atau operator perkalian yang ditulis dalam notasi juxtaposition. Artinya, perkalian antara
dua unsur a dan b cukup ditulis sebagai
ab

atau
(a)(b),
tanpa ada penggunaan lambang operator biner. Pada khususnya, kedua lambang operator
biner dan yang didefinisikan di dalam dua sistem residu lengkap G = {0, 1, 2, 3,
4, 5} dan G = {30, 1, 14, 3, 2, 5} modulo 6 selanjutnya cukup diberi lambang + dan
disebut operator tambah. Demikian pula operator biner antara kelas-kelas residu C0,
C1, , C5 selanjutnya dilambangkan oleh operator tambah + tersebut.
Teorema 2.29

Setiap sistem residu lengkap modulo m membentuk grup terhadap operator tambah.
Dua sistem residu lengkap modulo m membentuk dua grup yang saling isomorf. Di
sini kita membicarakan grup tambah.
Soal-soal.

1. Di antara himpunan-himpunan yang dilengkapi operator biner berikut, yang manakah


yang membentuk grup?
a. Himpunan semua bilangan genap {2z | z Z} terhadap operator + aritmatik.
b. Himpunan semua bilangan ganjil {2z + 1 | z Z} terhadap operator + aritmatik.
c. Himpunan semua bilangan bulat Z terhadap operator (pengurangan) aritmatik.

66

d. Himpunan semua bilangan bulat kelipatan 7, H = {7z | z Z} terhadap operator +

aritmatik.
e. Himpunan semua bilangan rasional Q terhadap operator + aritmatik.
f. Himpunan semua bilangan rasional Q terhadap operator kali aritmatik.
g. Himpunan semua bilangan rasional tak nol Q {0} terhadap operator kali

aritmatik.
2. Misalkan G = {(1, 1), (1, 1), (1, 1), (1, 1)} dan operasi binar dalam G
didefinisikan (a, b)(c, d) = (ac, bd). Apakah G merupakan grup?
2.11 Grup Perkalian, Gelanggang dan Lapangan

Menurut Definisi 2.3, suatu sistem residu tereduksi modulo m adalah himpunan
bilangan-bilangan r1, r2, , rk sedemikian rupa sehingga untuk setiap i, (ri, m) = 1 dan
untuk setiap pasang indeks i dan j yang berbeda, ri rj (mod m) serta untuk setiap
bilangan bulat x dengan (x, m) = 1, berlaku x ri (mod m) untuk suatu indeks i, 1 i k.
Telah kita ketahui bahwa banyaknya bilangan-bilangan dalam sistem residu tereduksi
modulo m adalah k = (m).
Teorema 2.30

Setiap sistem residu tereduksi modulo m membentuk grup terhadap operator kali
aritmatik dan ordo dari grup ini adalah(m). Di sini kita berbicara grup kali.

Secara umum, untuk setiap bilangan bulat positif m > 1,


Zm = {x Z | 1 x < m dan (x, m) = 1}

membentuk suatu grup yang terdiri atas sebanyak (m) unsur-unsur, selain merupakan
salah satu sistem residu tereduksi modulo m.
Contoh 2.16

Terhadap operasi kali kongruen modulo 10, himpunan


Z10 = {x Z10 | (x, 10) = 1}

= {1, 3, 7, 9}
membentuk grup. Berdasarkan tabel operasi kali antara unsur-unsur Z10 berikut,
langsung terlihat bahwa aksioma-aksioma grup abel dipenuhi oleh Z10.
67

Ditinjau dari teori bilangan, Z10 adalah salah satu dari sistem residu tereduksi
modulo 10. Karena terdapat sebanyak tak hingga sistem residu tereduksi modulo 10
yang lain, misalnya {1, 1, 3, 3}, {11, 33, 43, 49}, ..., dsb, tabel grup Z10 bisa
diwakili oleh salah satu di antaranya. Tabel grup Z10 yang diwakili {1, 1, 3, 3}
juga ditampilkan dalam tabel berikut.

3 1

3 3

1 1 3

Dua Penyajian Grup Z10

Contoh 2.17

Sistem residu tereduksi modulo 9 bisa dinyatakan sebagai himpunan


G = {1, 2, 4, 5, 7, 8}.

Selanjutnya, kongruensi-kongruensi
(2)(4) 8 (mod 9),

(4)(5) 2 (mod 9)

dan

(5)(7) 1 (mod 9) dsb,

masing-masing akan ditulis sebagai perkalian


(2)(4) = 8,

(4)(5) = 2

dan

(5)(7) = 1, dsb

Terhadap operator perkalian ini, dengan mudah dibuktikan bahwa G adalah sebuah
grup dengan 1 sebagai unsur identitas.
Unsur 21, balikan dari unsur 2, adalah 5 sebab (2)(5) = 1. Kita boleh menulis
21 = 5.
Tentu saja 51 = 2 (balikan dari 5 adalah 2). Dengan cara yang sama diperoleh
41 = 7
dan

81 = 8.

Demikian pula, sistem residu tereduksi modulo 9 yang lain, misalnya


G = {10, 2, 5, 4, 25, 26}

68

akan membentuk grup terhadap operator kali. Di dalam grup ini, berlaku (10)(2) = 2,
(5)(7) = 10, (5)1 = 25, dst.
Kedua grup G dan G isomorf karena dari pengawanan : G G dengan (1) = 10,

(2) = 2, (4) = 5, (5) = 4, (7) = 25 dan (8) = 26 bisa dibuktikan bahwa untuk
setiap a, b G berlaku (ab) = (a)(b). Misalnya, ((2)(4)) = (8) = 26 = (2)(5) =

(2)(4). Di sini kita bisa menulis 26 = (2)(5) sebab dengan menggunakan bilanganbilangan dalam sistem residu tereduksi G, (2)(5) 26 (mod 9).
Teorema 2.31

Dalam setiap grup G dan setiap a, b, c G berlaku implikasi (Hukum Pembatalan)


ac = bc

a = b.

Jika G berhingga dan eG adalah unsur identitas dari G, maka untuk setiap a G
terdapat tepat satu bilangan bulat positif terkecil r dengan ar = eG.
Bukti.

Bagian pertama Teorema bisa dibuktikan dengan mengalikan dari kiri masing-masing
ruas ab = ac dengan a1. Untuk membuktikan bagian kedua, perhatikan deret
eG, a, a2, a3, a4, ... .

Karena deret ini terdiri atas unsur-unsur grup hingga G, tentu ada unsur yang berulang.
Artinya terdapat dua bilangan bulat positif s dan t dengan s t dan as = at. Tanpa
mengurangi berlaku umumnya bukti, anggap s < t. Karena as = at ekuivalen dengan ats =
eG, maka himpunan H = {i Z+ | ai = eG} dan memuat r = min H.
Definisi 2.11

Misalkan G dengan identitas eG adalah grup dan a G. Jika as = eG untuk suatu s


Z+, unsur a disebut berordo hingga dan min{i Z+ | ai = eG} disebut ordo dari a.

Jika tak ada s Z+ yang memenuhi as = eG, maka a dikatakan berordo tak hingga. G
disebut grup sikilik jika terdapat a G sedemikian rupa sehingga unsur-unsur

..., a4, a3, a2, a1, eG, a, a2, a3, a4, ...
meliputi seluruh unsur-unsur G. Dalam hal ini, unsur a dikatakan merentang G dan
disebut perentang dari G.

69

Teorema 2.31 menyatakan bahwa setiap unsur dari grup hingga berordo hingga.
Secara umum, setiap grup G memiliki paling sedikit satu unsur eG yang berordo hingga.
Bahkan ada grup tak hingga yang semua unsur-unsurnya berordo hingga.
Jika sebuah grup hinga G adalah grup siklik dengan perentang a, maka
G = {eG, a, a2, ..., , ar1}

di mana r adalah ordo dari a. Perpangkatan lain dari a adalah sama dengan salah satu dari
eG, a, a2, ..., , ar1.
Teorema 2.32

Ordo dari setiap unsur dari G membagi ordo dari G. Jika G berodo n maka untuk
setiap a G berlaku an = eG.
Bukti.

Misalkan a G berordo r sehingga


eG, a, a2, ..., , ar1

(2.10).

adalah r unsur-unsur G yang berbeda. Jika ordo G adalah n = r, teorema otomatis terbukti.
Sebaliknya jika ordo G melebihi r, maka terdapat b1 G yang berbeda dari r unsur-unsur
tersebut. Selanjutnya mudah dibuktikan bahwa
b1, ab1, a2b1, ..., , ar1b1

(2.11)

adalah r unsur-unsur G yang berbeda satu sama lain dan berbeda dari r unsur-unsur G
dalam (2.10). Pertama kali dibuktikan semua unsur dalam (2.11) berbeda satu sama lain,
sebab seandainya aib1 = ajb1 dengan i j, maka dari Teorema 2.31 (Hukum Pembatalan)
diperoleh ai = aj, suatu kontradiksi.
Untuk membuktikan setiap unsur dalam ekspresi (2.11) berbeda dengan setiap unsur
dalam (2.10), andaikan aib1 = aj untuk suatu i dan j. Setelah kedua ruas dikali ai dari kiri,
diperoleh b1 = aji, yaitu b1 sama dengan salah satu di antara unsur ak dalam ekspresi
(2.10), kembali terjadi kontradiksi sehingga pengandaian aib1 = aj harus diingkar.
Jika ordo dari G adalah n = 2r, an = a2r = (ar)2 = eG2 = eG dan teorema terbukti. Apabila
ordo G melebihi 2r, maka pasti terdapat b2 G yang berbeda dari 2r unsur-unsur yang
diekspresikan melalui ekspresi (2.10) dan (2.11). Dengan cara yang sama bisa dibuktikan
bahwa r unsur-unsur
b2, ab2, a2b2, ..., , ar1b2

(2.12)
70

berbeda satu sama lain dan semua berbeda dari 2r unsur-unsur yang disajikan melalui
ekspresi (2.10) dan (2.11). Karena G berhingga, proses ini pada suatu saat berhenti pada
barisan r unsur-unsur terakhir
bk1, abk1, a2bk1, ..., , ar1bk1,

(2.13)

dan tak ada lagi unsur G di luar kr unsur-unsur yang dinyatakan melalui ekspresi (2.10),
(2.11), .... sampai terakhir: ekspresi (2.13). Jadi n = kr adalah ordo G dan bagian pertama
dari teorema terbukti. Lebih jauh, untuk setiap a G berlaku an = akr = (ar)k = eGk = eG.
Perlu dicatat, dari Teorema 2.32 bisa diturunkan Teorema Fermat (Teorema 2.7)
dan Teorema Euler (Teorema 2.8) dengan cara memilih hmpunan bilangan-bilangan bulat
Zm = { x Z | 1 x < m, (x, m) = 1} sebagai grup G. Pada saat menggunakan grup ini,

kita perlu menterjemahkan bahasa teori bilangan ke bahasa teori grup. Dengan cara yang
sama kita, menterjemahkan bahasa dalam Definisi 2.7 a masuk ke pangkat h modulo m
menjadi a Zm berordo h. Juga dalam Definisi 2.8, istilah akar primitif modulo m
diterjemahkan menjadi perentang dari grup Zm.
Definisi 2.12

Suatu gelanggang G adalah sebuah himpunan dengan paling sedikit dua unsur dan
operator biner dan yang memenuhi syarat: G terhadap merupakan grup abel,
bersifat tertutup terhadap , operator bersifat asosiatif dan distributif (relatif
terhadap operator ). Identitas dari disebut unsur nol 0, identitas dari disebut
unsur satuan 1. Jika G {0} membentuk grup abel terhadap , G disebut lapangan.

Kedua operator dan masing-masing biasa disebut operator tambah dan


operator kali. Persyaratan untuk operator yang disebut dalam Definisi 2.12 di atas
cukup ditulis dalam notasi juxtaposition (tanpa menulis lambang operator ) sebagai
a(bc) = (ab)c, a(b + c) = ab + ac, (a + b)c = ac + bc.

Pada umumnya unsur-unsur a, b dan c bukan bilangan sehingga kedua operator


dan bukan operator tambah dan kali aritmatik. Tetapi dalam buku ini, kita hanya
membicarakan grup yang berunsurkan bilangan-bilangan dengan operator tambah dan
kali kongruen modulo m.

71

Teorema 2.33

Untuk setiap m > 1, himpunan Zm = {0, 1, 2, ..., m 1} terhadap operator tambah


dan kali modulo m merupakan gelanggang. Gelangganng ini merupakan lapangan
jika dan hanya jika m prima.
Bukti.

Telah dibuktikan dalam Teorema 2.29, setiap sistem residu lengkap modulo m terhadap
operator tambah modulo m merupakan grup abel. Sifat asosiatif dan distributif dari
operator kali modulo m diturunkan dari sifat yang sama di antara bilangan-bilangan real.
Jadi Zm merupakan gelanggang.
Selanjutnya dari Teorema 2.30, setiap sistem residu tereduksi modulo m membentuk grup
terhadap operator kali modulo m. Jika m = p adalah bilangan prima, maka sistem residu
tereduksi {1, 2, ..., p 1} = Zp {0} membentuk grup abel. Jadi Zp merupakan lapangan.
Dari lain pihak, jika m bukan bilangan prima, maka terdapat a, b Z dengan 1 < a b <
m sedemikian rupa sehingga m = ab. Dalam hal ini, {1, 2, ..., m 1} = Zm {0} tidak

membentuk grup sebab kongruensi ax 1 (mod m) tak memiliki solusi (Teorema 2.13)
yang berarti unsur a Zm {0} tak memiliki balikan. Sebagai akibatnya, Zm bukan
lapangan.
Soal-Soal

1. Buktikan bahwa grup kali Z9 isomorf dengan grup tambah Z6.


2. Buktikan bahwa untuk setiap bilangan bulat m > 1, grup tambah Zm siklik dengan 1
sebagai salah satu perentangnya. Kemudian buktikan setiap unsur Zm bisa berperan
sebagai perentang.
3. Berapakah nilai terkecil m sehingga grup kali Zm tidak siklik?
4. Buktikan Teorema 2.32 dengan cara yang analog dengan pembuktian Teorema 2.8
5. Buktikan Teorema 2.32 dengan cara yang analog dengan pembuktian Teorema 2.8.
Sebaliknya buktikan Teorema 2.8 dengan mengikuti cara pembuktian Teorema 2.32.
6. Buktikan Z adalah gelanggang yang bukan lapangan.
7. Buat tabel operasi kali dan tambah antar unsur-unsur Z7.
8. Misalkan a|m, katakan m = aq dengan 1 < a < m. Buktikan {0, a, 2a, ..., (q 1)a}
dengan operasi tambah dan kali modulo m adalah gelanggang yang bukan lapangan.

72

3. Resiprositi Kuadratik
3.1 Residu Kuadratik
Definisi 3.1

Jika m bulat positif, setiap a dengan (a, m) = 1 disebut residu kuadratik modulo m
jika kongruensi x2 a (mod m) ada solusinya. Jika tak ada solusinya, maka a disebut
nonresidu kuadratik modulo m.

Apakah a + km adalah sebuah residu atau non-residu kuadratik tergantung pada apakah a
adalah residu atau non-residu, modulo m, maka semua bilangan-bilangan residu atau
nonresidu kuadratik adalah semua bilangan-bilangan dalam sistem residu (tereduksi)
modulo m. Bilangan residu kuadratik modulo 5 adalah 1 dan 4, sementara bilangan
nonresidu kuadratik modulo 5 adalah 2 dan 3.
Definisi 3.2

( )

Jika p > 2 prima dan (a, p) = 1 maka simbol Legendre ap didefinisikan bernilai 1
jika a adalah residue kuadratik dan bernilai 1 jika a adalah nonresidu kuadratik,
modulo p.

Karena untuk setiap k Z, kongruensi x2 a + kp (mod p) ekuivalen dengan kongruensi


x2 a (mod p) untuk sembarang bilangan bulat k dan sembarang bilangan prima p

berlaku kesamaan

( a+pkp ) = ( ap ) .

Teorema 3.1

Jika p > 2 prima dan a, b Z dengan (a, p) = 1 dan (b, p) = 1. maka

( )
b. ( ap ) ( bp ) ( ab
p)

a. ap a(p1)/2 (mod p)

( ) ( )
( 1p ) = 1 dan ( p1 ) (1)

c. a b (mod p) berakibat ap bp

( ) = 1,

d. ap

(p1)/2

(3.1)

Bukti.

Bagian a langsung diturunkan dari Akibat Teorema 2.28, bagian b diperoleh


dengan menerapkan bagian a dan bagian c diperoleh dengan menerapkan b = a + kp pada
ekspresi (3.1). Dua kesamaan pertama di bagian d diperoleh langsung dari definisi
sedangkan kesamaan terakhir diperoleh dari a.
Pembuktian teorema berikut menggunakan pilihan sistem residu tereduksi modulo p
khusus: 1, 2, , p 1. Dalam hal ini, nilai rata-rata ke-(p 1) bilangan-bilangan tak nol
dalam sistem residu lengkap ini adalah p/2.
Teorema 3.2 (Lemma Gauss)

Misalkan p > 2 prima dan (a, p) = 1. Perhatikan semua bilangan-bilangan bulat a, 2a, 3a,

, (p 1)a/2 dan penyajian (p 1)/2 bilangan-bilangan ini sebagai bilagan-bilangan


positif terkecil dalam sistem residu tereduksi modulo p. Jika k menyatakan banyaknya
bilangan-bilangan di antara (p 1)/2 bilangan-bilangan residu tereduksi modulo p yang
nilainya melebihi p/2, maka

( ap ) = (1) .
k

Bukti.

Dalam penyajian (p 1)/2 bilangan-bilangan bulat a, 2a, 3a, , (p 1)a/2 ke


dalam sistem residu tereduksi {1, 2, , p 1}, misalkan r1, r2, , rk menyatakan semua
bilangan-bilangan residu tak nol yang melebihi p/2 dan s1, s2, , sl adalah bilangan
residu yang tersisa. Dengan kata lain, bilangan-bilangan residu ri dan si memenuhi
p +1 p + 2

p1

a ri (mod p), ri { 2 , 2 ,..., p 1 } dan a sj (mod p), sj {1, 2, , 2 }


di mana , {1, 2, ..., (p 1)/2} selalu berbeda (sehingga + < p 1).
Jadi r1, r2, , rk, s1, s2, , sl adalah sekumpulan k + l = (p 1)/2 bilangan-bilangan
residu yang berbeda. Perhatikan bahwa untuk setiap i = 1, 2, , k berlaku 0 < p ri p/2
dan masing-masing p r1, p r2, , p rk berbeda satu sama lain dan tak satu pun yang
sama dengan sj, 1 j l. Sebab seandainya ada suatu bilanngan p ri sedemikian rupa
sehingga p ri = sj, maka p a a (mod p) untuk suatu , {1, 2, ..., (p 1)/2}.
Ini berakibat a a (mod p) atau a + a 0 (mod p). Karena (a, p) = 1, bisa ditarik

74

kesimpulan bahwa + 0 (mod p), yaitu + adalah kelipatan positif dari p,


kontradiksi dengan fakta bahwa jumlah + selalu kurang dari p 1.
Jadi p r1, p r2, , p rk, s1, s2, , sl adalah (p 1)/2 barisan bilanganbilangan residu yang saling berbeda dan masing-masing nilainya lebih kecil dari p/2.
Artinya, unsur-unsur dari barisan ini tidak lain adalah seluruh (p 1)/2 bilangan-bilangan
residu awal 1, 2, , (p 1)/2 dalam sistem residu tereduksi {1, 2, , p 1}, mungkin
hanya berbeda urutan. Akibatnya, hasil kali sebanyak (p 1)/2 unsur-unsur ini bisa
dinyatakan dengan dua cara berbeda tetapi sama nilainya;
(p r1)(p r2)(p r3) (p rk)(s1)(s2) (sl) = (1)(2)(3) (p 1)/2.
Dari sini diperoleh kongruensi-kongruensi
( r1)( r2)( r3) ( rk)(s1)(s2) (sl) (1)(2)(3) (p 1)/2 (mod p);
(1)k(r1)(r2)(r3) (rk)(s1)(s2) (sl) (1)(2)(3) (p 1)/2 (mod p);
(1)k(a)(2a)(3a) (p 1)a/2 (1)(2)(3) (p 1)/2 (mod p)
Dengan membatalkan hasil kali (2)(3) (p 1)/2 di kedua ruas, diperoleh (1)ka(p1)/2
1 (mod p), artinya (1)k a(p1)/2 (mod p).
Contoh 3.1

Misalkan p = 13 dan pilih a = 4. Dalam sistem residu tereduksi modulo 13, perhatikan
barisan yang terdiri atas (p 1)/2 = 6 bilangan-bilangan
a = 4, 2a = 8, 3a = 12, 4a = 16, 5a = 20, 6a = 24

yang kongruen modulo 13 dengan barisan dari bilangan-bilangan residu


4, 8, 12, 3, 7, 11
modulo 13. Di antara ke- 6 bilangan residu ini, terdapat k = 4 bilangan residu yang
nilainya lebih dari p/2 = 6.5, yaitu
r1 = 8,

r2 = 12,

r3 = 7

dan

r4 = 11.

Sisanya ada sebanyak l = 2 bilangan residu


s1 = 4 dan s2 = 3.

Ke-6 bilangan-bilangan p r1 = 13 8 = 5, p r2 = 13 12 = 1, p r3 = 13 7 = 6, p
r4 = 13 11 = 2, s1 = 4 dan s2 = 3 membentuk barisan

5, 1, 6, 2, 4, 3,
yang tidak lain adalah barisan dari 6 bilangan-bilangan awal tak nol

75

1, 2, 3, 4, 5, 6
dalam sistem residu tereduksi {1, 2, ..., 11, 12} modulo 13, keduanya hanya berbeda
urutan. Berdasarkan Lemma Gauss di atas,
(1)4 412/2 (mod 13)

1 46 (mod 13).

atau

Definisi 3.3

Untuk sembarang bilangan real x, lambang x menyatakan bilangan bulat terbesar yang
lebih kecil atau sama dengan x.
Contoh 3.2

5 = 5,

3.5 = 3,

10.225 = 10

dan
1.25 = 2.
Teorema 3.3

( )

Jika p > 2 prima dan (a, 2p) = 1, maka ap = (1)t dengan t =

( ) ( )

jika a = 2, maka ap =

( p 1) / 2

=1

pa , dan

2 = (1) ( p2 1) / 8 .
p

Bukti.

Perhatikan, (a, 2p) = 1 berakibat (a, p) = 1. Di sini kita akan menggunakan notasi
yang sama dengan notasi dalam Teorema 3.2, yaitu mengunakan lambang bilanganbilangan bilangan-bilangan a, 2a, , (p 1)a/2 yang kongruen modulo p dengan
bilangan-bilangan residu r1, r2, , rn, s1, s2, , sk, walaupun urutan barisannya berubah..
Sesungguhnya, bilangan residu ri dan sj merupakan sisa hasil bagi oleh p terhadap
bilangan a, = 1, 2, , (p 1)/2. Apakah a ganjil atau genap, bilangan bulat yang
merupakan hasil bagi a oleh p adalah q = a/p. Dengan kata lain,

a = pq + ri = p pa + ri
atau mungkin

a = pq + sj = p pa + sj,

tergantung pada keadaan apakah a ri (mod p) atau a sj (mod p). Apabila setiap
suku ini dijumlahkan sepanjang indeks = 1, 2, , (p 1)/2, diperoleh

76

( p 1) / 2

a =

=1

( p 1) / 2

=1

i =1

j =1

p pa + ri + s j .

Dari lain pihak, mengingat barisan bilangan-bilangan residu 1, 2, , (p 1)/2 sama


dengan barisan (p r1), (p r2), , (p rn), s1, s2, , sk, maka
( p 1) / 2

=1

i =1

j =1

i =1

j =1

( p ri ) + s j = np ri + s j

Selisih kedua persamaan terakhir adalah


(a 1)

( p 1) / 2

= p(

=1

( p 1) / 2

Tetapi

=1

( p 1) / 2

=1

pa n) +2 ri .

i =1

p2 1
sehingga ruas kiri berubah,
8
(a 1)

( p 1) / 2
n
p2 1
= p ( pa n) +2 ri .
8
=1
i =1

Dalam modulo 2, dan karena p ganjil, diperoleh


p 2 1 ( p 1) / 2 a
(a 1)
p n (mod 2).
8
=1
Jika a ganjil, a 1 0 (mod 2) sehingga

( p 1) / 2

=1

pa (mod 2).

Jika a = 2, maka pa = 2p 0 (mod 2) sehingga


p2 1
n
(mod 2).
8
Perhatikan, sebagian Teorema 3.3 adalah pernyataan ulang Teorema 3.2 dengan
bahasa yang berbeda. Dalam Teorema 3.2, digunakan dua barisan, barisan bilangan bulat

a, 2a, 3a, , (p 1)/2


dan barisan bilangan-bilangan residu kongruennya

r1, r2, r3, , rn, s1, s2, , sk


sedangkan dalam Teorema 3.3, barisan kedua dinyatakan oleh sisa hasil bagi a, = 1, 2,
3, , (p 1)/2 oleh p. Sebagai akibatnya, kedua teorema menyatakan pangkat n dalam
(1)n dengan cara yang berbeda.

77

Soal-soal.

1. Jika hasil bagi b oleh a memberikan sisa tak negatif r dengan 0 r < a dan hasil
baginya q; yaitu b = qa + r, buktikan bahwa q = b/a..
2. Buktikan bahwa 3 adalah residu kuadratik modulo 13, tetapi merupakan nonresidu
kuadratik modulo 7.

( )

3. Cari nilai-nilai ap dari ke-12 kasus yang diperoleh apabila a = 1, 2, 2, 3 dan p =


11, 13, 17.
4. Buktikan bahwa residu kuadratik modulo 11 adalah 1, 3, 4, 5, 9 dan cari semua solusi
dari sepuluh kongruensi-kongruensi x2 a (mod 11) dan x2 a (mod 112) yang
diperoleh dari nilai-nilai a = 1, 3, 4, 5, 9.
5. Di antara kongruensi berikut, manakah yang memiliki solusi dan berapa banyak
solusinya?

a. x2 2 (mod 61)

b. x2 2 (mod 59)

c. x2 2 (mod 61)

d. x2 2 (mod 59)

e. x2 2 (mod 122)

f. x2 2 (mod 118)

g. x2 2 (mod 61)

h. x2 2 (mod 118).

6. Misalkan p prima dan (a, p) = (b, p) = 1. Buktikan jika x2 a (mod p) dan x2 b


(mod p) tak ada solusinya, maka x2 ab (mod p) mempunyai solusi.
7. Misalkan g adalah akar primitif dari bilangan prima p > 2. Buktikan bahwa semua
bilangan residu kuadratik modulo p kongruen dengan g2, g4, g6, , gp1 sedangkan
semua bilangan nonresidu kuadratik modulo p kongruen dengan g, g3, g5, , gp2.
3.2 Resiprositi Kuadratik

Teorema 3.3 berlaku untuk setiap a Z dengan (a, 2p) = 1, di mana p > 2 prima,
yaitu untuk setiap bilangan bulat ganjil a yang tak mengandung faktor p. Pada khususnya,
teorema berlaku untuk setiap bilangan prima q p.
Teorema 3.4 (Resiprositi Kuadratik)

Jika p > 2 dan q > 2 adalah dua bilangan prima, maka

( )( )
p
q

q
p = ( 1)

p1 q1
2 2 .

78

Bukti.
Misalkan

S1 = { (x, y) | 1 x (p 1)/2 } dan 1 y < qx/p }

dan

S2 = { (x, y) | 1 y (q 1)/2 } dan 1 x < py/q }.


Perhatikan himpunan sebanyak ((p 1)/2)((q 1)/2) pasangan-pasangan bilangan bulat

T = { (u, v) | 1 u (p 1)/2, 1 v (q 1)/2 }.


Misalkan (u, v) T. Satu kemungkinan v < qu/p akan mengakibatkan (u, v) S1.
Kemungkinan lain v > qu/p berakibat u < pv/q sehingga (u, v) S2. Kemungkinan v =

qu/p mustahil karena ini berarti p|u padahal 0 u (p 1)/2. Ini membuktikan bahwa
jika (u, v) T maka (u, v) S1 atau (u, v) S2, yaitu

T S1 S2.
Sebaliknya apabila (x, y) S1, maka 1 x (p 1)/2 dan
p 1

1 y < (q/p)x (q/p)(p 1)/2 = ( p ) ( 2 ) < 2 .

Karena y bulat sedangkan q ganjil, ketaksamaan 1 y < q/2 yang diperoleh di atas akan
ekuivalen dengan ketaksamaan
1 y (q 1)/2.
Terbukti apabila (x, y) S1, maka (x, y) T. Dengan cara yang analog bisa dibuktikan
bahwa jika (x, y) S2, maka (x, y) T. Ini membuktikan bahwa S1 S2 T sehingga
terbukti T = S1 S2.
Sekarang kita menghitung banyaknya pasangan-pasangan di S1 dan di S2 secara
terpisah. Untuk setiap x dengan 1 x < (p 1)/2, banyaknya pasangan (x, y) di S1 sama
dengan banyaknya indeks untuk y = 1, 2, , qx/p. Jadi banyaknya pasangan-pasangan
(x, y) di S1 adalah
( p 1) / 2

x =1

qx

p .

Dengan cara yang analog, diperoleh bahwa banyaknya pasangan-pasangan (x, y) di S2


adalah
( p 1) / 2

y =1

py

q .

79

Karena S1 dan S2 saling lepas dan total banyaknya pasangan-pasangan dalam T = S1 S2,
adalah ((p 1)/2)((q 1)/2, maka
( p 1) / 2

qx

p +

x =1

( p 1) / 2

y =1

py

q = ((p 1)/2)((q 1)/2.

Jadi,
( q 1) / 2

( )( )
p
q

q
p = (1)

( )
q

Mengingat p
dengan

( )

q
p , diperoleh

y=1

py
q

(1)

x=1

qx
p

( p 1) / 2

= (1)

x=1

qx +
p

( q 1) / 2

y=1

py
q

( )( )
p

= 1, maka apabila kedua ruas q


p1 q1
2

( )

p
2
q = (1)

( ) ( )
q

( p 1) / 2

Demikian pula, p = q (1)

( ) ( )
q
p =

q
p

= (1)

= (1)

p1 q1
2 2 .

p1 q1
2 2

dikalikan

p1 q1
2 .

q
2
p (1)

p1 q1
2 2 .

Contoh 3.3.

( )

42 .
Tentukan 61

Jawab.

( 6142 ) = ( 611 ) ( 612 ) ( 613 ) ( 617 ) .


Selanjutnya, dicari nilai dari faktor-faktor di ruas kanan kesamaan ini satu persatu.

( 611 ) = (1)
( 612 ) = (1)

( 613 ) = ( 613 ) (1) 2

( ) ( )

60/2

=1

(612 1) / 8

= (1)3720/8 = (1)465 = 1.

31 611
2

( )( )
= 13 (1) 2 2 = 13 = 1.

71 611
2 =

7 = 61 ( 1) 2
7
61

()

2 60

()

5
2 2 = 5 = 7 ( 1)( 2 )( 2 ) = 2 = ( 1) 8 =
5
7 ( 1)
7
5

( )

6 60

( ) ( )

6 4

( )

24

1.
Jadi,

( 6142 ) = (1)(1)(1)(1) = 1.
Cara lain:
80

Termotivasi oleh upaya penerapan sebanyak mungkin teorema-teorema yang ada,


solusi di atas bisa lebih panjang daripada solusi yang lain. Solusi berikut lebih pendek.
42 = 19 = 61 . ( 1)( 2 )( 2 ) = 4 = 1.
61
19
61
19

( ) ( ) ( )

60 18

( )

Contoh 3.4

Tentukan semua bilangan prima p agar a = 3 adalah bilangan residu kuadratik modulo p.

Jawab.
Menurut Teorema 3.4,

( 3p ) = ( 3p ) (1) .
( 1 ) = 1, jika p 1(mod 3)
p
( 3 ) = 23
( 3 ) = 1, jika p 2 (mod 3)

( p1) / 2

Karena

sedangkan
maka

1, jika p 1(mod 4)
(1)( p1) / 2 =
1, jika p 3(mod 4)

( 3p ) = 1

jika dan hanya jika


p 1 (mod 3)

dan

p 1 (mod 4)

p 2 (mod 3)

dan

p 3 (mod 4).

atau jika
Ini berarti, jika dan hanya jika
p 1 (mod 12)

atau

p 11 (mod 12),

yaitu p = 11, 13, 23, 37, , dst adalah bilangan-bilangan prima yang memenuhi.
Hasil semacam di atas seringkali berguna untuk menentukan primalitas suatu
bilangan. Pada khususnya, hasil di atas bisa diterapkan dalam contoh berikut.
Contoh 3.5

Renungkan bilangan 9997. Perhatikan, 9997 = 1002 3 sehingga 3 1002 (mod p)


jika p|9997. Akibatnya, ada dua kemungkinan, p = 3 atau 3 x2 (mod p) mempunyai
solusi. Kemungkinan p = 3 mustahil, sebab jelas 39997. Kemungkinan kedua adalah
ekuivalen dengan pernyataan

( 3p ) = 1. Dari Contoh 3.4, ( 3p ) = 1 dipenuhi oleh


81

semua bilangan prima p yang memenuhi p 1 (mod 12) atau p 11 (mod 12). Dalam
hal ini, kita cukup mencari satu nilai p|9997 yang memenuhi p <

9997 . Alasannya?

Bilangan prima 9997 > p > 9997 dengan p|9997 ada jika dan hanya jika terdapat
bilangan bulat q dengan 9997 = pq dan 2 q < 9997 , jika dan hanya jika ada faktor
prima p|q. Jelas p|9997 dan p q < 9997 . Jadi, bilangan prima p dengan
p > 9997 dan p|9997 ada jika dan hanya jika ada bilangan prima p < 9997 dengan
p|9997. Jadi untuk setiap bilangan prima p dengan 9997 > p > 9997 , selalu terdapat
bilangan prima p dengan 2 p < 9997 .
Dengan asumsi p <

9997 , kongruensi p 1 (mod 12) dipenuhi oleh bilangan-

bilangan 1, 13, 25, 37, , 97 sedangkan kongurensi p 11 (mod 12) dipenuhi oleh
12, 23, 35, , 95. Karena p harus prima, dari kedua daftar bilangan-bilangan ini,
hanya sebelas bilangan prima 11, 13, 23, 37, , yang perlu diuji sebagai kandidat
dari p. Ternyata 13 membagi 9997 dan 9997 = 13769. Jadi 9997 bukan bilangan
prima.
Pertanyaan berikutnya, apakah 769 merupakan bilangan prima?
Seandainya 769 bilangan prima, maka bisa dicari bilangan p yang membagi 769,
sehingga otomatis juga membagi 9997. Tetapi hanya ada tiga bilangan: 11, 13 dan 23,
di antara sebelas kandidat bilangan prima p|9997 yang juga memenuhi p <

769 .

Tak satu pun di antara ketiganya membagi 769. Kesimpulannya, 769 adalah bilangan
prima.
Soal-soal.

1. Buktikan, jika p dan q adalah dua bilangan prima berbentuk 4k + 3 dan jika x2 p
(mod q) tak menghasilkan solusi, maka x2 q (mod p) menghasilkan tepat dua solusi.
2. Buktikan, jika p adalah residu kuadratik dari suatu bilangan prima q > 2 sedangkan p
berbentuk 4k + 1, maka q adalah residu kuadratik modulo p.
3. Manakah di antara kongruensi-kongruensi berikut yang menghasilkan solusi?
a. x2 5 (mod 227)

b. x2 5 (mod 229)

c. x2 5 (mod 227)

d. x2 5 (mod 229)

82

e. x2 7 (mod 1009)

f. x2 7 (mod 1009)

(Bilangan-bilangan 227, 229 dan 1009 adalah prima).

( p)

4. Cari semua nilai q untuk sembilan kemungkinan pasangan p dan q yang diperoleh
dari pilihan p = 7, 11 atau 13 dan q = 227, 229 atau 1009.
5. Cari semua bilangan prima p sedemikian rupa sehingga kongruensi x2 11 (mod p)
menghasilkan solusi.

( )
Cari semua bilangan prima p yang memenuhi ( 5p ) = 1.

6. Cari semua bilangan prima p yang memenuhi 10


p = 1.
7.

8. Untuk modulo bilangan prima berapakah agar 2 menjadi residu kuadratik?


3.3 Simbol Jacobi
Definisi 3.4

Misalkan (P, Q) = 1, Q adalah bilangan positif dan ganjil di mana Q = q1 q2 qs dengan


qj, j = 1, 2, , s, adalah bilangan-bilangan prima yang lebih besar dari 2, tidak harus
berbeda satu sama lain. Didefinisikan simbol Jacobi berikut

( )
P
Q

( ) adalah simbol Legendre.

di mana qPj

( qP ) ,
s

j =1

Jika Q prima dan ganjil, maka nilai simbol Jacobi sama dengan nilai simbol
Legendre. Suatu bilangan a disebut residu kuadratik modulo Q jika dan hanya jika (a, Q)

( )

a =
= 1 dan a adalah residu kuadratik modulo dari setiap prima yang membagi Q. Jika Q

( )

( )

P = 1. Tetapi jika P = 1, tidak bisa


1, a bukan residu kuadratik modulo Q. Jelas Q
Q

disimp-ulkan bahwa P adalah residu kuadratik modulo Q. Sebagai contoh penyangkal,

( 92 ) = 1 tetapi x

2 (mod 9) tak memberikan solusi.

Perhatikan,
(PP, QQ) = 1
berakibat
(P, Q) = 1, (P, Q) = 1, (P, Q) = 1, (P, Q) = 1,
(PP, Q) = 1, (PP, Q) = 1, (P, QQ) = 1 dan (P, QQ) = 1.
83

Teorema 3.5

Misalkan Q dan Q adalah dua bilangan positif yang ganjil. Jika (PP, QQ) = 1, maka

( QP ) ( PQ' ) = ( PPQ' )
P'
d. ( P ' P ) = ( Q
')
Q 'Q
' = P .
P P (mod Q) berakibat ( P
Q ) (Q)

( QP ) ( QP' ) = ( QQP ' )


c. ( PQ ) = ( P ) = 1.
Q

a.

b.

2
2

e.

Bukti.

( )

P . Bagian b diperoleh dari definisi dan


Pembuktian a langsung diperoleh dari definisi Q

penggunaan Teorema 3.1.b. Untuk membuktikan c, substitusi P = P pada bagian a dan Q


= Q pada bagian b. Hasilnya, diperoleh

( )
( QP''QP ) = ( PQ'P' ) ( PQ'P )
= ( PQ' P' )
P' P
= (Q
' ) ( Q' )
P' .
= (Q
')

( QP ) = ( QP ) ( QP ) = 1 dan ( PQ ) = ( QP ) ( QP ) = 1,
2

P = 1. Untuk membuktikan d, diuraikan sebagai berikut:


sebab Q
2

2
2

(dari a)

(Q )

2
(sebab menurut c, P ' P2 = 1).

(dari b)

( )

2
(sebab menurut c, P
= 1).
Q'

Faktorisasi prima Q = q1q2qs memberikan P P (mod qj), dan menurut Teorema 3.1.c,

( Pq' ) = ( qP ) , j = 1, 2,, s. Dari Definisi 3.4, disimpulkan ( PQ' ) = ( QP ) .


j

Lemma 3.1

Jika a dan b ganjil, maka


a. a 1 + b 1 = ab 1 (mod 2); dan
2
2
2
2
2
2 2
b. a 1 + b 1 = a b 1 (mod 2).
8
8
8

Bukti.
84

ab 1 ( a 1 + b 1 ) = ab 1 a + b 2 = ab a b + 1 = (a 1)(b 1) 0 (mod 2).


2
2
2
2
2
2
2

Demikian pula,
a2b2 1 ( a 2 1 + b2 1 ) = a2b2 1 a2 + b2 2 = a2b2 a2 b2 + 1 = (a2 1)(b2 1)
8
8
8
8
8
8
8

0 (mod 2),
sebab (a + 1), (a 1), (b + 1) dan (b 1) adalah bilangan-bilangan ganjil, katakan (a + 1)
= 2(x + 1), (a 1) = 2x, (b + 1) = 2(y + 1) dan (b 1) = 2y, untuk suatu bilangan bulat x
dan y, kemudian
(a2 1)(b2 1)/8 = (a + 1)(a 1)(b + 1)(b 1)/8 = 2(x + 1)(2x)(2(y + 1))(2y)/8
= 2xy(x + 1)(y + 1) 0 (mod 2).
Teorema 3.6.

( )

( )

2 = ( 1) ( Q2 1) / 8 .
Misalkan Q ganjil dan Q > 0, maka Q1 = (1)(Q1)/2 dan Q

Bukti.

( )

1 =
Q

(
s

j =1

(1)

1 =
qj

( q j 1) / 2

j=1( q j 1) / 2
s

= (1)

j =1

Dengan menggunakan lemma bagian a beberapa kali, besar pangkat di atas adalah
s
q j 1
s ( q 1)

Q 1
j
j =1
=
=
.

2
2
2
j =1

( )

Terbukti, Q1 = (1)

Q1
2 .

Demikian pula,

( )
2
Q

(
s

j =1

2
qj

(1)

( q 2j 1)/8

j=1( q2j 1)/8


s

= (1)

j =1

sehingga dengan menggunakan lemma di atas beberapa kali, besar pangkat di atas adalah
s
q2j 1
s
(q2j 1)
Q2 1
j =1
=
=
.

8
8
8
j =1
Ini membuktikan bahwa

( )=
2
Q

Q2 1
(1) 8 .

85

Teorema 3.7 (Resiprositi Umum)

Jika P dan Q adalah sepasang bilangan bulat yang ganjil dan jika (P, Q) = 1, maka

( QP )( QP ) = (1)

( P21 )( Q21 )

Bukti.
r

Dengan menganggap P =

( )
P
Q

pi =1 dan Q =
i =1

( qP )

q
j =1

j =1

j =1
s

(Teorema 3.5.a digunakan berapa kali/Definisi 3.4)

( q )

(Teorema 3.5.b digunakan beberapa kali)

(Resiprositi khusus pasangan bilangan prima)

j =1 i =1
s
r

pi
j

)
= ( ) (1)

j =1 i =1
s
r
j =1 i =1

p 1 q j 1
qj
( i )(
)
2
2
(

1)
pi
(

pi 1 q j 1
)(
)
2
2

j=1 i=1(

pi 1 q j 1
)(
)
2
2

qj
pi

( )

Q
= ( P ) (1)

Q
= ( P ) (1)
Q
= ( P ) (1)
s

= P (1)

pi 1
j =1 2

i =1

i=1

pi 1) (

(rumus

q j 1

i =1

i =1

i =1

aibi = ( ai )( bi ) dan Definisi 3.4)

q j 1)

j =1

(Lemma 3.1, digunakan beberapa kali)

( P 1) ( Q 1)
2
2

(Definisi 3.4).

Teorema di atas menunjukkan bahwa simbol Jacobi mengikuti aturan resiprositi.

( ) dengan Q sembarang bilangan bulat ganjil adalah perluasan dari

P
Simbol Jacobi Q

simbol Legendre

( ap ) yang diberlakukan hanya untuk satu bilangan prima ganjil p.

( )

P = 1 jika
Mungkin ada yang berpendapat bahwa lebih natural untuk mendefinisikan Q

P adalah residu kuadratik modulo Q. Tetapi apabila didefinisikan demikian, aturan


resiprositi tak akan diperoleh, misalnya karena x2 5 (mod 9) tak memiliki solusi, maka

( )

menurut definisi ini 95 = 1, sedangkan karena x2 4 9 (mod 5) memiliki solusi x = 2,

( ) ( )

maka 95 = 54 = 1. Jadi

( )( )
5
9

9 = 1 1 = (1)(
5

51 9 1
2 )( 2 )

86

Itulah sebabnya, kita tak memilih definisi di atas, tetapi memilih Definisi 3.4 untuk lebih

( ) ( ) ( 53 ) = 1.

mengutamakan berlakunya aturan resiprositi. Menurut Definisi 3.4, 95 = 53

Dengan memilih Definisi 3.4, keterkaitan simbol Jacobi dengan residu kuadratik
menjadi hilang. Tetapi ini tidak berarti bahwa simbol Jacobi tak bisa digunakan untuk
komputasi-komputasi seperti yang disajikan di bagian 3.2. Justru simbol Jacobi memiliki
peran penting dalam komputasi-komputasi semacam. Sesungguhnya aturan resiprositi
khusus dalam Teorema 3.4 tidak berlaku jika salah satu dari P atau Q bukan prima ganjil.
Tetapi Teorema 3.7 bisa digunakan dalam penyelesaian komputasi semacam, misalnya
317
= ( 105
( 105
)
317 )
2
= ( 105
)

(dengan aturan resiprositi umum, Teorema 3.7)

= 1.
Soal-soal.

( ) ( ) ( ) ( )

23 , 51 , 70 , 35 .
1. Hitung 83
71
73
97

2. Tentukan, apakah bentuk kuadratik berikut memiliki solusi.

a. x2 10 (mod 127)

b. x2 73 (mod 173)

c. x2 137 (mod 401)

d. x2 11 (mod 61)

e. x2 43 (mod 79)

f. x2 31 0 (mod 103).

3. Buktikan, jika p dan q adalah dua bilangan prima yang berbentuk 4k + 1, maka

( qp ) = ( qp ) .
( p ) = 0.
p 1

4. Buktikan bahwa

j =1

5. Buktikan bahwa jika p > 2 prima dan (a, p) = 1, maka ax2 + bx + c 0 (mod p)
dengan d = b2 4ac memiliki dua, satu atau tak ada solusi sesuai dengan masingmasing kondisi: d adalah residu kuadratik, d kongruen 0 dan d bukan residu kuadratik
modulo p.

87

6. a. Misalkan p adalah prima dengan bilangan residu kuadratik a. Buktikan bahwa x2

a (mod p2) menghasilkan tepat dua solusi, dengan menulis x = x1 + py, di mana

x1 adalah solusi dari x2 a (mod p).


b. Gunakan induksi matematika untuk membuktikan secara umum bahwa x2 a
(mod pk) tepat memiliki tepat dua solusi.
7. Misalkan, pembagi prima dari bilangan positif ganjil m adalah p1, p2, , pn, dan
misalkan (a, m) = 1. Buktikan bahwa x2 a (mod m) ada solusinya jika dan hanya

( )

jika pa = 1 untuk setiap i = 1, 2, , n.


i
8. Bilangan-bilangan prima berpangkat pa bagaimanakah yang memenuhi syarat berikut:
ada dua bilangan bulat x dan y yang memenuhi (x, p) = 1, (y, p) = 1 dan x2 + y2 0
(mod pa)?
9. Bilangan-bilangan bulat n manakah yang memenuhi syarat berikut: ada dua bilangan
bulat x dan y yang memenuhi (x, p) = 1, (y, p) = 1 dan x2 + y2 0 (mod n)?

88

4. Beberapa Persamaan Diophantine


4.1 Persamaan Diophantine

Ada banyak masalah yang memerlukan lebih dari sekedar mencari semua solusi
dari suatu persamaan. Masalah ini dinyatakan sedemikian rupa sehingga solusi yang
= 26/ = 52 adalah
diinginkan harus memenuhi syarat-syarat lain. Sebagai contoh, 26
65
65
hasil yang benar walaupun secara aljabar, cara ini salah. Masalah yang ditanyakan adalah
mencari semua bilangan pecah positif yang berprilaku sama (jika dicoret angka satuan
pembilang dan angka puluhan penyebut yang sama, hasilnya benar). Dengan kata lain,
10 x + y

kita mencari semua bilangan x, y dan z yang memenuhi 10 y + z = xz . Persamaan ini bisa
diubah ke bentuk (y x)z = 10(y z)x, tetapi kita hanya tertarik dengan solusi-solusi x, y
dan z yang positif dan kurang dari 10. Kita tak akan menjabarkan cara mendapatkan
49 .
, 16 , 26 dan 98
solusi-solusi berikut: 19
95 64 65

Dalam contoh di atas, (y x)z = 10(y z)x adalah persamaan tak tentu yang
memiliki banyak sekali solusi-solusi. Kita memilih solusi-solusi yang memenuhi syarat
tambahan: positif dan kurang dari 10. Masalah demikian disebut masalah Diophantine
dan kita telah membahas sebuah persamaan Diophantine. Di luar masalah yang hanya
sekedar memenuhi rasa ingin tahu, ada banyak persamaan Diophantine yang penting
untuk dipelajari. Pada umumnya, syarat tambahan yang diberikan pada solusi adalah
harus berupa bilangan bulat atau kadang-kadang, harus rasional. Seringkali solusi yang
disyaratkan harus positif.
Ada banyak sekali persamaan Diophantine dan tak ada metoda umum untuk
menyelesaikannya. Kita hanya membahas beberapa di antaranya yang paling sederhana.
Kita juga membahas masalah-masalah lain yang terkait dengan persamaan Diophantine.
Sebagai contoh, Teorema 4.5 menyatakan bahwa pada hakekatnya persamaan

x12 + x22 + x32 + x42 = n


memiliki paling sedikit satu solusi. Tetapi kita tidak akan berusaha mencari semua solusi.

4.2 Persamaan ax + by = c

Setiap persamaan linier dalam dua peubah dengan koefisien-koefisien bulat bisa
ditulis dalam bentuk ax + by = c, dengan a, b, c Z. Solusi masalah ini trivial, kecuali
dalam kasus a dan b bukan bilangan nol. Jadi diasumsikan a 0 dan b 0. Teorema 1.3
menyatakan keberadaan x0 dan y0 sedemikian rupa sehingga g = (a, b) = ax0 + by0. Nilai

x0 dan y0 bisa diperoleh dengan menerapkan algoritma Euklid yang diperluas (Akibat
Teorema 1.11, seperti yang dikerjakan dalam Contoh 1.2) pada |a| dan |b|.
Jika gc, persamaan ax + by = c tak memiliki solusi bulat. Jika g|c, kita gunakan
kedua solusi x0 dan y0 dari persamaan ax + by = g untuk mendapatkan solusi x1 = (c/g)x0
dan y1 = (c/g)y0 dari persamaan ax + by = c.
Untuk mencari semua solusi bulat, misalkan r dan s adalah sembarang solusi bulat.
Jadi ar + bs = c = ax1 + by1, atau
a (r x ) = b (s y ).
1
1
g
g

(4.1)

Tetapi menurut Teorema 1.7, ( ag , bg ) = 1 sehingga menurut Teorema 1.10, ag |(s y1) dan
b |(r x ). Dengan kata lain, terdapat t, u Z sedemikian rupa sehingga s y = a u dan
1
1
g
g

r x1 = bg t. Apabila kedua hasil ini disubstitusi ke ekspresi (4.1), disimpulkan t = u.


Jadi, setiap solusi bulat r dan s dari persamaan Diophantine ax + by = c berbentuk

r = x1 + bg t
dan
dengan t Z.

s = y1 ag t

Soal-soal.

1.

Buktikan bahwa semua solusi-solusi bulat dari 3x + 5y = 1 bisa ditulis dalam bentuk

x = 2 + 5t, y = 1 3t, atau dalam bentuk x = 2 5t, y = 1 + 3; atau dalam bentuk


x = 3 + 5t, y = 2 3t.
2.

Cari semua solusi bulat dari 10x 7y = 17.

3.

Jika ax + by = c bisa diselesaikan, maka ada solusi x0, y0 dengan 0 x0 < |b|.

4.

Buktikan. ax + by = c bisa diselesaikan jhj (a, b) = (a, b, c).


90

4.3 Solusi-Solusi Bulat Positif

Misalkan a, b, c Z+ dan akan dicari semua solusi bulat positif s dan r dari persamaan ax
+ by = c. Cara penyelesaiaan sejalan dengan Bagian4.2, tetapi nilai-nilai t dibatasi
sedemikian rupa sehingga r dan s selalu positif. Dalam hal ini, bilangan t berada dalam
g

interval b x1 < t < a y1. Nilai terkecil yang mungkin dari t adalah b x1 + 1 4) dan
g

nilai terbesar yang mungkin dari t adalah a y1 + 1. Jadi, banyak solusi adalah
g

N = a y1 + 1. b x1 + 1 + 1.
Dengan menerapkan ketaksamaan

x + y x + y x + y + 1,
diperoleh
g

( a y1 b x1 + 1) N a y1 b x1 .
g

gc

Karena a y1 b x1 = ab (by1 + ax1) = ab , akhirnya diperoleh


gc

(c + 1) N ab .
Jadi, selalu terdapat solusi positif jika gc > ab.
Soal-soal.

1.

Cari semua solusi bulat positif dari

a.

5x + 3y = 52

b. 15x + 7y = 111

c.

40x + 63y = 521

d. 123x + 57y = 531

e.

12x + 501y = 1

f. 12x + 501y = 274

2.

Buktikan 101x + 37y = 3819 memiliki solusi bulat positif.

3.

Jika a, b Z berlawanan tanda dan (a, b) = 1, buktikan bahwa untuk setiap bilangan
bulat c, persamaan ax + by = c selalu memiliki solusi.

4.

Misalkan a, b Z+ dengan (a, b) = 1 dan S = {ax + by | x, y Z+ {0}}. Buktikan


S memuiat semua bilangan bulat yang lebih besar dari c = ab a b, tetapi c S.

5.

Seperti butir soal sebelumnya, tetapi S = {ax + by | x, y Z+ }. Bilangan bulat


terbesar apakah yang berada dalam S, tidak berada lagi dalam S?

4)

Untuk setiap bilangan real x, x menyatakan bilangan bulat terbesar yang lebih kecil atau sama dengan
x. Dengan kata lain, x adalah fungsi pembulatan ke bawah (Bahasa Inggris: floor function).

91

4.4 Persamaan Linear Yang Lain

Persamaan

a1x1 + a2x2 + + akxk = c,

(k > 2)

(4.2)

bisa dibawa ke kasus yang dibahas di Bagian 4.2, kasus dengan hanya dua peubah. Kita
bisa berasumsi bahwa semua ai tidak nol dan (a1, a2, ..., ak)|c. Tulis

xk1 = u + v,

xk = u + v,

(4.3)

di mana , , dan akan ditentukan sedemikian rupa sehingga = 1. Dengan


demikian akan diperoleh u = xk1 xk dan v = xk1 + xk sehingga u dan v bulat jhj

xk1 dan xk bulat. Jika dipilih


a

= ( a k, a ) ,
k 1 k

= ( a k,a1 ) ,
k 1 k

maka (, ) = 1 sehingga dengan menggunakan perluasan algoritma Euklid (Akibat


Teorema 1.11) seperti yang dibahas di Bagian 4.2, dan bisa dicari sebagai solusi dari

x y = 1.
Persamaan (4.2) sekarang menjadi

a1x1 + a2x2 + + ak2xk2 + (ak1 + ak)u = c,

(4.4)

dengan banyak peubah berkurang satu. Perhatikan,

ak1 + ak = (ak1, ak) + (ak1, ak) = (ak1, ak),


(a1, a2, ..., (ak1, ak)) = (a1, a2, ..., ak).
Jadi persamaan (4.4) memiliki sifat-sifat yang sama dengan persamaan (4.2), yaitu
pembagi persekutuan terbesar dari koefisien-koefisiennya membagi c, tak ada koefisien
yang nol. Jika k > 3, proses ini bisa diulang terapkan ke persamaan (4.4) sehingga didapat
persamaan baru dengan k 2 peubah. Demikian seterusnya, proses diulang sampai pada
akhirnya diperoleh persamaan baru dengan hanya dua peubah.
Lebih jauh, pembahasan di Bagian 4.2 menyatakan bahwa jika persamaan linear
dengan dua peubah memiliki solusi, maka solusi umumnya bisa dinyatakan melalui satu
parameter t. Demikian pula halnya persamaan (4.2) dengan k parameter, solusi umumnya
dinyatakan melalui k 1 parameter. Hal ini bisa dibuktikan dengan menggunakan induksi
matematika pada k.
Anggap semua bentuk persamaan (4.4) dengan k 1 peubah memilki solusi x1, x2,
, xk2, u yang dinyatakan melalui k 2 parameter-parameter v1, v2, ..., vk2. Dengan
92

menggunakan (4.3), solusi-solusi x1, x2, , xk2, u + v dan u + v dari persamaan


(4.2) bisa dinyatakan oleh k 1 parameter v1, v2, ..., vk2 dan v. Mudah dibuktikan bahwa
solusi-solusi ini berbentuk xi = bi + di,1v1 + di,2v2 + ... + di,k1v k1, di mana v = vk1.
Jika kita diminta menyelesaikan sistem yang terdiri atas s persamaan dengan r
peubah

aj,1 x1 + a j,2x2 + + a j,rxr = c j,

(j = 1, 2, , s)

(4.5)

kita awali dengan menyelesaikan persamaan pertama, j = 1. Jika solusinya ada, maka
solusi ini berbentuk

xi = bi + di,1v1 + di,2v2 + ... + di,r1v r1.


Substitusi solusi ini ke dalam persamaan-persamaan berikutnya dan kemudian selesaikan
persamaan kedua, j = 2, untuk v1, v2, ..., vr1 melalui r 2 parameter baru. Pengulangan
proses ini pada akhirnya memberikan solusi sistem persamaan (4.5). Jika pada proses di
atas, terdapat satu persamaan yang tak ada solusinya, maka disimpulkan seluruh sistem
persamaan (4.5) tak ada solusinya.
Soal-soal.

Cari semua solusi dari


1.

x + 2y + 3z = 1

2. x + 2y + 3z = 10

3.

5x 2y 4z = 1

4. 5x 2y 4z = 10

5.

3x 6y + 5z = 11

6. 6x + 48y 78z = 5

4.5 Persamaan x2 + y2 = z2

Kita akan mencari solusi bulat positif dari persamaan x2 + y2 = z2. Misalkan x, y, z
adalah sebuah solusi dan tulis g = (x, y) sehingga g2 | z2 dan g | z. Dari (x, y, z) = ((x, y), z),
disimpulkan (x, y, z) = g. Berdasarkan simetri, g = (x, y, z) = (x, y) = (x, z) = (y, z) dan

( gx ) + ( gy ) = ( gz )
2

dengan

( gx , gy ) = 1,

( gx , gz ) = 1,

( gy , gz ) = 1.

Suatu solusi x1, y1, z1 yang sepasang-sepasang saling relatif prima disebut solusi
primitif. Jadi setiap solusi x, y, z berbentuk x = gx1, y = gy1 dan z = gz1 untuk suatu solusi

93

primitif x1, y1, z1. Sebaliknya jika x1, y1, z1 adalah solusi primitif, maka untuk setiap
bilangan bulat positif g, ketiga bilangan gx1, gy1 dan gz1 merupakan solusi primitif. Jadi
kita hanya perlu mencari solusi primitif, hal yang segera kita kerjakan.
Pertama kali kita amati bahwa x dan y tidak bisa bersama-sama genap atau
bersama-sama ganjil. Seandainya bersama-sama ganjil misalnya, maka x2 1 (mod 4)
dan y2 1 (mod 4) sehingga z2 2 (mod 4) dan tak ada z yang memenuhi, kontradiksi.
Karena peran x dan y dalam x2 + y2 = z2 bersifat simetri, kita bisa membatasi diri
pada kasus y genap, x dan z ganjl. Tulis persamaan x2 + y2 = z2 ke bentuk yang ekuivalen

( )

2
z+ x zx = y .
2 2
2

Perhatikan.

sehingga

(4.6).

( z+2x , z2x ) | z+2x + z2x = z,


( z+2x , z2x ) | z+2x z2x = x,

( z+2x , z2x ) = 1.

(4.7)

Persamaan (4.6) menunjukkan bahwa terdapat bilangan bulat r dan s sehingga

z + x = r2
2

dan

z x = s2.
2

Jelas r > s dan keduanya dipilih positif. Lebih jauh, x = r2 s2, y = 2rs dan z = r2 + s2 dan
dari (4.7) disimpulkan bahwa (r, s) = 1. Karena z ganjil, paritas dari r dan s berlawanan,
salah satu genap dan yang lain ganjil.
Sebaliknya jika r dan s adalah dua bilangan yang memenuhi: (r, s) = 1, r > s > 0, r
dan s paritasnya berlawanan, dan jika x = r2 s2, y = 2rs dan z = r2 + s2, maka x, y dan z
adalah tiga bilangan bulat positif yang memenuhi

x2 + y2 = (r2 s2)2 + (2rs)2 = (r2 + s2)2 = z2.


Selanjutnya mudah dibuktikan (x, y) = 1, y genap. Jadi x, y, z adalah suatu solusi primitif
dengan y genap. Diskusi ini membuktikan teorema berikut.
Teorema 4.1

Solusi primitif dari x2 + y2 = z2 dengan y genap adalah x = r2 s2, y = 2rs, z = r2 + s2


di mana r > s > 0, (r, s) = 1 dan paritas kedua bilangan r dan s berlawanan.

94

Soal-soal.

1.

Cari semua solusi dari x2 + y2 = z2 untuk 0 < z < 30

2.

Buktikan, jika x2 + y2 = z2 maka salah satu dari x atau y adalah kelipatan 3 dan salah
satu dari x, y atau z adalah kelipatan 5.

3.

Setiap solusi dari x2 + y2 = z2 disebut tripel Pythagoras. Cari semua tripel Pythagoras
yang unsur-unsurnya: a. membentuk deret tambah; b. membentuk deret ukur.

4.

Jika n 3 adalah sembarang bilangan bulat, tunjukkan terdapat tripel Pythagoras


dengan n sebagai salah satu unsurnya.

5.

Tentukan semua bilangan bulat positif n yang memenuhi x2 y2 = n.

6.

Buktikan setiap bilangan bulat n dapat dinyatakan dalam bentuk n = x2 + y2 z2.

4.6 Persamaan x4 + y4 = z2

Dalam usaha mencari solusi bulat positif dari persamaan x4 + y4 = z2, sama seperti
pembahasan Bagian 4.5, kita membatasi pada tripel solusi berupa tiga bilangan bulat
positif x, y, z yang memenuhi (x, y) = 1. Kita andaikan terdapat tripel solusi bulat positif x,

y, z yang memenuhi (x, y) = 1.


Karena persamaan x4 + y4 = z2 ekuivalen dengan bentuk (x2)2 + (y2)2 = z2, tripel x2,

y2, z adalah solusi dari bentuk persamaan yang dibahas di Bagian 4.5. Dalam hal ini bisa
dikenakan asumsi bahwa y2, jadi y juga, adalah bilangan bulat genap. Berdasarkan
Teorema 4.1 di Bagian 4.5, terdapat dua bilangan bulat positif u dan v yang berlawanan
paritas dan memenuhi

x2 = u2 v2,

y2 = 2uv,

z = u2 + v2,

u > v > 0,

(u, v) = 1.

Seandainya u genap, maka v ganjil dan akan diperoleh x2 0 1 3 (mod 4). Karena tak
ada x yang memenuhi, ditarik kesimpulan bahwa u ganjil dan v genap. Dengan demikian,

( 2y ) = u ( 2v ) ,

(u, 2v ) = 1.

Ini berarti ada bilangan-bilangan bulat r dan s yang memenuhi

u = r2 ,

( 2v ) = s ,
2

y = 2rs,

(r, s) = 1,

r > 0,

s > 0,

r ganjil.

Lebih jauh, dari x2 + v2= u2 diturunkan x2 + 4s4 = r4. Karena (r, 2s) = 1, hasil diskusi di
Bagian 4.5 bisa diterapkan pada persamaan terakhir (yang ekuivalen dengan persamaan)

x2 + (2s2)2 = (r2)2,

95

yaitu terdapat bilangan bulat positif dan yang berlawanan paritas dan memenuhi

x = 2 2 ,

2s2 = 2,

r 2 = 2 + 2 ,

> > 0,

(, ) = 1.

Karena s2 = , terdapat bilangan bulat > 0 dan > 0 sedemikian rupa sehingga = 2
dan = 2 dengan (, ) = 1. Dari sini diperoleh r2 = 4 + 4 yang terlihat sama dengan
bentuk awal persamaan.
Dalam usaha menyederhanakan persamaan, kita kembali ke bentuk persamaan
semula. Sekilas, tampaknya usaha kita sia-sia. Tetapi kita mungkin mendapatkan sesuatu.
Kita mengawali dengan tripel solusi x, y, z dan berakhir dengan tripel solusi r, , . Kita
barangkali telah mendapatkan solusi lain. Sesungguhnya

z = u2 + v2 = r4 + 4s4 > r4
sehingga z > r dan r, , adalah solusi yang lain. Lebih jauh, kita mendapatkan solusi z >

r > 0.
Jika kita mengulang proses di atas, tetapi diawali dari tripel solusi r, , ; kembali
akan diperoleh solusi baru r1, 1, 1 dengan r > r1. Dengan mengulang kembali proses di
atas, akan diperoleh barisan tripel solusi (r, , ), (r1, 1, 1), (r2, 2, 2), ... yang masingmasing bersesuaian dengan bilangan ri dengan r0 = r > r1 > r2 > ... > 0. Tetapi ri adalah
barisan tak hingga dari bilangan-bilangan bulat yang terus turun nilainya sehingga pada
suatu indeks k, rk 0, suatu kontradiksi. Jadi pengandaian bahwa ada solusi bulat positif

x, y, z yang memenuhi (x, y) = 1 harus diingkar.


Seandainya x, y, z adalah sembarang solusi dengan xy 0, maka |x|/g, |y|/g, |z|/g2,
di mana g = (x, y), merupakan tripel solusi bulat positif dengan (|x|/g, |y|/g). Hal yang
telah dibuktikan tidak mungkin terjadi. Jadi setiap tripel solusi x, y, z selalu memenuhi xy
= 0. Kita baru saja selesai membuktikan teorema berikut.
Teorema 4.2

Persamaani x4 + y4 = z2 hanya memiliki dua jenis tripel solusi yang hampir trivial:
x = 0, y, z = y2
atau

x, y = 0, z = x2.

Fakta bahwa x4 + y4 = z2 tak memiliki solusi bulat positif membawa pada suatu
kesimpulan bahwa persamaan x4 + y4 = z4 tak memiliki solusi bulat positif. Ini adalah

96

kasus khusus dari isi teorema yang sangat terkenal: Teorema Terakhir Fermat.
Catatan Bersejarah Tentang Teorema Terakhir Fermat

Teorema Terakhir Fermat yang pertama kali dipublikasikan pada tahun 1673
menyatakan bahwa persamaan

xn + yn = zn
tak memiliki solusi bulat untuk n > 2. Selama lebih dari tiga abad, tak ada yang bisa
membuktikan pernyataan Fermat secara penuh (walaupun sudah diberi nama Teorema),
kecuali bukti-bukti parsial.
Pada tahun 1993, Andrew Wiles dalam forum bergengsi Isaac Newton Institute

for Mathematical Sciences di London mengklaim telah berhasil membuktikan konjektur


Taniyama-Shimura yang salah satu implikasinya adalah terbuktinya pernyataan Fermat.
Tetapi beberapa kesalahan yang diketemukan oleh para penilai tulisan Andrew Wiles
membuatnya harus bekerja beberapa tahun lagi untuk memperbaikinya dalam dua tulisan:
"Modular elliptic curves and Fermat's Last Theorem" dan "Ring theoretic properties of
certain Hecke algebras" (ditulis bersama mantan muridnya Richard Taylor), keduanya
diterbitkan pada tahun 1995 dalam jurnal berprestise Annals of Mathematics.
Perlu dicatat, kedua tulisan Andrew Wiles di atas dianggap sebagai tulisan tingkat
tinggi dalam matematika yang memerlukan banyak alat dari berbagai cabang matematika
sehingga hanya bisa dimengerti oleh sedikit matematikawan di dunia. Teorema Terakhir
Fermat hanyalah merupakan salah satu akibat kecil yang bisa diturunkan dari hasil yang
diperoleh dari tulisan tersebut.
Soal-soal.

1. Untuk setiap bilangan bulat n 0 (mod 4), buktikan xn + yn = zn tak memiliki solusi
dengan xy 0.
2. Buktikan bahwa x4 + y4 = z2 tak memiliki solusi yang memenuhi xy 0. Petunjuk:
Gunakan metoda pembuktian yang digunakan di Bagian 4.5.
3. Buktikan x4 y4 = z2 tak memiliki solusi yang memenuhi yz 0.
4. Diberikan segitiga siku-siku dengan panjang sisi-sisinya bulat. Buktikan luas segitiga
tidak bisa berbentuk kuadrat dari suatu bilangan bulat.

97

4.7 Jumlah Dari Empat Kuadrat

Dalam diskusi di bagian ini, tidak dilakukan usaha pencarian solusi persamaan
Diophantine, melainkan hanya akan menunjukkan bahwa (Lagrange, 1770) persamaan

x12 + x22 + x32 + x42 = n


memiliki solusi bulat, apabila n Z+. Kesamaan aljabar berikut ditemukan Euler.
Lemma 4.1 5)

(x12 + x22 + x32 + x42)( y12 + y22 + y32 + y42)


= (x1 y1 + x2 y2 + x3 y3 + x4 y4)2 + (x1 y2 x2 y1 + x3 y4 x4 y3)2

(4.8)

+ (x1 y3 x2 y4 x3 y1 + x4 y2)2 + (x1 y4 + x2 y3 x3 y2 x4 y1)2

Bukti.
Kesamaan dibuktikan hanya dengan melihat hasil penjabaran di kedua ruas.
Kesamaan di atas menunjukkan bahwa apabila X dan Y bisa dinyatakan sebagai jumlah 4
buah kuadrat, maka hal yang sama berlaku untuk hasil kali XY. Dari hasil ini dan karena
1 = 12 + 02 + 02 + 02

dan

2 = 12 + 12 + 02 + 02,

cukup dibuktikan bahwa setiap bilangan prima ganjil p bisa dinyatakan sebagai jumlah
kuadrat 4 buah bilangan. Pembuktiannya dipecah atas dua bagian.
Teorema 4.3

Jika p > 2 prima, maka terdapat bilangan m, x1, x2, x3, x4 Z dengan 1 m < p yang
memenuhi
mp = x12 + x22 + x32 + x42.

(4.9)

Bukti.
Misalkan
S1 = {02, 12, 22, ,

( p2 1 )

dan

S2 = {02 1, 12 1, 22 1, , ( p2 1 ) 1}.
2

Karena x2 y2 (mod p) berakibat p|(x + y) atau p|(x y), maka tak ada dua unsur dalam S1
yang kongruen modulo p satu sama lain. Demikian pula, tak ada dua unsur dalam S2 yang
kongruen modulo p satu sama lain. Karena S1 S2 terdiri atas p + 1 unsur sedangkan

5)

Dengan menggunakan quaternionnya Hurwitz, x = x1 + x2i + x3j + x4 k dan y = y1 + y2i + y3j + y4 k,


Teorema 4.1 dapat dinyatakan dalam bentuk: |x| |y| = |xy|.

98

sistem residu lengkap modulo p hanya terdiri atas p unsur, maka jelas terdapat unsur x2
S1 yang kongruen modulo p dengan suatu unsur y2 1 S2. Dengan kata lain

x2 + y2 + 1 0 (mod p),
untuk suatu x, y {0, 1, 2, ,

( p2 1 ) }. Jadi, terdapat x, y {0, 1, 2, , ( p2 1 ) } dan m

Z sedemikian rupa sehingga x2 + y2 + 1 = mp. Ini berakibat

1m=

1
p

(x2 + y2 + 1)

( ( p2 1 ) + ( p2 1 ) +1) <
2

1
p

1
p

p2

( 2 +1) < p.
Terbukti, mp = x2 + y2 + 12 + 02 (mp adalah jumlah dari 4 kuadrat) dengan 1 m < p.
Walaupun banyaknya nilai m dalam teorema di atas mungkin lebih dari satu, tetapi nilai

m yang terkecil adalah 1, seperti yang dibuktikan dalam teorema berikut.


Teorema 4.4

Jika m Z+ adalah bilangan bulat positif terkecil yang memenuhi ekspresi (4.9)
dalam Teorema 4.3, maka m = 1.
Bukti.
Karena setiap himpunan bilangan-bilangan positif memiliki unsur terkecil, bilangan m
Z+ terkecil jelas ada. Seandainya bilangan m yang terkecil ini adalah bilangan genap,

maka mp = x12 + x22 + x32 + x42 juga genap sehingga hanya ada tiga kasus yang mungkin
terjadi: tidak ada, ada sebanyak dua di antaranya atau semua solusi x1, x2, x3 dan x4 adalah
genap.
Dalam kasus dua di antaranya genap, tanpa mengurangi berlaku umumnya bukti, kita
anggap x1 dan x2 genap sedangkan x3 dan x4 ganjil. Dengan demikian untuk semua kasus
berlaku x1 x2 dan x3 x4 adalah genap. Lebih jauh,

( x +2x ) + ( x 2x ) + ( x 2+ x ) + ( x 2 x ) = m2 p
1

sehingga m/2 memenuhi ekspresi (4.9) dalam Teorema 4.3, kontradiksi dengan ketentuan

m adalah bilangan bulat positif terkecil yang memenuhi sifat demikian.


Seandainya m > 1 ganjil, 3 m < p. Untuk setiap i {1, 2, 3, 4}, kita definisikan
bilangan yi yang memenuhi

yi xi (mod m)

dan

m21 yi m21

(4.10)

sehingga

99

y12 + y22 + y32 + y42 x12 + x22 + x32 + x42 (mod m).
Ini berarti

y12 + y22 + y32 + y42 = mn


untuk suatu n yang memenuhi
1 2
0 n 1p 4( m
2 ) < m.
Seandainya n 0, diperoleh y1 = y2 = y3 = y4 = 0 dan x1 x2 x3 x4 0 (mod m). Ini

berakibat

x12 + x22 + x32 + x42 0 (mod m2)


sehingga p 0 (mod m), kontradiksi dengan ketentuan 3 m < p. Di simpulkan, n > 0.
Dengan menggunakan Lemma 4.1,

m2np = (x12 + x22 + x32 + x42)( y12 + y22 + y32 + y42)

(4.11)

= A12 + A22 + A32 + A42


di mana Ai, i = 1, 2, 3, 4; adalah suku-suku di ruas kanan ekspresi (4.8). Dengan
menggunakan (4.10) mudah dibuktikan bahwa untuk setiap i {1, 2, 3, 4), Ai 0 (mod

m). Setelah (4.11) dibagi m2, didapat bentuk semacam ekspresi (4.9)
np =

(m) ( ) ( ) ( )
A1

+ m2

+ m3

+ m4

dengan 0 < n < m, kontradiksi dengan ketentuan m sebagai bilangan bulat positif terkecil
yang memenuhi ekspresi (4.9). Dari hasil ini ditarik kesimpulan bahwa m = 1.
Perlu dicatat bahwa ada bilangan bulat positif yang tidak bisa dinyatakan sebagai
jumlah sebanyak kurang dari empat kuadrat bilangan-bilangan bulat, misalnya bilangan 7
= 22 + 12 + 12 + 12. Dari fakta ini, dan hasil-hasil di atas diperoleh teorema berikut.
Teorema 4.5

Setiap bilangan bulat positif adalah jumlah dari empat kuadrat bilangan-bilangan
bulat dan tak ada jaminan bilangan bulat tersebut bisa dinyatakan sebagai jumlah
sebanyak kurang dari empat kuadrat bilangan-bilangan bulat.
Soal-soal.

Buktikan, tak ada bilangan bulat berbentuk 8k + 7 yang bisa ditulis sebagai jumlah tiga
buah kuadrat dari bilangan-bilangan bulat. Pertanyaan yang sama untuk setiap bilangan
bulat berbentuk 4m(8k + 7)
100

Daftar Pustaka
1. S.C. Coutinho, The Mathematics of Chipers: Number Theory and RSA Cryptography,
A.K. Peters, 1998.
2. G.H. Hardy, E.M. Wright, An Introduction to the Theory of Numbers, Oxford
University Press, Edisi Ke-4, 1975.
3. I. Niven, H.S. Zuckerman, H.L. Montgomery, An Introduction To The Theory of
Numbers, John Wiley & Sons, 5-th Edition, 2006.
4. V. Shoup, A Computational Introduction to Number Theory and Algebra (Version 2),
http://www.shoup.net/ntb/ (downloaded 10 Juni 2009)

101

Indeks
akar primitif
- modulo m, 58
algoritma Euklid, 8
perluasan -, 9
algoritma pembagian, 3
Teorema -, 3
asosiatif, 62
balikan, 62
bilangan
- kompleks, 15
- majemuk, 15
bilangan kompleks
modulus -, 15
bilangan majemuk, 13
bilangan prima, 13
distributif, 71
faktorisasi, 14, 16
fungsi Euler, 38
gelanggang, 71
grup, 62
- abel, 62
- hingga, 63
- kali, 67
- siklik, 69
- tak hingga, 63
- tambah, 66
habis dibagi, 2
habis membagi, 2
hukum pembatalan, 24,
69
indeks, 60
induksi matematis, 39
isomorf, 63

jarak
- dua bilangan prima,
19
juxtaposition, 66, 71
kelipatan persekutuan,
11
- terkecil, 11
keterbagian, 2
kongruen modulo m, 23
kongruensi, 23
- ax b, 33
- derajat 1, 32
- derajat 2, 56
- derajat n, 31
kuadratik
nonresidu -, 73
residu -, 73
lapangan, 71
Lemma Gauss, 74
masuk ke pangkat, 57
operator biner, 62
ordo, 63
- unsur, 70
pembagi persekutuan, 4
- terbesar, 4
perentang, 69
persamaan Diophantine,
89
quaternion Hurwitz, 98
representasi, 14
- kanonik, 14
- tunggal, 14
residu, 8

- pangkat ke-n, 57
kelas-kelas -, 65
resiprositi kuadratik, 78
resiprositi umum, 86
saling prima
relatif -, 7
sepasang-sepasang
relatif -, 7
simbol Jacobi, 83
simbol Legendre, 73
sistem residu
- tereduksi modulo p,
59
sistem residu lengkap
modulo m, 25
sistem residu tereduksi
modulo m, 25
skema, 49, 51
solusi primitif, 93
Teorema Dasar
Aritmatik, 17
Teorema Euklid, 19
Teorema Fermat, 26
- generalisasi, 26
Teorema Sisa Cina, 35
Teorema Terakhir
Fermat, 97
Teorema Wilson, 27
tertutup, 15, 62
unsur identitas, 62
vektor
panjang -, 15

You might also like